<<

Series Editor Series Editor Michael Laposata Diagnostic Standards of Care Michael Laposata Diagnostic Standards of Care Diagnostic Standards of Care Diagnostic Standards Transfusion Quality in Laboratory Diagnosis

Quentin G. Eichbaum, MD, PhD, MPH, MFA, FCAP, Assistant Dean for Program Development, Associate Professor of , Microbiology and , Vanderbilt University School of Medicine Garrett S. Booth, MD, MS, Assistant Professor of Pathology, Microbiology and Immunology, Vanderbilt University School of Medicine Transfusion Medicine Pampee P. Young, MD, PhD, Associate Professor of Pathology, Microbiology and Immunology, Vanderbilt University School of Medicine Transfusion The Diagnostic Standards of Care Series presents common errors associated with diagnoses in , using case examples to illustrate effective analysis based on current evidence and standards. In addition to being a practical diagnostic guide, each volume demonstrates the use of quality assurance and the role of the pathologist in ensuring Medicine quality and patient safety. Quality in Laboratory Diagnosis Transfusion Medicine addresses common issues and errors seen in the transfusion process. The goal is to alert general practitioners and trainees to the latest scientific advances and spiraling regulations in Eichbaum • Booth Young as well as introduce them to the most common types of transfusion errors encountered in clinical settings.

Transfusion Medicine Features

n Descriptions of potential errors in regulatory compliance, operational processes, and patient safety n Descriptions of potential errors in clinical decision making in blood transfusion, including when to premedicate patients, warfarin reversal, Quentin G. Eichbaum and the diagnostic intricacies in TRALI and TTP n Pocket-sized for portability Garrett S. Booth Pampee P. Young Recommended Shelving Category: Pathology

11 West 42nd Street New York, NY 10036 www.demosmedpub.com 9 781936 287963

Transfusion Medicine Quality in Laboratory Diagnosis

EEichbaum_87963_PTR_FM_10-01-12_i-xvi.inddichbaum_87963_PTR_FM_10-01-12_i-xvi.indd I 111/10/121/10/12 12:1912:19 PMPM Diagnostic Standards of Care

MICHAEL LAPOSATA, MD, PhD Series Editor

Coagulation Disorders Quality in Laboratory Diagnosis Michael Laposata, MD, PhD

Clinical Microbiology Quality in Laboratory Diagnosis Charles W. Stratton, MD

Laboratory Management Quality in Laboratory Diagnosis Candis A. Kinkus

Forthcoming in the Series /Immunology

EEichbaum_87963_PTR_FM_10-01-12_i-xvi.inddichbaum_87963_PTR_FM_10-01-12_i-xvi.indd iiii 111/10/121/10/12 12:1912:19 PMPM Diagnostic Standards of Care Series

Transfusion Medicine Quality in Laboratory Diagnosis

Quentin G. Eichbaum , MD, PhD, MPH, MFA, FCAP Assistant Dean for Program Development Associate Professor of Pathology, Microbiology and Immunology Director, Transfusion Medicine Fellowship Program Associate Professor of and Administration Vanderbilt University School of Medicine Associate Director of Transfusion Medicine Vanderbilt University Medical Center Nashville, Tennessee Garrett S. Booth, MD, MS Assistant Professor of Pathology, Microbiology and Immunology Vanderbilt University School of Medicine Associate Director of Transfusion Medicine Vanderbilt University Medical Center Nashville, Tennessee Pampee P. Young, MD, PhD Associate Professor of Pathology, Microbiology and Immunology Vanderbilt University School of Medicine Director of Transfusion Medicine Vanderbilt University Medical Center Nashville, Tennessee

New York

EEichbaum_87963_PTR_FM_10-01-12_i-xvi.inddichbaum_87963_PTR_FM_10-01-12_i-xvi.indd iiiiii 111/10/121/10/12 12:1912:19 PMPM Visit our website at www.demosmedpub.com

ISBN: 978-1-936287-96-3 e-book ISBN: 978-1-617051-60-9

Acquisitions Editor: Richard Winters Compositor: S4Carlisle Publishing Services

© 2013 Demos Medical Publishing, LLC. All rights reserved. This book is protected by copyright. No part of it may be reproduced, stored in a retrieval system, or transmitted in any form or by any means, electronic, mechanical, photocopying, recording, or otherwise, without the prior written permission of the publisher. Medicine is an ever-changing science. Research and clinical experience are continually expand- ing our knowledge, in particular our understanding of proper treatment and drug . The authors, editors, and publisher have made every effort to ensure that all information in this book is in accordance with the state of knowledge at the time of production of the book. Nevertheless, the authors, editors, and publisher are not responsible for errors or omissions or for any consequences from application of the information in this book and make no warranty, express or implied, with respect to the contents of the publication. Every reader should examine carefully the package in- serts accompanying each drug and should carefully check whether the dosage schedules mentioned therein or the contraindications stated by the manufacturer differ from the statements made in this book. Such examination is particularly important with drugs that are either rarely used or have been newly released on the market. Proudly sourced and uploaded by [StormRG] Kickass Torrents | TPB | ET | h33t

Library of Congress Cataloging-in-Publication Data Eichbaum, Quentin. Transfusion medicine : quality in laboratory diagnosis / Quentin Eichbaum, Garrett Booth, Pampee Young. p. ; cm. — (Diagnostic standards of care) Includes bibliographical references and index. ISBN 978-1-936287-96-3 ISBN 1-936287-96-X I. Booth, Garrett. II. Young, Pampee. III. Title. IV. Series: Diagnostic standards of care. [DNLM: 1. Blood Transfusion—adverse effects. 2. Hematologic —diagnosis. 3. Medical Errors—prevention & control. 4. Patient Safety—standards. 5. Quality Assurance, . WB 356] 615.3’90289—dc23 2012029073 Special discounts on bulk quantities of Demos Medical Publishing books are available to corporations, professional associations, pharmaceutical companies, health care organiza- tions, and other qualifying groups. For details, please contact: Special Sales Department Demos Medical Publishing , LLC 11 W. 42nd Street, 15th Floor New York, NY 10036 Phone: 800–532–8663 or 212–683–0072 Fax: 212–941–7842 E-mail: [email protected]

Printed in the United States of America by Gasch Printing. 12 13 14 15 / 5 4 3 2 1

Eichbaum_87963_PTR_FM_10-01-12_i-xvi.indd iv 11/10/12 12:19 PM QE: To the transfusionists and laboratory technologists in the developing world, who strive with limited resources to improve the quality and safety of blood transfusion.

GB: To the residents at Vanderbilt University Medical Center.

PY: To my mentors and my students from whom I have learned much.

EEichbaum_87963_PTR_FM_10-01-12_i-xvi.inddichbaum_87963_PTR_FM_10-01-12_i-xvi.indd v 111/10/121/10/12 12:1912:19 PMPM EEichbaum_87963_PTR_FM_10-01-12_i-xvi.inddichbaum_87963_PTR_FM_10-01-12_i-xvi.indd vivi 111/10/121/10/12 12:1912:19 PMPM Contents

Series Foreword xi Preface xi ii Acknowledgments xvi

1 Product-Related Errors 1

Inappropriate Use of (FFP) to Correct Mildly Elevated Prothrombin Time (PT) 2

Inappropriate Use of Fresh Frozen Plasma (FFP) for Volume Expansion 5

Inappropriate Use of Rh Immune Globulin (RhIG) in Pregnancy 7

Rh Immune Globulin (RhIG)—Inadequate Dosing 9

Inappropriate Use of 12

Platelet Inactivation as a Result of Cold Exposure 15

2 Errors in Procedures 19

A Positive Type and Screen Will Result in Relative Delay in the Issue of Blood 20

Error in Blood Sample Collection Resulting in Inaccurate Type and Screen 23

EEichbaum_87963_PTR_FM_10-01-12_i-xvi.inddichbaum_87963_PTR_FM_10-01-12_i-xvi.indd viivii 111/10/121/10/12 12:1912:19 PMPM viii Contents

Misinterpretation of Laboratory Tests for 25

ABO Typing Discrepancy Due to Less Common ABO Subgroups 28

Inappropriate Use of Autologous Blood 31

Rapid Transfusion in Chronic May Result in Volume Overload 34

Cold Agglutinin (CAD)—Insignifi cance of Low Antibody Titers 36

Hypocalcemic Toxicity from Therapeutic Plasma Exchange (TPE) 38

Hemolysis Following Transfusion 41

Development of Secondary to Intensive Therapeutic Plasma Exchange (TPE) 44

Heparin-Flushed Lines—Inadvertent Exposure to Blood Circulation 47

3 Errors Involving Specifi c Clinical Scenarios 51

Occult Anemia—Searching for a Delayed Hemolytic Transfusion Reaction 53

Identifi cation and Management of Clinically Signifi cant Alloantibodies in Pregnancy 56

Liberal Versus Restrictive Transfusion Strategies 59

Non-Evidence-Based Practices in Prophylactic Platelet Transfusions for Minor Procedures 61

Eichbaum_87963_PTR_FM_10-01-12_i-xvi.indd viii 11/10/12 12:19 PM Contents ix

Molecular Differences in the RhD Protein and the Need for Rh Immune Globulin 63

Recognition of Immune-Mediated Hemolysis in a Pediatric Patient 66

Inappropriate for Patients on Aspirin 70

Unexpected Posttransfusion Purpura (PTP) 73

Phenotype Matching to Mitigate Alloimmunization in 76

Refusing Blood Transfusion, When Patient and Beliefs Fail to Align 79

Crypt Antigen Activation, Adverse Consequences 82

Failure to Recognize Specifi c Risk Factors That Are Associated with Adverse Reactions in Blood Donors 85

Misinterpretation of the Direct Antiglobulin Test (DAT) 87

Warfarin Reversal—Inappropriate Use of Fresh Frozen Plasma 90

Anti-Kell Alloantibodies—Missed Diagnosis of Hemolytic Disease of the Fetus and Newborn (HDFN) 94

IgA Defi ciency—Misinterpretations and Assumptions 97

Misinterpreting the Cause of Hypotension during Transfusion 100

Inappropriate Application of Premedication for Transfusion 103

EEichbaum_87963_PTR_FM_10-01-12_i-xvi.inddichbaum_87963_PTR_FM_10-01-12_i-xvi.indd ixix 111/10/121/10/12 12:1912:19 PMPM x Contents

Incomplete Evaluation of Platelet Refractoriness 106

Thrombotic Thrombocytopenic Purpura (TTP)— Missed Diagnosis 109

Transfusion-Related Acute Lung Injury (TRALI)— Failure to Diagnose 112

Failure to Recognize That Serious, Potentially Fatal Hemolytic Transfusion Reactions Can Occur with Blood Products 115

Failure to Recognize Drug-Induced Hemolytic Anemia (DIHA) 118

Failure to Recognize That Donor HIV Exposure Is Associated with a Specifi c Federally Mandated Process for Recipient Notifi cation 121

Risk of Hyperkalemia from RBC Transfusion 124

Failure to Recognize That Intensive Plasma Exchange Can Cause Metabolic 127

Failure to Recognize the Risk of Alloimmune in a Primigravida 130

Unnecessary Platelet Transfusions 133

Index 137

EEichbaum_87963_PTR_FM_10-01-12_i-xvi.inddichbaum_87963_PTR_FM_10-01-12_i-xvi.indd x 111/10/121/10/12 12:1912:19 PMPM Series Foreword

“Above all, do no harm.” This frequently quoted admonition to healthcare providers is highly regarded, but despite that, there are few books, if any, that focus primarily on how to avoid harming patients by learning from the mistakes of others. Would it not be of great benefi t to patients if all healthcare pro- viders were aware of the thrombotic consequences from heparin- induced thrombocytopenia before a patient’s leg is amputated? The clinically signifi cant, often lethal, thrombotic events that occur in patients who develop heparin-induced thrombocytopenia would be greatly diminished if all healthcare providers appropriately moni- tored platelet counts in patients being treated with intravenous unfractionated heparin. It was a desire to learn from the mistakes of others that led to the concept for this series of books on diagnostic standards of care. As the test menu in the clinical laboratory has enlarged in size and complexity, errors in selection of tests and errors in the interpreta- tion of test results have become commonplace, and these mistakes can result in poor patient outcomes. This series of books on diag- nostic standards of care in , microbiology, transfusion medicine, hematology, clinical chemistry, immunology, and labo- ratory management are all organized in a similar fashion. Clinical errors, and accompanying cases to illustrate each error, are presented within all of the chapters in several discrete categories: errors in test selection, errors in result interpretation, other errors, and diagnostic controversies. Each chapter concludes with a summary list of the standards of care. The most common errors made by thousands of

Eichbaum_87963_PTR_FM_10-01-12_i-xvi.indd xi 11/10/12 12:19 PM xii Series Foreword

healthcare providers daily are the ones that have been selected for presentation in this series of books. Practicing ordering tests with which they are less familiar would benefit significantly by learning of the potential errors associated with ordering such tests and errors associated with interpreting an infrequently encountered test result. Medi- cal trainees who are gaining clinical experience would benefit significantly by first understanding what not to do when it comes to ordering laboratory tests and interpreting test results from the clinical laboratory. Individuals working in the clinical laboratory would also benefit by learning of the common mistakes made by healthcare providers so that they are better able to provide helpful advice that would avert the damaging consequences of an error. Finally, laboratory managers and hospital administrators would benefit by having knowledge of test ordering mistakes to improve the efficiency of the clinical laboratory and avoid the cost of per- forming unnecessary tests. If the errors described in this series of books could be greatly reduced, the savings to the healthcare system and the improve- ment in patient outcomes would be dramatic.

Michael Laposata, MD, PhD Series Editor

EEichbaum_87963_PTR_FM_10-01-12_i-xvi.inddichbaum_87963_PTR_FM_10-01-12_i-xvi.indd xiixii 111/10/121/10/12 12:1912:19 PMPM Preface

Blood transfusion is the most frequently performed medical proce- dure in the United States. Yet patient safety in transfusion remains a serious concern. Since the inception of blood banking more than 70 years ago, countless lives have been saved through transfusion. For many years, transfusion was considered a generally “benign” procedure not requiring close quality control or clinical supervi- sion. Many blood banks and transfusion services seemingly operate on virtual “auto-pilot” with technologists diligently issu- ing blood products without legitimately querying any such requests. There is often scant clinical supervision on the wards over the transfu- sion process. Adverse events are therefore quite likely underestimated and underreported. The transfusion of blood has been metaphorically and cultur- ally associated with “giving life.” For many providers and recipients of blood, “more” blood was therefore considered “better.” We now know this not to be true. Years of clinical experience and careful research have taught us that transfusion is not an entirely “benign” procedure, but is essentially “invasive” and associated with a multi- tude of potentially adverse events. As one of the cases in this book illustrates, more “conservative” transfusion thresholds are often safer. With the growth of complex new in medicine, the demand for blood products and derivatives has also burgeoned. Along with this trend, the scope of transfusion medicine as a discipline has also grown rapidly. Advances in research and medical technology have made available different preparations of blood products and a broader array of blood derivatives. Molecular biology and new pro- tein purifi cation methods have permitted the cloning and purifi cation of proteins such as recombinant coagulation factors, IVIG, and RhIG that have proved to be life-saving in hemolytic disorders such as sickle

EEichbaum_87963_PTR_FM_10-01-12_i-xvi.inddichbaum_87963_PTR_FM_10-01-12_i-xvi.indd xiiixiii 111/10/121/10/12 12:1912:19 PMPM xiv Preface

cell disease and hemolytic disease of the fetus and newborn (HDFN). Advances in technology, such as the and photopheresis instruments, have similarly improved survival rates in trans- plants, and have provided for leukemias and for dealing with graft-versus-host disease, a comorbidity of transplantation. But with this growing complexity has also come the increased potential for transfusion errors. The consequence of error is increased morbidity and mortality that ultimately translates into escalating healthcare costs. Transfusion errors lead to increased wastage of blood that further raises costs and creates shortages of a precious pub- lic good. The FDA and CDC have only recently implemented proac- tive hemovigilance programs to more accurately detect and describe adverse outcomes in transfusion so as to preempt such outcomes and avert errors, and to develop therapeutic and supportive strategies. To deal with this landscape of medical error, blood banking and transfusion practice have today become the most regulated sections in laboratory medicine. Transfusion specialists and medical tech- nologists are compelled to not only keep abreast of the scientifi c advances in their fi eld but also to keep pace with spiraling regu- lations. The and trainee may be oblivious to these ongoing advances and regulations. This lack of knowledge may further feed the potential for error. The goal of this book is to educate physicians in other subspecialties about the most common types of transfusion errors they are likely to encounter in various clinical scenarios. Medical practitioners will frequently rely on “handbooks” to keep themselves abreast of knowledge outside their own specialty. Such handbooks usually just provide a compendium of the fi eld’s basic knowledge but do not alert the practitioner to potential errors. Our book is designed to approach the topic of transfusion from this pragmatic angle of how to avoid errors. To emphasize, our focus is on errors commonly encountered rather than on the more complicated multifaceted errors that involve not only clinicians but also the laboratory and the health delivery system. Over many years of interacting with clinical teams on vari- ous medical services and holding daily medical rounds in transfusion medicine, we have become aware of repeated themes in transfusion

EEichbaum_87963_PTR_FM_10-01-12_i-xvi.inddichbaum_87963_PTR_FM_10-01-12_i-xvi.indd xivxiv 111/10/121/10/12 12:1912:19 PMPM Preface xv

errors. We have divided this book into three sections relating to errors in (1) blood products, (2) transfusion procedures, and (3) clinical transfusion scenarios. The products section is the shortest because blood products are quality controlled in the blood bank, and the er- rors that occur here would pertain mostly to medical technologists and transfusion specialists. The clinical scenarios section is the meat of the book because this is where the practitioner would most likely be implicated. While most of the errors we describe are clear-cut, a few verge closer toward the margins of controversy—for instance, decisions about when to premedicate patients, strategies of warfarin reversal, and the diagnostic intricacies in TRALI and TTP. Our hope is that this book will be of value to practitioners and trainees in helping them understand what not to do in a blood transfu- sion. While this approach may skip some foundational knowledge in transfusion medicine, it has the advantage of ultimately improving the safety of blood transfusion for patients.

Quentin G. Eichbaum, MD, PhD, MPH, MFA, FCAP Garrett S. Booth, MD, MS Pampee P. Young, MD, PhD

EEichbaum_87963_PTR_FM_10-01-12_i-xvi.inddichbaum_87963_PTR_FM_10-01-12_i-xvi.indd xvxv 111/10/121/10/12 12:1912:19 PMPM Acknowledgments

We would like to thank Richard Winters, Executive Editor, Demos Publishing, for his editorial advice and for his patience in waiting for us to complete this volume in the Quality in Laboratory Diagnosis Series; Dr. Michael Laposata, MD, PhD, Series Editor, for his con- stant encouragement, expert advice, and judicious editing skills; and Ms. LeAnne Caston for her assistance in preparing the manuscript.

EEichbaum_87963_PTR_FM_10-01-12_i-xvi.inddichbaum_87963_PTR_FM_10-01-12_i-xvi.indd xvixvi 111/10/121/10/12 12:1912:19 PMPM 1 Product-Related Errors

Inappropriate Use of Fresh Frozen Plasma (FFP) to Correct Mildly Elevated Prothrombin Time (PT) 2 Inappropriate Use of Fresh Frozen Plasma (FFP) for Volume Expansion 5 Inappropriate Use of Rh Immune Globulin (RhIG) in Pregnancy 7 Rh Immune Globulin (RhIG)—Inadequate Dosing 9 Inappropriate Use of Cryoprecipitate 12 Platelet Inactivation as a Result of Cold Exposure 15

Eichbaum_87963_PTR_CH01_10-01-12_001-018.indd 1 11/10/12 12:51 PM 2 1. Product-Related Errors

INAPPROPRIATE USE OF FRESH FROZEN PLASMA (FFP) TO CORRECT MILDLY ELEVATED PROTHROMBIN TIME (PT)

OVERVIEW

In patients scheduled for minimally invasive surgical procedures, whose PT or international normalized ratio (INR) is only slightly elevated, the concern for signifi cant bleeding may be unwarranted, and there is generally no need to transfuse FFP to replenish coagulation factors. When the PT/INR is only mildly elevated, infusion of FFP to replenish coagulation factors will have little impact on further lowering the PT/INR, due to the physiologic reserve of these coagulation factors.

Patients may be unnecessarily transfused with FFP in an attempt to decrease a slightly elevated PT/INR back into the normal range. Such transfusion of FFP may, moreover, have the unwanted conse- quence of precipitating an adverse event in the form of a transfusion reaction. Potential adverse events to transfusion of FFP include vol- ume overload, allergic reactions, transfusion-related lung injury, and transmission of infectious agents.

Case with Error

A 65-year-old male with unexplained jaundice is scheduled for a liver biopsy. Reviewing his chart, the surgeon notices that his PT is 16.4 seconds (elevated) and partial thromboplastin time (PTT) is 32 seconds (normal). He calls the internist and suggests that they infuse 2 units of FFP before he is willing to perform the procedure. Two units of FFP are infused, and a “stat” PT test is ordered. The result is 16 seconds. The surgeon is not satisfi ed and suggests trans- fusing another 2 units of FFP. Halfway through infusion of the second unit, the patient develops a fever of 39 o C from his baseline of 37o C.

EEichbaum_87963_PTR_CH01_10-01-12_001-018.inddichbaum_87963_PTR_CH01_10-01-12_001-018.indd 2 111/10/121/10/12 12:5112:51 PMPM Standards of Care 3

His blood pressure is 158/90, pulse 94, and oxygen saturation on room air 90%. The transfusion is stopped, and an adverse reaction report is submitted. The procedure has to be rescheduled.

Clinical Pitfall

Failure to recognize that coagulation screening tests can be poor pre- dictors of bleeding and that use of FFP to lower a minimally elevated PT into the normal range may be counterproductive. The surgeon’s insistence on having the PT within the normal range (<12 seconds) before proceeding with the liver biopsy is unwarranted and has led to the unnecessary transfusion of several units of FFP. Transfusion of 4 units of FFP has had only minimal impact in decreasing the PT.

Explanation and Consequences

A PT of 16.4 seconds is only slightly elevated and should not have required transfusion of 4 units of FFP to prevent bleeding in this minimally invasive procedure. Coagulation screening assays can be poor predictors of bleeding when results are only mildly elevated. The transfusion of 4 units of FFP failed to lower the PT into the normal range. Moreover, the unnecessary infusion of FFP in this patient led to rescheduling the procedure.

STANDARDS OF CARE

Coagulation screening test results should not be too conservatively interpreted, but should be assessed in the setting of the hemostatic defect, the patient’s underlying condition, the procedure to be performed, and the likelihood of bleeding. A slightly elevated PT/INR (12–17 seconds; INR 1.0–1.7) is usu- ally not a cause for concern in a patient undergoing a minimally invasive or bedside procedure, who is not bleeding, and who has no history of excess bleeding or bruising.

EEichbaum_87963_PTR_CH01_10-01-12_001-018.inddichbaum_87963_PTR_CH01_10-01-12_001-018.indd 3 111/10/121/10/12 12:5112:51 PMPM 4 1. Product-Related Errors

RECOMMENDED READING

Chowdhury P, Saayman AG, Paulus U, et al. Effi cacy of standard dose and 30 ml/kg fresh frozen plasma in correcting laboratory parameters of haemostasis in critically ill patients. Br. J. Haematol. 2004;125:69–73. O’Shaughnessy DF, Atterbury C, Bolton Maggs P, et al. Guidelines for the use of fresh frozen plasma, cryoprecipitate and . Br. Soc. Haematol. 2004;126:11–28.

Eichbaum_87963_PTR_CH01_10-01-12_001-018.indd 4 11/10/12 12:51 PM Inappropriate Use of FFP for Volume Expansion 5

INAPPROPRIATE USE OF FRESH FROZEN PLASMA (FFP) FOR VOLUME EXPANSION

OVERVIEW

Plasma is used primarily for the purpose of preventing bleeding and to treat hemorrhage in patients with acquired or congenital coagulation defects. Besides FFP, other plasma products are also available, including plasma frozen within 24 hours of phlebotomy (FP24), which is often used interchange- ably with FFP; thawed plasma (derived from FFP or FP24 that has been thawed and kept at 1–6o C and can be stored for up to 5 days); and cryoprecipitate-reduced plasma that consists of the supernatant that is removed when cryoprecipitate is made from FFP.

Appropriate indications for the use of plasma products include coag- ulation factor replacement in congenital factor defects where factor concentrates are unavailable, massive transfusion, plasma exchange transfusions, reversal of warfarin anticoagulation in the setting of severe bleeding, and treatment of disseminated intravascular coagula- tion. Plasma products should not be used for volume expansion, as a source of nutrients, to treat immunodefi ciency, or to promote . As with other blood products, administration of plasma prod- ucts may be associated with adverse reactions, so non-medically indi- cated usage should be carefully avoided.

Case with Error

A 56-year-old male patient with traumatic injury, including third-degree burns from a motor vehicle collision, undergoes a massive transfusion in the operating room (OR). He is eventually stabilized and returned to the intensive care unit. His blood pressure is low, and he appears vol- ume depleted, although there is no overt bleeding and his PT and PTT are within normal limits. The doctor considers infusing crystalloid, but

EEichbaum_87963_PTR_CH01_10-01-12_001-018.inddichbaum_87963_PTR_CH01_10-01-12_001-018.indd 5 111/10/121/10/12 12:5112:51 PMPM 6 1. Product-Related Errors

then decides that FFP might additionally provide essential biological factors for a patient who has suffered burns. The doctor orders 2 units of FFP, which are infused rapidly over 30 minutes. The patient’s blood pressure improves, but as the second unit is nearing completion, he becomes short of breath and develops a fever of 39.5 oC (his pretrans- fusion temperature was 37o C).

Clinical Pitfall

Failure to understand the appropriate clinical usage of plasma products. Plasma products are used to prevent bleeding and to treat acquired and congenital coagulation defects. They should not be used for vol- ume expansion or as a source of nutrients. Blood products are asso- ciated with adverse events, and their misuse can precipitate such an event and complicate treatment of an underlying condition.

Explanation and Consequences

This patient’s volume depletion could more appropriately have been treated with crystalloid, nonprotein colloids, or with 5% albumin. The unnecessary infusion of FFP as a volume expander and source of nutrients appears to have triggered an adverse event.

STANDARDS OF CARE

Volume depletion should generally be treated with normal saline or crystalloid, and not with plasma or other blood products. Plasma products should be used to prevent bleeding or to treat acquired and congenital coagulation defects.

RECOMMENDED READING

O’Shaughnessy DF, Atterbury C, Bolton Maggs P, et al. British Committee for Standards in Haematology, Blood Transfusion Task Force. Guidelines for the use of fresh frozen plasma, cryoprecipitate and cryosupernatant. Br. Soc. Haematol . 2004;126:11–28.

EEichbaum_87963_PTR_CH01_10-01-12_001-018.inddichbaum_87963_PTR_CH01_10-01-12_001-018.indd 6 111/10/121/10/12 12:5112:51 PMPM Inappropriate Use of Rh Immune Globulin (RhIG) in Pregnancy 7

INAPPROPRIATE USE OF Rh IMMUNE GLOBULIN (RhIG) IN PREGNANCY OVERVIEW

The use of pooled, human-derived immunoglobulins directed against the RhD antigen (RhIG) is a success story of modern immunohematology and . Prior to the 1970s, hemolytic disease of the fetus and newborn (HDFN) was a common clinical problem, with considerable neonatal morbid- ity and mortality. Previous treatments, including exchange trans- fusions and phototherapy, were both risky and fi nancially cumbersome. With the advent of routine prophylactic adminis- tration of RhIG at 28–30 weeks gestation, and again at the conclusion of pregnancy, alloimmunization to RhD decreased by 90%, as did the incidence of HDFN.

Case with Error

The transfusion medicine service is consulted on a maternal-fetal blood screen and the dosage of RhIG prophylaxis to administer to a 34-year-old Hispanic G5P4 female ( A Rh-negative) who is postoperative day one following a Cesarean-section delivery of a healthy Rh-positive male baby. Throughout the woman’s pregnancy, the transfusion medicine service has been performing routine anti- body titers to the RhD antigen. The titers have ranged from <1 to 4. Multiple antibody screens performed through her gestational period have demonstrated only this single alloantibody (anti-D).

Clinical Pitfall

Failure to recognize the appropriate use of RhIG. Correctly identifying which patients require RhIG is essential in preventing alloimmunization in an Rh-negative woman carrying an Rh-positive baby. It is important to remember that these interventions

Eichbaum_87963_PTR_CH01_10-01-12_001-018.indd 7 11/10/12 12:51 PM 8 1. Product-Related Errors

are not without risk, including possible infectious disease transmission. In patients who have previously been sensitized to the RhD antigen, either by prior pregnancy and/or by transfusion, RhIG prophylaxis is not indicated.

Explanation and Consequences

For patients who are typed as Rh-negative with a concurrent negative antibody screen, there is a role for RhD alloimmunization prophylaxis. Fetal red blood cells (RBCs) can traverse the placenta and enter the maternal circulation (fetal-maternal hemorrhage), thereby sensitizing the mother to a foreign RBC antigen. Antibodies against the RhD antigen (anti-D) are capable of crossing the placenta, where they can ultimately cause fetal RBC destruction, decreased fetal tissue oxygenation, and, most concerning, hydrops fetalis and fetal demise. However, once a patient has made an alloantibody against RhD, there is no longer a chance of prevent- ing the antibody from developing. There is thus no utility in giving RhIG to patients who have developed an anti-D alloantibody.

STANDARDS OF CARE

Both AABB and ACOG recommend routine ABO/Rh typing of pregnant females, with additional intervention at 28–30 weeks for those patients who type as Rh-negative and with a negative anti- body screen. Additional RhIG should be administered at the time of delivery, or in cases where the fetal-maternal blood barrier has been disrupted (ie, amniocentesis).

RECOMMENDED READING

Fung Kee Fung K, Eason E, Crane J, et al. Prevention of Rh alloim- munization. J. Obstet. Gynaecol. Can . 2003;25:765–773. Klein HG, Anstee DJ. Haemolytic disease of the fetus and newborn. In: Klein HG, Anstee DJ, eds. Mollison’s Blood Transfusion in Clini- cal Medicine . 11th ed. Hoboken, NJ: Wiley-Blackwell; 2005:496–545.

EEichbaum_87963_PTR_CH01_10-01-12_001-018.inddichbaum_87963_PTR_CH01_10-01-12_001-018.indd 8 111/10/121/10/12 12:5112:51 PMPM Rh Immune Globulin (RhIG)—Inadequate Dosing 9

Rh IMMUNE GLOBULIN (RhIG)— INADEQUATE DOSING

OVERVIE W

Pregnant Rh-negative females, carrying an Rh-positive baby, who experience a fetomaternal hemorrhage (FMH), of even just a few milliliters of blood, are at increased risk for alloimmunization to the RhD antigen unless they receive an ade- quate dose of RhIG. The volume of blood causing anti-D alloim- munization varies among patients and appears to be related to factors such as the immunologic responsiveness of the mother and the immunogenicity of the Rh-positive RBCs. The rosette test serves as the initial screen for the presence of FMH. The volume of FMH (percentage of fetal red cells in the maternal circulation) is then determined by the Kleihauer-Betke test (an acid elution test) or, more precisely and reliably, by fl ow cytometry. Combinations of these tests can also be used to identify and quantify such hemorrhage. RhIG provides prophylaxis to prevent alloimmunization to the RhD blood group antigen in Rh-negative patients exposed to Rh-positive RBCs during transfusion, placental bleeding, or pregnancy. The mechanism of action of RhIG remains unclear, but the correct level of dosing has been empirically determined and is important to prevent alloimmunization. The appropriate dose of RhIG is determined by a calcula- tion that takes into account the percentage of fetal RBCs in the maternal circulation and the mother’s blood volume. Inadequate dosing of RhIG may fail to protect the mother from anti-D allo- immunization, which may result in hemolytic disease of the newborn in subsequent pregnancies.

EEichbaum_87963_PTR_CH01_10-01-12_001-018.inddichbaum_87963_PTR_CH01_10-01-12_001-018.indd 9 111/10/121/10/12 12:5112:51 PMPM 10 1. Product-Related Errors

Case with Error

A 26-year-old G1P0 female in her 22nd week of pregnancy is involved in a motor vehicle collision and is taken to a local hospital for treat- ment of minor injuries. Soon after arrival she has a spontaneous abor- tion. The obstetrics resident-on-call notes from her blood type and screen that she is Rh-negative and immediately administers to her a 300 μg standard dose of RhIG. A year later she returns to the hospital pregnant again. Her obstetri- cian is surprised that she now shows an anti-D antibody despite having previously been treated with RhIG when she had her spontaneous abor- tion the year before. The obstetrician follows the trend of her anti-D titer, which escalates to a high titer of 1 to 64 in her third trimester, resulting in fetal anemia.

Clinical Pitfall

Failure to correctly assess the amount of FMH and to administer the appropriate dose of RhIG prophylaxis to avert anti-D alloimmuniza- tion in an Rh-negative mother carrying an Rh-positive fetus. The provider appears to have administered too low a dose of RhIG. The resident has failed to correctly assess the volume of FMH by fi rst screening for FMH using the rosette test, and then quantitating the volume of FMH by the Kleihauer-Betke test or fl ow cytometry, before issuing the appropriate dose of RhIG.

Explanation and Consequences

A standard recommendation for RhIG dosing is 300 μg per 30 mL of fetal or 15 mL of fetal RBCs present in the maternal cir- culation. The resident in this case administered a 300 μg dose of RhIG to the mother following her abortion, without calculating the volume of FMH by the Kleihauer-Betke test or by fl ow cytometry. It is quite possi- ble that the volume of FMH during her abortion may have been greater than 30 mL and that she was inadequately dosed with RhIG. Alloim- munization to the RhD antigen was therefore not prevented. During her next pregnancy, this maternal alloantibody crossed the placenta in high titers to coat and lyse the fetal RBCs, leading to the fetal anemia.

Eichbaum_87963_PTR_CH01_10-01-12_001-018.indd 10 11/10/12 12:51 PM Recommended Reading 11

STANDARDS OF CARE

When FMH occurs during routine delivery, or is suspected as a consequence of placental bleeding, a sample of maternal blood should be collected for FMH testing within an hour of the event. The rosette test is performed to screen for FMH and, if positive, is suc- ceeded by either the Kleihauer-Betke test or, preferably, by the more sensitive fl ow cytometric testing, to quantify the volume of FMH. The dose of RhIG administered to the mother is calculated by giv- ing 300 μg vial doses of RhIG per 30 mL of fetal whole blood, or per 15 mL of fetal RBCs, in the maternal circulation (Note: other dose sizes of RhIG are also available). Calculation for RhIG dosing: ✓ Maternal blood volume (mL) = 70 mL/kg × maternal weight (kg) [ use 5,000 mL if maternal weight is not known ] ✓ Volume of fetal bleed (mL) = Percentage of fetal RBCs × maternal blood volume ✓ Dose of RhIG (300 mg vials) to administer = fetal bleed (mL)/30 mL of whole blood (or 15 mL of RBCs ) ✓ If the number to the right of the decimal point is <5, round the number down and add one additional dose of RhIG (eg, 3.4 S 3 + 1 = 4); if the number to the right of the decimal >5, round up and add an additional dose of RhIG (eg, 2.8 S 3 + 1 = 4)

RECOMMENDED READING

Judd WJ. Practice guidelines for prenatal and perinatal immunohema- tology, revised. Transfusion 2001;41:1445–1452.

EEichbaum_87963_PTR_CH01_10-01-12_001-018.inddichbaum_87963_PTR_CH01_10-01-12_001-018.indd 1111 111/10/121/10/12 12:5112:51 PMPM 12 1. Product-Related Errors

INAPPROPRIATE USE OF CRYOPRECIPITATE

OVERVIEW

The appropriate use of cryoprecipitate is highly variable. In part, this is due to a misunderstanding of what this contains. However, there is also a dearth of evi- dence from randomized controlled trials about appropriate usage. Its indiscriminate usage, as evident from numerous blood bank audits, appears to be driven by the misguided belief that it represents a “super concentrated” form of FFP. This is not true, as cryoprecipitate has a different composition than FFP.

Case with Error

A 55-year-old female with a history of congestive heart failure reports to the emergency department directly from her warfarin clinic. She has a printed report indicating that her morning INR laboratory result is 9.5. She is clinically stable with no signs of bleeding. She is neu- rologically intact but has an audible cardiac murmur best appreciated over the right sternal border. The resident admitting her into the emer- gency department requests 2 units of type AB plasma (“stat”) in an effort to emergently reverse the effect of her warfarin anticoagulation. However, the attending physician does not want to cause an exacerba- tion of her underlying cardiac dysfunction, and therefore decides to administer cryoprecipitate because of its smaller volume.

Clinical Pitfall

Failure to recognize the appropriate usage of cryoprecipitate and to calculate the correct dose. Given the patient’s stable status and with no eminent planned, emergent warfarin reversal is not necessary. Moreover, the use of cryoprecipitate is not indicated. The resident physician failed to understand that cryoprecipitate is not a concentrated form of FFP and

EEichbaum_87963_PTR_CH01_10-01-12_001-018.inddichbaum_87963_PTR_CH01_10-01-12_001-018.indd 1212 111/10/121/10/12 12:5112:51 PMPM Standards of Care 13

that there are no vitamin K-dependent coagulation factors in cryopre- cipitate. These applications of cryoprecipitate transfusion offer little clinical utility. To the contrary, they result in additional donor expo- sures, with concomitant infectious and noninfectious disease risks. Misuse of cryoprecipitate is also encountered in the OR, where it is erroneously believed to aid in surgical hemostasis, even when fi brino- gen levels are normal. Inappropriate ordering and issuing of cryopre- cipitate is an example of blood product mismanagement.

Explanation and Consequences

Cryoprecipitate is a concentrate of high-molecular-weight plasma proteins that precipitate in the cold when FFP is thawed at 4°C. This concentrate contains a limited spectrum of proteins: fi brinogen, von Willebrand factor (vWF), factor VIII, factor XIII, and fi bronec- tin. This product has a shelf life of 1 year, when frozen at −18°C. Cryoprecipitate is most often used to treat patients with hypofi brino- genemia (usually fi brinogen ≤100 mg/dL [1.00 g/L]) or dysfi brino- genemia. Rare cases of factor XIII defi ciency can be treated with either cryoprecipitate or FFP, or prophylactically with commercially available factor XIII purifi ed from human plasma (eg, Corifact).

STANDARDS OF CARE

As per AABB Standar ds, cryoprecipitate contains specifi ed amounts per unit of fi brinogen (minimum of 150 mg) and factor VIII (mini- ,(IU 80ف) mum 80 IU). It also contains factor XIII (40–60 IU), vWF and fi bronectin (40–60 IU). It is used primarily in the control of bleeding associated with fi brinogen defi ciency and in the treatment of factor XIII defi ciency, but can also be used as a second-line ther- apy for von Willebrand disease and hemophilia A. The number of unit bags of fi brinogen to administer, based on level of fi brinogen desired, is calculated as follows: 1. Calculate the total blood volume: • Body weight (kg) × 70 mL/kg 2. Calculate the total plasma volume: • Total blood volume × (1 − hematocrit)

Eichbaum_87963_PTR_CH01_10-01-12_001-018.indd 13 11/10/12 12:51 PM 14 1. Product-Related Errors

3. Calculate the milligrams of fi brinogen needed: • (Total plasma volume) × (concentration change in fi brinogen desired): • The change in fi brinogen level desired is determined by subtracting the current level from the desired level of fi brinogen; for example, if the desired level is 175 mg/dL and the current fi brinogen level is 50 mg/dL: 175 mg/dL −50 mg/dL = 125 mg/dL • Multiply the change in concentration times the total plasma volume, but divide the answer by 100 to correct the units (dL to mL) 4. Calculate the number of bags needed to reach the desired fi brinogen level: • Number of bags needed/150 mg per bag

RECOMMENDED READING

Ansell J, Hirsh J, Hylek E, et al. Pharmacology and management of the vitamin K antagonists. Chest 2008;133(suppl):160S–198S. Stehling LC, Doherty DC, Faust RJ, et al. Practice Guidelines for Blood Component Therapy: A Report by the American Society of Anesthesiologists Task Force on Blood Component Therapy. Anesthe- siology 1996;84:732–747. Fresh-Frozen Plasma, Cryoprecipitate, and Administration Practice Guidelines Development Task Force of the College. Practice parameter for the use of fresh-frozen plasma, cryoprecipitate, and platelets. JAMA 1994;271:777–781.

EEichbaum_87963_PTR_CH01_10-01-12_001-018.inddichbaum_87963_PTR_CH01_10-01-12_001-018.indd 1414 111/10/121/10/12 12:5112:51 PMPM Platelet Inactivation as a Result of Cold Exposure 15

PLATELET INACTIVATION AS A RESULT OF COLD EXPOSURE

OVERVIEW

Whole blood and blood components must be stored and handled in accordance with regulatory standards to main- tain therapeutic potency and ensure the safety of patients. Whereas packed red cells and plasma are stored and transported at cold temperatures (ranging from 1–6o C), platelets are required to be maintained at room temperature, between 20 oC and 24 o C. Exposure of platelets, even briefl y, to temperatures under 15o C renders them inactive and unsuitable for clinical use.

Case with Error

An aneurysm repair is scheduled for a 73-year-old male patient who is otherwise healthy. The preprocedure complete blood count is within normal parameters. The perioperative medical team requests 6 units of RBCs and 3 units of FFP to be sent to the operating room (OR) prior to the start of the procedure. The RBC and FFP units are sent in a cooler packed with ice. During the procedure, there is signifi cant bleeding, and 7 additional units of RBCs and 3 additional units of FFP are requested. Two units of platelets are also requested. These products are sent separately via the pneumatic tube to the OR. The aneurysm repair is completed soon thereafter. An OR staff member is asked to return all unused products to the blood bank. For convenience, the staff member places the unused platelet product in the cooler together with the other components and returns them to the blood bank. The returned RBC and FFP units are suitable for reissue as their tempera- ture is measured at 5o C. The platelets had to be discarded.

EEichbaum_87963_PTR_CH01_10-01-12_001-018.inddichbaum_87963_PTR_CH01_10-01-12_001-018.indd 1515 111/10/121/10/12 12:5112:51 PMPM 16 1. Product-Related Errors

Clinical Pitfall

Failure to keep platelet products at appropriate temperature. It is usual for surgeons to request packed RBCs, plasma, and plate- lets in advance to preempt the need to resuscitate an acutely bleeding patient during surgery. The blood bank accommodates these requests by packing RBCs and FFP in a cooler, and platelets in a separate room- temperature carrier. If these products are not utilized, they should be returned to the blood bank. Not infrequently, the platelets are inad- vertently placed in the cooler with the other unused blood products prior to return of unused products to the blood bank. As a result, the platelets are cooled to temperatures at which they can no longer be used, and are thus wasted.

Explanation and Consequences

Platelets need to be stored at room temperature (20–24 oC). Even a brief exposure of platelets to temperatures under 15 oC leads to their increased clearance from circulation following transfusion. Recently it has been demonstrated that cooling platelets leads to the irrevers- ible reorganization of the von Willebrand factor receptor (GPIb) clusters on the platelet membrane, resulting in their enhanced clear- ance from the circulation by the reticuloendothelial system. None- theless, room temperature is still not an ideal temperature because platelets lose hemostatic function faster at this temperature, and bacterial growth is also facilitated. Bacterial contamination and the ensuing clinical sequelae from transfusing contaminated platelets are currently the major risk factor for transfusion-transmitted dis- ease and morbidity.

STANDARDS OF CARE

Platelets should be issued in a specialized bag with a label warning not to expose them to cold temperatures or ice. The transfusion service should educate hospital providers about appropriate temperatures for keeping platelets.

EEichbaum_87963_PTR_CH01_10-01-12_001-018.inddichbaum_87963_PTR_CH01_10-01-12_001-018.indd 1616 111/10/121/10/12 12:5112:51 PMPM Recommended Reading 17

Platelets that are exposed to cold must be discarded because such exposure targets them in vivo for destruction by liver and spleen macrophages following transfusion.

RECOMMENDED READING

Hoffmeister KM, Flebinger TW, Falet H, et al. The clearance mecha- nism of chilled blood platelets. Cell 2003;112:87–97. Hoffmeister KM. The role of lectins and glycans in platelet clearance. J. Thromb. Haemost . 2011;9(suppl 1):35–43K.

EEichbaum_87963_PTR_CH01_10-01-12_001-018.inddichbaum_87963_PTR_CH01_10-01-12_001-018.indd 1717 111/10/121/10/12 12:5112:51 PMPM EEichbaum_87963_PTR_CH01_10-01-12_001-018.inddichbaum_87963_PTR_CH01_10-01-12_001-018.indd 1818 111/10/121/10/12 12:5112:51 PMPM 2 Errors in Procedures

A Positive Type and Screen Will Result in Relative Delay in the Issue of Blood 20 Error in Blood Sample Collection Resulting in Inaccurate Type and Screen 23 Misinterpretation of Laboratory Tests for Hemolysis 25 ABO Typing Discrepancy Due to Less Common ABO Subgroups 28 Inappropriate Use of Autologous Blood 31 Rapid Transfusion in Chronic Anemia May Result in Volume Overload 34 Cold Agglutinin Disease (CAD)—Insignifi cance of Low Antibody Titers 36 Hypocalcemic Toxicity from Therapeutic Plasma Exchange (TPE) 38 Hemolysis Following Platelet Transfusion 41 Development of Coagulopathy Secondary to Intensive Therapeutic Plasma Exchange (TPE) 44 Heparin-Flushed Lines—Inadvertent Exposure to Blood Circulation 47

EEichbaum_87963_PTR_CH02_10-01-12_019-050.inddichbaum_87963_PTR_CH02_10-01-12_019-050.indd 1919 111/10/121/10/12 12:5312:53 PMPM 20 2. Errors in Procedures

A POSITIVE TYPE AND SCREEN WILL RESULT IN RELATIVE DELAY IN THE ISSUE OF BLOOD OVERVIEW

Timely issue and delivery of red blood cells (RBCs) are critical to optimal patient care. Experienced surgeons often have some estimates of expected blood loss for a given procedure. In many institutions, surgeons request RBCs to be packed in a validated “cooler” prior to the start of surgery based on these estimated blood losses. However, surgical outcome can be quite variable, and bleeding is often unanticipated. This inherent uncertainty and complexity in surgical practice under- scores the importance of the reliable and timely release of blood products from the blood bank.

In order to issue crossmatched RBC products, the blood bank must have verifi ed the patient’s blood type and must have tested the patient’s plasma to determine whether any non-ABO RBC antibodies are pres- ent (eg, D, Kell, and Jka). If the screen is negative, RBCs can be issued with an immediate spin crossmatch (which takes a few minutes to per- form manually) or an electronic crossmatch (performed in less time by computer verifi cation of the patient’s blood type and the absence of any antibodies in plasma). Most clinicians, including surgeons, assume that RBCs can be obtained immediately when the screen is negative. When the screen is positive, the crossmatch process is more involved and requires at least an hour to complete.

Case with Error

A 15-year-old male is scheduled for an orthopedic procedure to repair a recently sustained knee injury. The patient is otherwise in good health. The surgical team notes that the blood screen is positive for an anti-M alloantibody, which is noted in the report as “clinically insignifi cant.” During the procedure there is unanticipated blood loss, causing the

Eichbaum_87963_PTR_CH02_10-01-12_019-050.indd 20 11/10/12 12:53 PM A Positive Type and Screen Will Result in Relative Delay in the Issue of Blood 21

patient to become hypotensive and signifi cantly tachycardic (rise in heart rate of 15 beats/minutes). A crossmatched unit is requested from the blood bank. After waiting 30 minutes, the operating room calls the blood bank to inquire about the blood and is informed that the crossmatch is “in progress.” The decision is made to transfuse a unit of group O (uncrossmatched) trauma blood.

Clinical Pitfall

Failure to anticipate a potential delay in the issue of blood based on the patient’s blood type and screen result. In some patients, the plasma antibody screen for RBC antibodies is positive, but the blood bank is unable to identify the offending anti- body. The report may describe the result as “nonspecifi c.” On other occasions, the screen will demonstrate the presence of an antibody, such as an “anti-M” alloantibody, that is usually clinically insignifi - cant. Anti-M antibodies have only rarely been shown to cause delayed hemolytic transfusion reactions. The presence of a nonspecifi c anti- body or a clinically insignifi cant antibody may erroneously suggest to the clinical team that crossmatch-compatible units will be available shortly. However, these cases still require a crossmatch process that could delay the availability of compatible blood in urgent situations.

Explanation and Consequences

If the plasma screen to identify RBC antibodies is positive (even due to a “nonspecifi c” or “clinically insignifi cant” antibody), the crossmatch test still takes 1 hour to complete. It is important for surgeons and anes- thesiologists to understand the crossmatch processes for patients with positive versus negative RBC antibody screening tests. Many surgeons are reluctant to use trauma or non-crossmatched blood, although the clinical consequence of using non-crossmatched or trauma blood, if the patient’s screen is negative, is generally negligible. Particularly if the patient is hemodynamically unstable, such concerns about crossmatch- ing should not delay transfusion. Similarly, if the screen demonstrates a clinically insignifi cant antibody, there is a high likelihood that trauma blood is suitable and will have no adverse clinical consequences.

EEichbaum_87963_PTR_CH02_10-01-12_019-050.inddichbaum_87963_PTR_CH02_10-01-12_019-050.indd 2121 111/10/121/10/12 12:5312:53 PMPM 22 2. Errors in Procedures

STANDARDS OF CARE

Two units of crossmatched RBCs should be on reserve for each patient scheduled for surgery who has a positive antibody screen. The clinical team should be notifi ed by the blood bank whenever there is an anticipated delay in providing crossmatch-compatible units.

RECOMMENDED READING

Berry-Dortch S, Woodside CH, Boral LI. Limitations of the immediate spin crossmatch when used for detecting ABO incompatibility. Trans- fusion 1985;25:176–178. Kuriyan M, Fox E. Pretransfusion testing without serologic crossmatch: approaches to ensure patient safety. Vox Sang. 2000;78:113–118.

EEichbaum_87963_PTR_CH02_10-01-12_019-050.inddichbaum_87963_PTR_CH02_10-01-12_019-050.indd 2222 111/10/121/10/12 12:5312:53 PMPM Error in Blood Sample Collection Resulting in Inaccurate Type and Screen 23

ERROR IN BLOOD SAMPLE COLLECTION RESULTING IN INACCURATE TYPE AND SCREEN OVERVIEW

The location and manner in which a blood sample is col- lected for laboratory testing may be a source of signifi cant medical error. For instance, a blood sample collected from a heparin-fl ushed line may show an elevated partial thromboplas- tin time (PTT). A blood sample collected from a vein in the same extremity and “upstream” from a vein into which blood is being transfused will likely yield inaccurate laboratory results as it will be mixed with transfused blood.

Case with Error

A 36-year-old female victim of a motor vehicle collision is airlifted to the nearest trauma center. She arrives in the emergency department (ED) with two lines rapidly transfusing RBCs into the dorsal veins of each hand. In the hospital, a massive transfusion protocol is acti- vated that includes group O Rh-negative red blood cells (RBCs), AB plasma, and platelets. A peripheral vein is accessed in the antecubital area of her right arm from which a sample for a blood type and screen is drawn. The medical technologist performing the type and screen assay notices a discrepancy between the forward and reverse typing of the RBCs: the forward type is group O and the reverse type is “mixed fi eld.” The medical technologist is perplexed by the unusually ele- vated hematocrit of the sample drawn from this profusely bleeding patient. After performing additional typing, the technologist requests another sample. The second sample is group A Rh-positive (different from the group O Rh-negative blood transfused). As per laboratory protocol, a third sample is then drawn, and also types as group A Rh-positive.

Eichbaum_87963_PTR_CH02_10-01-12_019-050.indd 23 11/10/12 12:53 PM 24 2. Errors in Procedures

Clinical Pitfall

Failure to recognize the importance of correct blood sampling tech- nique for blood typing. In a trauma patient being massively transfused with group O Rh- negative RBCs and AB plasma, drawing a blood sample from a site proximal to the transfusion site can lead to mistyping of the patient’s blood due to mixing of patient’s blood and the transfused group O RBCs. This could result in a mistransfusion.

Explanation and Consequences

The fi rst sample was drawn proximal to the site where group O Rh- negative blood was being rapidly transfused, and therefore the forward type matched that of the transfused blood. Had the medical technolo- gist accepted group O Rh-negative as the patient’s true blood type, this could have led to a mistransfusion. If the patient had been switched to type O plasma following the massive transfusion, this could have caused acute hemolysis of her group A RBCs because of the native isohemagglutinins (anti-A and anti-B) in group O plasma.

STANDARDS OF CARE

Samples for a type and screen assay should not be collected from a vein proximal to the site where donor blood is simultaneously being transfused in a patient. Specimens received by blood bank laboratories must at a minimum have the correct patient information on the label, including, but not limited to, patient last name, patient fi rst name, date, time, and phlebotomist identifi cation.

RECOMMENDED READING

McClain CM, Patel C, Szklarski P, Booth GS. Identifi cation of blood draw error during trauma resuscitation. Transfusion 2012;52(9):1855–1856. Duran JA, González AA, García DD, et al. Best blood sample draw site during liver transplantation. Transplant Proc. 2009; 41(3):991–993.

EEichbaum_87963_PTR_CH02_10-01-12_019-050.inddichbaum_87963_PTR_CH02_10-01-12_019-050.indd 2424 111/10/121/10/12 12:5312:53 PMPM Misinterpretation of Laboratory Tests for Hemolysis 25

MISINTERPRETATION OF LABORATORY TESTS FOR HEMOLYSIS OVERVIEW

In emergency transfusions, there is typically no time to perform a patient’s blood type and screen, and therefore universal donor group O Rh-negative RBCs are transfused. The major concern is to maintain the patient’s blood oxygen carry- ing capacity by monitoring the hemoglobin (Hgb)/hematocrit levels, and to prevent symptomatic anemia.

Without an antibody screen, the potential risk exists for a delayed hemolytic transfusion reaction in patients who have been alloimmunized to minor RBC antigens. Such hemolysis could lead to a worsening ane- mia. Usually, but not always, out-of-group transfusions have clinical consequences. However, in massive transfusions, when the equivalent of more than a total adult blood volume of blood is not screened for reactiv- ity against clinically signifi cant antigens, the risk of delayed hemolytic reactions (DHTRs) increases due to the exposure to large amounts of donor blood. Hemolytic reactions may be identifi ed by a panel of laboratory tests that can include a lactate dehydrogenase (LDH) level, biliru- bin (total, direct, indirect), liver enzymes (aspartate aminotransfer- ase, AST; alanine aminotransferase, ALT), the reticulocyte count, a haptoglobin level, and a peripheral blood smear. Numerous medical conditions other than hemolysis, especially hepatorenal and cardiac conditions, are associated with abnormalities in these test values. In the setting of transfusion, failure to take into account other underly- ing comorbidities affecting these test values can lead to the erroneous conclusion that RBC hemolysis has occurred.

Case with Error

A 36-year-old female on dialysis with end-stage renal disease is brought with a ruptured arteriovenous fi stula to the emergency department, bleeding

Eichbaum_87963_PTR_CH02_10-01-12_019-050.indd 25 11/10/12 12:53 PM 26 2. Errors in Procedures

profusely. A massive transfusion protocol is initiated, during which she receives 12 units of group AB plasma, 2 units of apheresis platelets, and 12 units of group O Rh-negative packed RBCs. Her records from an outside hospital show that several years ago she was shown to have the following alloantibodies: anti-Kell, anti-E, anti-c, and anti-Jka. The blood bank resident is concerned about the possibility of a DHTR and orders LDH, bilirubin, and haptoglobin tests to assess for hemolysis. He calls the operating room and is informed that the patient has coded and has to be intubated. The LDH and bilirubin results from the laboratory are subsequently shown to be elevated and the haptoglobin decreased. The resident is now convinced that the patient is experiencing a DHTR, but he is perplexed to fi nd that her new antibody screen is negative. The patient’s direct antiglobulin test (DAT) is also negative.

Clinical Pitfall

Failure to recognize that elevated LDH, bilirubin, and haptoglobin values do not always indicate RBC hemolysis but may be associated with other underlying comorbidities. Elevated LDH and bilirubin laboratory tests may be evident in a transfused patient who also has cardiac and hepatorenal comorbidities. It should not be assumed to be indicative of RBC destruction, even if the patient also exhibits multiple alloantibodies. As the patient’s cur- rent antibody screen is negative, the abnormal test results are unlikely to indicate antibody-mediated RBC destruction, but are more likely due to the underlying comorbidities.

Explanation and Consequences

Ordering LDH, bilirubin, and haptoglobin tests in a setting of alloim- munization is a reasonable approach for ruling out hemolysis, espe- cially in the case of this profusely bleeding patient whose clinical situation rapidly worsened. The current indirect antibody screen of this patient was negative suggesting that her previously identifi ed alloanti- bodies were currently at a titer below the level of serologic detection or

EEichbaum_87963_PTR_CH02_10-01-12_019-050.inddichbaum_87963_PTR_CH02_10-01-12_019-050.indd 2626 111/10/121/10/12 12:5312:53 PMPM Recommended Reading 27

capacity to cause RBC destruction. Moreover, the short time frame of the reaction also does not fi t the longer kinetics of a DHTR. Her elevated LDH was subsequently revealed to be due to a myocardial infarction she suffered in the emergency department, and her elevated bilirubin values likely a result of her renal malfunction. During massive transfusion, haptoglobin may also be marginally depressed due to release of Hgb from RBCs.

STANDARDS OF CARE

A negative antibody screen and DAT in a patient with a history of alloimmunization reduces the likelihood of a DHTR. Alternate comorbidities should be considered in interpreting laboratory test values typically used in the evaluation for RBC lysis (eg, LDH, bilirubin, and haptoglobin). A peripheral blood smear should be examined to check for immune hemolysis.

RECOMMENDED READING

Vamvakas EC, Pineda AA, Reisner R, et al. The differentiation of delayed hemolytic and delayed serologic transfusion reactions: incidence and predictors of hemolysis. Transfusion 1995;35:26–32.

EEichbaum_87963_PTR_CH02_10-01-12_019-050.inddichbaum_87963_PTR_CH02_10-01-12_019-050.indd 2727 111/10/121/10/12 12:5312:53 PMPM 28 2. Errors in Procedures

ABO TYPING DISCREPANCY DUE TO LESS COMMON ABO SUBGROUPS

OVERVIEW

A blood type is based on the presence or absence of inher- ited antigenic structures on the surface of RBCs. The ABO system is the most important blood group system in human blood transfusion. The associated anti-A and anti-B antibodies are both class IgM and IgG immunoglobulins. ABO IgM and IgG antibodies are produced in the fi rst years of life, probably as a result of sensitization to environmental substances such as food, bacteria, and viruses. ABO typing involves both antigen typing and antibody detection. The antigen typing is referred to as the forward typing, and the antibody detection is the reverse typing.

Case with Error

A 23-year-old female with chronic heart failure due to congenital car- diomyopathy is being evaluated for heart transplantation. As part of the routine laboratory pre-transplantation evaluation, a sample is col- lected for a type and screen test and sent to the hospital blood bank. Her forward type reveals that her red cells did not react with the anti-A or anti-B antisera, and her reverse type identifi es both anti-A and anti-B in her plasma. These results are interpreted as blood type O. Her blood is also sent for human leukocyte antigen (HLA) typing. The HLA laboratory repeats the typing process and determines that she is actually a subtype of blood group A. This disparity prompts a review of the original blood typing, revealing an unusually weak (1+) anti-A, which is consistent with a subtype of the blood group A that produces a weak anti-A1. Further testing indicates that the patient’s erythrocytes do not react with the anti-A1 lectin, Dolichos bifl orus .

EEichbaum_87963_PTR_CH02_10-01-12_019-050.inddichbaum_87963_PTR_CH02_10-01-12_019-050.indd 2828 111/10/121/10/12 12:5312:53 PMPM Standards of Care 29

These results suggest that she is phenotypically a subtype of A other than A1, and explains why her immune system has produced an anti- A1 antibody. The patient’s is updated, and she eventu- ally receives an A2 graft.

Clinical Pitfall

Failure to recognize minor ABO subgroups and their clinical signifi cance. There are various subgroups within major blood groups A and B. The two principal subgroups of group A are A1 and A2, although other subgroups also exist. A1 cells agglutinate in the presence of D. bifl orus, and anti-A1 lectin, whereas other subtypes of A do not. The red cells of approximately 80% of group A individuals in the United States are A1; the remaining 20% are A2 or other weaker sub- groups. Patients with A2 or the other subgroups may produce an anti- A1 antibody, which may necessitate the transfusion of type O cells to these individuals.

Explanation and Consequences

The majority of heart transplants performed on adults are ABO anti- gen compatible so as to avert hyperacute rejection. ABO antigens, expressed on cardiac endothelium, cross react with preformed anti- blood group antibodies in the recipient leading to rejection. It is unlikely that the patient in this case would have suffered direct harm by receiving a type O heart because type O individuals do not express A or B antigens. However, type O hearts are relatively scarce. This patient was able to make use of a type A2 heart so that the type O organ could be used by another individual.

STANDARDS OF CARE

Failure to properly type both the donor and recipient can result in life-threatening harm to the recipient. It also represents poor man- agement of heart organs suitable for transplantation, a rare and valuable resource.

EEichbaum_87963_PTR_CH02_10-01-12_019-050.inddichbaum_87963_PTR_CH02_10-01-12_019-050.indd 2929 111/10/121/10/12 12:5312:53 PMPM 30 2. Errors in Procedures

RECOMMENDED READING

Costanzo MR, Dipchand A, Starling R, et al. The International Society of Heart and Lung Transplantation Guidelines for the care of heart transplant recipients . J. Heart Lung Transplant 2010;29:914–956.

Eichbaum_87963_PTR_CH02_10-01-12_019-050.indd 30 11/10/12 12:53 PM Inappropriate Use of Autologous Blood 31

INAPPROPRIATE USE OF AUTOLOGOUS BLOOD

OVERVIEW

It is estimated that almost half of all blood transfusions performed in the United States occur perioperatively. What is often not realized is that there are several alternatives to the transfusion of allogeneic blood. The most common alterna- tives include intraoperative blood salvage, acute normovolemic hemodilution, and transfusion with banked autologous blood. Each of these alternatives, however, carries advantages and dis- advantages. Transfusion of autologous blood is no exception.

Case with Error

A 69-year-old African American male presents to the Depart- ment for follow-up investigation of an abnormal mass detected on dig- ital rectal examination. The patient’s serum prostate specifi c antigen has been <4 ng/mL since initiation of testing 3 years ago. The clini- cal team confi rms the presence of an abnormal mass and schedules a biopsy of the prostate gland. The patient does not want to receive any blood products because he is concerned about “tainted blood” in the community. The urology team suggests that he donate his own (autologous) blood 1 week prior to surgery. A previous count of Hgb is 14.3 g/dL. Eight days prior to the surgery the patient donates 500 mL of whole blood for component fractionation of a single RBC product to be made available for transfusion, if deemed necessary, in the peri- operative time period.

Clinical Pitfall

Failure to understand the appropriate use of autologous RBC transfusions. The decision to donate a unit of blood must be driven by several fac- tors, the most important of which is the clinical likelihood of requiring

EEichbaum_87963_PTR_CH02_10-01-12_019-050.inddichbaum_87963_PTR_CH02_10-01-12_019-050.indd 3131 111/10/121/10/12 12:5312:53 PMPM 32 2. Errors in Procedures

a blood transfusion for the given procedure. Review of hospital blood ordering guidelines as well as professional board recommendations indicates that autologous should be considered only if the chance for blood transfusion exceeds 90%. Patients using their banked autologous units are not free from risk because bacterial con- tamination, cardiopulmonary overload, and clerical errors have all been associated with transfusion of autologous blood. Preoperative use of recombinant erythropoietin is both expensive and dependent on adequate iron stores and bone marrow function. The use of recombi- nant erythropoietin also has the potential for adverse events, including pure red cell aplasia, increased risk of serious cardiovascular events, thromboembolism, stroke, and even mortality when administered to target Hgb levels of >11 g/dL.

Explanation and Consequences

The patient’s biopsy was successful, and minimal surgical bleeding was observed at the biopsy site. On postoperative day 1, the patient was discharged home with Hgb of 14.2 g/dL without having received his autologous RBC product. The advantages of autologous blood donation include the prevention of alloimmunization, the prevention of transfusion- associated diseases, and reduced likelihood of transfusion reactions. However, even with sterilely collected autologous blood, the risk remains of an adverse event at the time of donation (eg, vasovagal reaction), as does the risk of bacterial contamination secondary to skin microfl ora. Autologous blood donation is more expensive and frequently results in either increased wastage (product never transfused) or over-transfusion due to the ready availability of the banked blood. Moreover, the combination of transfused autologous blood and intraoperative fl uids places patients at risk for cardiopulmonary overload. In this case, the low amount of anticipated blood loss for the prostate biopsy did not warrant the preprocedure autologous blood collection.

EEichbaum_87963_PTR_CH02_10-01-12_019-050.inddichbaum_87963_PTR_CH02_10-01-12_019-050.indd 3232 111/10/121/10/12 12:5312:53 PMPM Recommended Reading 33

STANDARDS OF CARE

Autologous blood donation should be performed prior to surgery to avoid unnecessary iatrogenic blood loss (minimum time >72 hours; optimal time frame is approximately 1 week prior to procedure). AABB and FDA standards indicate that autologous donors should have a Hgb count of at least 11 g/dL (hematocrit 33%) and should be relatively healthy.

RECOMMENDED READING

Billote DB, Glisson SN, Green D, Wixson RL. A prospective, ran- domized study of preoperative autologous blood donation for hip replacement surgery. J. Bone Joint Surg. Am . 2002;84-A:1299–1304. Brecher ME, Goodnough LT. The rise and fall of preoperative autolo- gous blood donation. Transfusion 2002;42:1618–1622. Forgie MA, Wells PS, Laupacis A, Fergusson D. Preoperative autolo- gous donation decreased allogeneic transfusion but increases expo- sure to all transfusion: results of a meta-analysis. Arch. Intern. Med . 1998;158:610–616.

EEichbaum_87963_PTR_CH02_10-01-12_019-050.inddichbaum_87963_PTR_CH02_10-01-12_019-050.indd 3333 111/10/121/10/12 12:5312:53 PMPM 34 2. Errors in Procedures

RAPID TRANSFUSION IN CHRONIC ANEMIA MAY RESULT IN VOLUME OVERLOAD OVERVIEW

Children and adults with severe chronic anemia (Hgb <5.0 g/dL) are usually transfused slowly in order to reduce the risk of transfusion-associated circulatory overload (TACO). Patients with severe chronic anemia have a relatively normal circulating blood volume (70–80 mL/kg), but with a compensatory increase in their plasma volume that increases their risk for TACO.

Case with Error

A 3-year-old child is bought into the ED because of observed changes in mental status. A lumbar puncture is performed, demonstrating blasts in the cerebrospinal fl uid. A complete blood count shows signifi cant leukocytosis with 25% blasts, consistent with a diagnosis of acute leu- kemia. The Hgb is 5.2 g/dL, and platelets are 100,000/μL. The patient is admitted to the service for further evaluation of this newly diagnosed leukemia. Prior to the transfer, the physician orders trans- fusion of 2 units of RBCs. The units are transfused over 40 minutes. Shortly after infusion, the patient becomes tachypneic (respiration rate 48 breaths/minute) and is ventilated and transferred to the ICU. Chest X-ray does not show any focal consolidations, but shows diffuse inter- stitial congestion consistent with volume overload.

Clinical Pitfall

Failure to recognize the increased risk of TACO in patients with chronic anemia. Although TACO has long been recognized as a risk in transfu- sion of chronically anemic patients, general practitioners are often not aware of this risk. In addition, physician transfusion orders do not

Eichbaum_87963_PTR_CH02_10-01-12_019-050.indd 34 11/10/12 12:53 PM Recommended Reading 35

often stipulate the transfusion rate. Patients with severe chronic ane- mia may, therefore, be transfused at the standard rate, rendering them susceptible to volume overload.

Explanation and Consequences

TACO remains a poorly understood complication of blood product transfusion and is mainly seen in small children (3 years or younger) and those >65 years with underlying cardiopulmonary dysfunction. If unrecognized, TACO can lead to pulmonary and systemic overload, cardiogenic pulmonary edema, and ultimately to clinical decompensa- tion. Elevated blood pressure, tachycardia, and increased pulmonary wedge pressure are often observed. TACO is precipitated by too large a volume of blood being transfused too rapidly. The risk of TACO increases when transfusion fl ow rates are not appropriately calculated based on the recipient’s weight in kilograms.

STANDARDS OF CARE

A single unit of transfused blood product can be suffi cient to cause TACO; vigilance for volume overload is required even with small volumes of transfused products. In , if the Hgb is <5 g/dL, transfuse over a period of 4 hours, the following calculated volume: number of milliliters = Hgb × weight in kilograms.

RECOMMENDED READING

Li G, Rachmale S, Kojicic M, et al. Incidence of transfusion risk fac- tors for transfusion-associated circulatory overload among medical intensive care unit patients. Transfusion 2011;51:338–343.

EEichbaum_87963_PTR_CH02_10-01-12_019-050.inddichbaum_87963_PTR_CH02_10-01-12_019-050.indd 3535 111/10/121/10/12 12:5312:53 PMPM 36 2. Errors in Procedures

COLD AGGLUTININ DISEASE (CAD)— INSIGNIFICANCE OF LOW ANTIBODY TITERS

OVERVIEW

Cold agglutinin disease (CAD) is caused by IgM autoanti- bodies that can agglutinate RBCs in vitro at cold tempera- tures (4–18°C). These autoantibodies can cause intravascular hemolysis in vivo, especially when present in high titers at warmer temperatures (30–37°C). The hemolysis is mediated by the classic complement pathway that lyses the RBC membrane, releasing Hgb and resulting in hemoglobinemia, hemoglobin- uria, and low free haptoglobin.

When the antibody concentration is not suffi ciently high to acti- vate the full complement cascade, complement activation is taken only to the C3 stage, producing RBCs coated with C3b without hemo- lysis. The titer and temperature at which hemolysis occurs can vary, but titers lower than 1 to 32 only rarely cause clinically signifi cant hemolysis. Evidence suggests that only titers of autoantibody greater to 1 to 1,000 are clinically signifi cant. When the antibody is not reac- tive at 30°C or higher, hemolysis is unlikely when the titers are low.

Case with Error

A 29-year-old female with a recent of Mycoplasma pneumoniae comes to the emergency department with acrocyanosis and dizziness. A complete blood count reveals mild anemia (Hgb 11 g/dL) but is otherwise normal. A DAT shows binding of C3d to the red cells. The LDH test level is elevated at 226 units/L, and a blood smear indicates mild RBC agglutination. The physician suspects CAD and orders a full CAD evaluation. The IgM cold agglutinin titer is 1 to 16, demon- strating anti-I specifi city. The antibody is found to be reactive at room temperature but not at 30°C or higher. The physician is concerned about the titer of the antibody and decides to transfuse the patient with 2 units of RBCs to treat the CAD.

EEichbaum_87963_PTR_CH02_10-01-12_019-050.inddichbaum_87963_PTR_CH02_10-01-12_019-050.indd 3636 111/10/121/10/12 12:5312:53 PMPM Recommended Reading 37

Clinical Pitfall

Failure to recognize that low titers of cold agglutinins can be detected in normal individuals and are only rarely clinically signifi cant. Cold agglutinin titers as low as 1 to 16 are generally not clinically signifi cant and do not cause hemolysis. Such titers are not diagnostic of CAD and should not be used as a basis for initiating treatment. The upper limit of cold agglutinin titers in normal individuals is consid- ered to be around 1 to 40. Hemolysis is usually seen at titers of 1 to 500–1,000 or higher, whereas patients with CAD typically have titers above 1 to 10,000.

Explanation and Consequences

This patient has a mild transient form of CAD mostly likely precipi- tated by her mycoplasma infection. The low titer of her IgM is not clinically signifi cant, and there was no need to order a full evaluation for CAD, or to initiate therapy. Supportive measures such as avoid- ance of cold weather and keeping the patient warm would have been suffi cient. Transfusion to treat her mild anemia is also not necessary because she is asymptomatic.

STANDARDS OF CARE

In CAD, at low IgM antibody titers, only supportive treatment such as avoidance of cold temperatures, is necessary. The need for transfusion should be assessed by the degree of the anemia.

RECOMMENDED READING

Garratty G, Petz LD. Approaches to selecting blood for transfusion to patients with autoimmune hemolytic anemia. Transfusion 2002; 42:1390–1392. Berentsen S, Beiske K, Tjønnfjord GE. Primary chronic cold aggluti- nin disease: an update on , clinical features and therapy. Hematology 2007;12(5):361–370.

EEichbaum_87963_PTR_CH02_10-01-12_019-050.inddichbaum_87963_PTR_CH02_10-01-12_019-050.indd 3737 111/10/121/10/12 12:5312:53 PMPM 38 2. Errors in Procedures

HYPOCALCEMIC TOXICITY FROM THERAPEUTIC PLASMA EXCHANGE (TPE)

OVERVIEW

Hypocalcemic toxicity is one of the most common adverse effects of TPE. The symptoms of this toxicity are associ- ated with a decrease in plasma ionized calcium. The normal range for total calcium concentration in the plasma is 8.5–10.5 mg/dL. Approximately 50% of this concentration is ionized, 40% is bound to proteins (primarily albumin), and 10% circulates in a form bound to anions. Citrate-based anticoagulants, notably sodium citrate, are a major cause of hypocalcemic toxicity as the citrate binds to and thereby lowers the intravascular ionized calcium levels. In addition, replacement fl uids such as human albumin and fresh frozen plasma (FFP) may contribute to hypo- calcemia. It is important to recognize the signs and symptoms of hypocalcemic toxicity because effective interventions are readily available.

Case with Error

A 50-year-old female has been placed on a twice monthly regimen of therapeutic apheresis to manage her myasthenia gravis (MG). Dur- ing one such treatment, she complains of fl u-like symptoms and notes that she has decreased appetite and increased fatigue. Her vital signs are normal, and a two-volume exchange is initiated using albumin as replacement fl uid. One hour into the procedure, the patient complains of perioral numbness and chest tightening. The patient is given oral calcium tablets, but the symptoms persist. A test for ionized calcium is performed yielding a result of 3.0 mg/dL (reference range in this laboratory: 4.5–5.6 mg/dL). An additional 4 g of calcium gluconate is administered intravenously.

EEichbaum_87963_PTR_CH02_10-01-12_019-050.inddichbaum_87963_PTR_CH02_10-01-12_019-050.indd 3838 111/10/121/10/12 12:5312:53 PMPM Standards of Care 39

Clinical Pitfall

Failure to monitor and treat hypocalcemia during a therapeutic apheresis procedure that leads to hypocalcemic toxicity. Hypocalcemic toxicity during therapeutic apheresis is often attributed solely to sodium citrate anticoagulant administered during the procedure. Other solutions used during treatment, such as albumin with a high avidity for ionized calcium, or FFP, can also contribute signifi cantly to the reduction in ionized calcium.

Explanation and Consequences

Typical symptoms of hypocalcemic toxicity include oral and acral paresthesias (pins and needles sensation around mouth and lips and in extremities), light headedness, a metallic taste, shivering or twitch- ing, chest tightening or pain, abdominal cramping, nausea/vomiting, tetany, seizures, or cardiac arrhythmias. Although modern apheresis instruments are designed to mitigate the potential for hypocalcemic toxicity by delivering a steady dose of citrate over the course of the procedure, maneuvers such as slowing the infusion rate of the citrate and replacement fl uids, or the addition of calcium gluconate directly into the replacement fl uid (ie, albumin), can also be used. For patients more susceptible to hypocalcemia, substitution of an alternative col- loid that does not bind calcium may be considered. If patients have persistent symptoms, it may be prudent to obtain an EKG to check for QT prolongation.

STANDARDS OF CARE

Treatments of hypocalcemic toxicity should be appropriately imple- mented; these include slowing the infusion rate, providing calcium replacements (intravenous or oral), and/or adding calcium directly to replacement fl uids such as albumin. For patients experiencing hypocalcemia, careful monitoring is required for life-threatening complications such as laryngospasm or cardiac arrhythmias.

EEichbaum_87963_PTR_CH02_10-01-12_019-050.inddichbaum_87963_PTR_CH02_10-01-12_019-050.indd 3939 111/10/121/10/12 12:5312:53 PMPM 40 2. Errors in Procedures

RECOMMENDED READING

Weinstein R. Hypocalcemic toxicity and atypical reactions in thera- peutic plasma exchange. J. Clin. Apher . 2001;16:210–211. Mokrzycki MH, Kaplan AA. Therapeutic plasma exchange: compli- cations and management. Am. J. Kidney Dis . 1994;23:817–827.

EEichbaum_87963_PTR_CH02_10-01-12_019-050.inddichbaum_87963_PTR_CH02_10-01-12_019-050.indd 4040 111/10/121/10/12 12:5312:53 PMPM Hemolysis Following Platelet Transfusion 41

HEMOLYSIS FOLLOWING PLATELET TRANSFUSION OVERVIEW

When ABO group-specifi c platelet concentrates are not available, it is common practice to transfuse ABO out-of- group platelets. An adult dose of platelets contains approxi- mately 250 mL of donor plasma. Infusion of this plasma can result in the passive transfer of anti-A and/or anti-B antibodies. Usually, this does not result in any serologically evident or clin- ically signifi cant hemolysis. Rarely, however, transfusion of out- of-group platelets can lead to signifi cant passive alloimmune hemolysis due to “passively” transferred antibodies in the resid- ual plasma in the product. Deaths have occurred due to such hemolysis attributable to transfusion of out-of-group platelets.

Case with Error

A 50-year-old female with blood group A Rh-positive recently diag- nosed with acute myeloid leukemia (AML) is pancytopenic as a result of induction chemotherapy that was initiated 1 week earlier. Her platelet count falls to below 10,000/μL, and a platelet transfusion is requested. She receives 50 mL of group O, Rh-positive single donor apheresis platelets. During the transfusion, she complains of back pain. The transfusion is stopped, and she is given 50 mg of diphenhydramine IV before the transfusion is resumed. After 150 mL, she complains of shortness of breath. The transfusion is stopped, and the patient is treated with hydrocortisone prior to completing the transfusion. She receives a total of 350 mL of the group O platelets. Two hours later she passes red urine. Her Hgb has decreased by 3 g/dL, and her bilirubin has risen to 13.5 mg/dL from normal levels prior to transfusion. Her serum LDH rises to 500 units/dL, and her haptoglobin is undetectable. Her DAT is positive, and the RBC eluate reveals an anti-A antibody. She is treated with 2 units of packed RBCs.

Eichbaum_87963_PTR_CH02_10-01-12_019-050.indd 41 11/10/12 12:53 PM 42 2. Errors in Procedures

Clinical Pitfall

Failure to diagnose a passively mediated hemolytic transfusion reaction due to a minor incompatibility between donor plasma recipient red cells. It is often assumed that a product that does not contain RBCs can- not result in a hemolytic transfusion reaction. Minor blood group anti- gen mismatches (ie, the donor plasma is incompatible with the recipi- ent RBCs) may occasionally result in clinically signifi cant hemolysis. If an acute reaction occurs during an ABO-incompatible platelet trans- fusion, the transfusion should be discontinued and a complete labora- tory and clinical evaluation pursued.

Explanation and Consequences

This patient experienced acute intravascular hemolysis due to the transfusion of an out-of-group platelet product with residual plasma contained passively transferred anti-A and anti-B isohemagglutinins. As this patient’s blood type is group A, the passive transfer of anti-B should not have any clinical sequelae in this case. The anti-A IgM class antibodies contained in the group O platelet product hemolyzed the patient’s group A RBCs. The titer of the donor platelet revealed an anti- A IgM titer of 1 to 8,000. Typical titers are between 1 to 8 and 1 to 64. While hemolysis is more likely to occur with a unit of apheresis plate- lets from a single donor, out-of-group hemolysis resulting from ran- dom pooled platelets has also been reported in the literature. The low availability of type specifi c platelet products, clinical urgency, and the economic pressure to avoid the outdating of platelets (given their short 5-day shelf life) are major reasons that out-of-group platelets are used. In most cases, the practice does not lead to a problem, as most adults can tolerate several hundred milliliters of incompatible plasma. The antibodies passively transferred in the product have limited impact as they are diluted throughout the recipient’s blood volume, and are further neutralized by soluble ABO antigens in the recipient (present in about 80% of people), or by ABO glycoconjugates on tissues. Problems arise when the antibody level in the donor is very high (>1 to 300 anti-A or anti-B), or the relative amount of plasma volume transfused is sig- nifi cant (such as in small children or in a massively transfused patient).

EEichbaum_87963_PTR_CH02_10-01-12_019-050.inddichbaum_87963_PTR_CH02_10-01-12_019-050.indd 4242 111/10/121/10/12 12:5312:53 PMPM Recommended Reading 43

STANDARDS OF CARE

The recommendations of the National Blood Service Transfusion Medicine Clinical Policies Group are (1) platelets from donors with identical ABO group are the component of choice, (2) administra- tion of ABO incompatible platelets is acceptable when platelets are in short supply, and (3) group O platelets should not be given if known to be a high titer for anti-A or anti-B isohemagglutinins.

RECOMMENDED READING

Josephson CD, Mullis N, Van Demark C, Hillyer CD. Signifi cant numbers of apheresis-derived group O platelet units have “high-titer” anti-A/A,B: implications for transfusion policy. Transfusion 2004; 44:805–808.

EEichbaum_87963_PTR_CH02_10-01-12_019-050.inddichbaum_87963_PTR_CH02_10-01-12_019-050.indd 4343 111/10/121/10/12 12:5312:53 PMPM 44 2. Errors in Procedures

DEVELOPMENT OF COAGULOPATHY SECONDARY TO INTENSIVE THERAPEUTIC PLASMA EXCHANGE (TPE)

OVERVIEW

Plasmapheresis, or therapeutic plasma exchange (TPE), is the process by which whole blood is withdrawn from an individual’s circulation, one of its components, such as plasma, is separated out, and the remainder of the blood is returned together with a replacement fl uid. In therapy for some diseases, such as thrombotic thrombocytopenic purpura, the replacement fl uid is plasma from healthy donors. In other diseases, such as myasthenia gravis (MG), the replacement fl uid can be albumin or a combination of albumin and normal saline. Such replacement fl uids have the benefi t of avoiding exposure to donor plasma that may induce infectious and immunologic complications.

Case with Error

A 35-year-old African American female presents to her doctor with weakness in both upper and lower extremities. During the week prior to her admission, the patient had progressive diffi culty walking. On examination, the patient demonstrates ptosis, slurred speech, unstable gait, and weakness in both upper extremities. There are no sensory defi cits. The patient is given the Tensilon test, a drug that temporarily blocks the degradation of acetylcholine. The test is positive. Blood tests for acetylcholine receptor antibodies are positive. Based on these fi ndings, the patient is diagnosed with MG and treated with high-dose prednisone. Owing to her rapid symptomatic deterioration, TPE is initiated with 2.5 blood volumes exchanged daily, via a femoral line using 5% albumin as replacement fl uid. By the fi fth day, some oozing is noted around her femoral line. Her INR and PTT are elevated to

EEichbaum_87963_PTR_CH02_10-01-12_019-050.inddichbaum_87963_PTR_CH02_10-01-12_019-050.indd 4444 111/10/121/10/12 12:5312:53 PMPM Standards of Care 45

3.5 and 55 seconds, respectively. Her platelet counts and hemoglobin level are unchanged from admission.

Clinical Pitfall

Failure to compensate for dilution of coagulation factors from . The goal of TPE is to remove pathologic substances from the plasma. In the case of MG, the pathologic element is an autoantibody. As removal of the patient’s plasma also removes other plasma pro- teins, the physician must consider the possibility that this procedure may concomitantly also deplete clotting factors.

Explanation and Consequences

Intensive TPE using replacement fl uids other than plasma ultimately leads to depletion of coagulation factors. A single-plasma volume exchange will typically reduce coagulation factor levels by 25%–50%, although factor VIII levels are less affected. Fibrinogen, due to its low extravascular distribution, is reduced by nearly 70%. In this patient, the daily plasma exchange of 2.5 blood volumes and replacement with 5% albumin resulted in an increased risk of bleeding. If a patient has normal hepatic function, coagulation factors return to near nor- mal levels within 2 days. Most patients can tolerate plasma exchange every other day for 1 to 2 weeks without plasma replacement. How- ever, with daily exchanges, the last 25% of exchanged volume could be done with plasma instead of albumin, to replenish coagulation factors.

STANDARDS OF CARE

Dilution of coagulation factors in procedures involving the removal of plasma must be assessed by the measurement of PT and PTT values before and after the procedure. Plasma exchange can also cause a decrease in platelets. Platelet counts should therefore also be monitored.

EEichbaum_87963_PTR_CH02_10-01-12_019-050.inddichbaum_87963_PTR_CH02_10-01-12_019-050.indd 4545 111/10/121/10/12 12:5312:53 PMPM 46 2. Errors in Procedures

RECOMMENDED READING

Yeh JH, Chiu HC. Coagulation abnormalities in serial double-fi ltration plasmapheresis. J. Clin. Apher . 2001;16:139–142. Keller AJ, Chirnside A, Urbaniak SJ. Coagulation abnormalities pro- duced by plasma exchange on the cell separator with special reference to fi brinogen and platelet levels. Br. J. Haematol . 1979;42:593–603.

EEichbaum_87963_PTR_CH02_10-01-12_019-050.inddichbaum_87963_PTR_CH02_10-01-12_019-050.indd 4646 111/10/121/10/12 12:5312:53 PMPM Heparin-Flushed Lines—Inadvertent Exposure to Blood Circulation 47

HEPARIN-FLUSHED LINES—INADVERTENT EXPOSURE TO BLOOD CIRCULATION

OVERVIEW

In apheresis procedures, the blood of a patient or donor is circulated through an apparatus that separates out one par- ticular constituent for collection or therapeutic intervention, and returns the remainder of the components to the circulation. Extracorporeal photopheresis (ECP) is an apheresis procedure (used for instance in the treatment of graft versus host disease [GVHD]) in which the patient’s white blood cells and platelets are separated out for treatment with photoactivating drugs and then exposed to ultraviolet light before being returned to the patient’s circulation where they exert a therapeutic effect. Pho- topheresis instruments may be fl ushed with heparin (or citrate- containing solutions) as anticoagulants. As some patients may not tolerate heparin, a careful clinical history should be taken prior to commencing procedures that include this anticoagulant.

While a patient with known heparin-induced thrombocytopenia would be unlikely to be scheduled for procedures that use heparin, patients with less obvious underlying conditions associated with a risk for bleeding (such as subarachnoid hemorrhage) may fail to be excluded. Such patients may inadvertently be exposed to heparin in different ways. There is a tendency to regard the fl ushing of instrument lines with heparin as standard practice and innocuous to patients. Yet serious complications such as drug interactions, iatrogenic hemorrhage and heparin-induced thrombocytopenia have been reported in association with such heparin fl ushing. If heparin is contraindicated for patients, citrate contained in solutions such as acid-citrate-dextrose formula A can be used as an anticoagulant.

EEichbaum_87963_PTR_CH02_10-01-12_019-050.inddichbaum_87963_PTR_CH02_10-01-12_019-050.indd 4747 111/10/121/10/12 12:5312:53 PMPM 48 2. Errors in Procedures

Case with Error Averted

A 56-year-old male with AML who had undergone a peripheral blood stem cell transplant and has now developed chronic GVHD is sched- uled for photopheresis. His posttransplant hospital course has been uncomplicated except for the development of GVHD. After being dis- charged from the hospital, he suffers a fall at home and develops a small but stable subarachnoid hemorrhage, as revealed during a visit to the hospital emergency department. A week later, he returns for his fi rst ECP procedure. Before con- necting him to the apparatus, the nurse reads the patient’s electronic record and learns about his recent fall. She decides to call his doctor. Unaware of the heparin-fl ushed lines (which can leak heparin into the circulation) in the photopheresis instrument, the doctor tells the nurse to proceed with the procedure. As she is about to connect the patient, the nurse recalls that the lines have been fl ushed with heparin, and stops the procedure.

Clinical Pitfall

Failure (averted) to recognize that apheresis and photopheresis instru- ment lines may be fl ushed with heparin as an anticoagulant and may pose a safety risk to patients with certain underlying conditions. Although the volume of heparin in such instrument lines is small and usually of little clinical consequence, several reported cases sug- gest the potential for causing hemorrhage in certain groups of patients.

Explanation and Consequences

The heparin-fl ushed ECP instrument lines could have aggravated this patient’s “stable” subdural hematoma, if this error had not been averted. His doctor ultimately recommended using a citrate-based anticoagulant instead of heparin to avoid this risk.

Eichbaum_87963_PTR_CH02_10-01-12_019-050.indd 48 11/10/12 12:53 PM Recommended Reading 49

STANDARDS OF CARE

Caution should be exercised in apheresis and ECP procedures as the lines may be fl ushed with heparin, which may pose a risk to certain groups of patients. Consider using alternative anticoagulants such as citrate for such patients.

RECOMMENDED READING

Stephens LC, Haire WD, Tatantolo S, et al. Normal saline versus heparin fl ush for maintaining central venous catheter patency during apheresis collection of peripheral blood stem cells (PBSC). Transfus. Sci . 1997;18:187–193. Passannante A, Macik BG. The heparin fl ush syndrome: a cause of iatrogenic hemorrhage. Am. J. Med. Sci . 1988;296:71–73. Rama BN, Haake RD, Bander SJ, et al. Heparin fl ush associated thrombocytopenia—induced hemorrhage: a case report. Nebr. Med. J. 1991;76:392–394.

EEichbaum_87963_PTR_CH02_10-01-12_019-050.inddichbaum_87963_PTR_CH02_10-01-12_019-050.indd 4949 111/10/121/10/12 12:5312:53 PMPM EEichbaum_87963_PTR_CH02_10-01-12_019-050.inddichbaum_87963_PTR_CH02_10-01-12_019-050.indd 5050 111/10/121/10/12 12:5312:53 PMPM Errors Involving Specifi c 3 Clinical Scenarios

Occult Anemia—Searching for a Delayed Hemolytic Transfu- sion Reaction 53 Identifi cation and Management of Clinically Signifi cant Alloantibodies in Pregnancy 56 Liberal versus Restrictive Transfusion Strategies 59 Non-Evidence-Based Practices in Prophylactic Platelet Transfusions for Minor Procedures 61 Molecular Differences in the RhD Protein and the Need for Rh Immune Globulin 63 Recognition of Immune-Mediated Hemolysis in a Pediatric Patient 66 Inappropriate Platelet Transfusion for Patients on Aspirin 70 Unexpected Posttransfusion Purpura (PTP) 73 Phenotype Matching to Mitigate Alloimmunization in Sickle Cell Disease 76 Refusing Blood Transfusion, When Patient and Physician Beliefs Fail to Align 79 Crypt Antigen Activation, Adverse Consequences 82 Failure to Recognize Specifi c Risk Factors That Are Associated with Adverse Reactions in Blood Donors 85 Misinterpretation of the Direct Antiglobulin Test (DAT) 87 Warfarin Reversal—Inappropriate Use of Fresh Frozen Plasma 90 Anti-Kell Alloantibodies—Missed Diagnosis of Hemolytic Disease of the Fetus and Newborn (HDFN) 94 IgA Defi ciency—Misinterpretations and Assumptions 97 Misinterpreting the Cause of Hypotension during Transfusion 100 Inappropriate Application of Premedication for Transfusion 103 Incomplete Evaluation of Platelet Refractoriness 106 Thrombotic Thrombocytopenic Purpura (TTP)—Missed Diagnosis 109

EEichbaum_87963_PTR_CH03_10-01-12_051-136.inddichbaum_87963_PTR_CH03_10-01-12_051-136.indd 5151 111/10/121/10/12 1:001:00 PMPM 52 3. Errors Involving Specifi c Clinical Scenarios

Transfusion-Related Acute Lung Injury (TRALI)—Failure to Diagnose 112 Failure to Recognize That Serious, Potentially Fatal Hemolytic Transfusion Reactions Can Occur with Blood Products 115 Failure to Recognize Drug-Induced Hemolytic Anemia (DIHA) 118 Failure to Recognize That Donor HIV Exposure Is Associated with a Specifi c Federally Mandated Process for Recipient Notifi cation 121 Risk of Hyperkalemia from RBC Transfusion 124 Failure to Recognize That Intensive Plasma Exchange Can Cause Metabolic Alkalosis 127 Failure to Recognize the Risk of Alloimmune Thrombocytopenia in a Primigravida 130 Unnecessary Platelet Transfusions 133

Eichbaum_87963_PTR_CH03_10-01-12_051-136.indd 52 11/10/12 1:00 PM Occult Anemia—Searching for a Delayed Hemolytic Transfusion Reaction 53

OCCULT ANEMIA—SEARCHING FOR A DELAYED HEMOLYTIC TRANSFUSION REACTION OVERVIEW

Adverse reactions to transfused blood products range from innocuous mild allergic reactions to the transmission of incurable . The spectrum of symptoms for such transfusion reactions is broad, but it is most crucial to recognize and rule out an acute hemolytic reaction as this is one of the most likely sources for morbidity and mortality in transfusion medicine. The most severe and acute reactions stem from trans- fusion of ABO incompatible red blood cells (RBCs). Preformed, naturally occurring isohemagglutinins (anti-A and anti-B anti- bodies) can result in rapid and robust intravascular hemolysis. However, the effects of exposure to blood components may not be immediately apparent, but can be delayed for weeks, months, or longer.

Case with Error

A 60-year-old Caucasian female is struck from behind while riding her bicycle, resulting in an open femoral fracture with signifi cant blood loss. The patient is transferred by ambulance to the closest emergency department (ED). Upon arrival in the ED, the patient is mildly hypoxic while breathing room air and the initial complete blood count (CBC) demonstrates a hemoglobin of 8.2 g/dL. The patient is emergently transfused with 2 units of RBCs in the ED before being transferred directly to surgery. Subsequent type and screen testing of a pretransfu- sion blood sample indicates her blood type to be A Rh-positive, with a negative indirect antibody screen. Following emergent surgery, the patient is transferred to a higher level care facility where she stabilizes and her laboratory values return to within normal limits. Twelve days after the accident, the patient is scheduled to be dis- charged home when her routine morning CBC shows an unanticipated

Eichbaum_87963_PTR_CH03_10-01-12_051-136.indd 53 11/10/12 1:00 PM 54 3. Errors Involving Specifi c Clinical Scenarios

1.5 g/dL decline in her hemoglobin. The clinical team considers a possible postoperative bleed, and an additional type and screen test is ordered preemptively in case surgical revision is required. The sur- gical site is inspected, but no obvious sign of bleeding can be identi- fi ed. Repeat laboratory testing demonstrates a mixed fi eld typing with a positive antibody screen. Direct antiglobulin testing (DAT) reveals binding that is 3+ IgG but no complement binding.

Clinical Pitfall

Failure to recognize a clinically signifi cant RBC antibody. The ordering of a blood type and screen in anticipation of a trans- fusion comprises two distinct processes—determination of the patient’s ABO/Rh blood type and screening for the presence of clinically sig- nifi cant antibodies in the patient’s serum. The blood typing test for the presence or absence of immunodominant sugar moieties (group A: N -acetylgalactosamine and group B: galactose) coupled with determi- nation of the presence or absence of Rh protein. The second compo- nent is the antibody screen, which is a two or three cell agglutination assay (the indirect Antiglobulin test), in which the patient’s serum is added to phenotypically characterized reagent RBCs to test for bind- ing of antibody to clinically signifi cant antigens, a process that results in agglutination of the RBCs. Any reactivity is considered a positive screen. Additional immunohematologic testing is performed to deter- mine the specifi city of the reaction. Errors can occur in both steps in this process, with discrepant forward and reverse typing or inaccurate antibody identifi cation.

Explanation and Consequences

This case illustrates a delayed hemolytic transfusion reaction (DHTR). The presence of two RBC populations on the forward type suggests that the RBC units transfused at the outside hospital emergency depart- ment were out-of-group units; furthermore, the new positive antibody screen suggests that the transfusion at the outside hospital resulted in formation of an alloantibody. Although numerous clinically signifi cant alloantibodies can result in a DHTR, the most frequently encountered

Eichbaum_87963_PTR_CH03_10-01-12_051-136.indd 54 11/10/12 1:00 PM Recommended Reading 55

antibody is anti-Jka (Kidd blood group). DHTRs are typically non- life-threatening, but their presence can be diffi cult to recognize, often presenting as a new onset occult anemia. This case demonstrates that RBC antibody levels can decrease below levels of laboratory detection leading to a negative antibody screen, but these antibodies can rapidly reappear via an anamnestic response upon re-exposure to cognate antigens contained in trans- fused blood products.

STANDARDS OF CARE

In delayed transfusion reactions, the offending antibody can be found in the plasma, on the surface of the RBCs, or both. A DAT should be performed on all suspected cases. Acid elution studies can further help to identify a potentially anamnestic response.

RECOMMENDED READING

Tormey CA, Stack G. The persistence and evanescence of blood group alloantibodies in men. Transfusion 2009;49:505–512. Amrein K, Schmid P, Mansouri Taleghani B. Delayed haemolytic transfusion reaction initially presenting as like syn- drome. Eur. J. Intern. Med. 2009;20:e122–e123.

EEichbaum_87963_PTR_CH03_10-01-12_051-136.inddichbaum_87963_PTR_CH03_10-01-12_051-136.indd 5555 111/10/121/10/12 1:001:00 PMPM 56 3. Errors Involving Specifi c Clinical Scenarios

IDENTIFICATION AND MANAGEMENT OF CLINICALLY SIGNIFICANT ALLOANTIBODIES IN PREGNANCY

OVERVIEW

Hemolytic disease of the fetus and newborn (HDFN) is a disorder describing the immune-mediated destruction of fetal or neonatal RBCs. The clinical repercussions of HDFN have been substantially mitigated since the 1970s with the advent of routine postpartum prophylactic Rh immune globulin (RhIG) therapy. Yet despite reductions of approximately 90% in alloimmunization to RhD, continued clinical hemovigilance is necessary to prevent fetal and neonatal demise. Clinically sig- nifi cant alloantibodies, other than anti-D, can cross the placenta, resulting in RBC destruction. With ongoing loss of RBCs, the developing neonate will attempt to compensate physiologically by increasing the formation of immature RBCs (reticulocyto- sis). With the increased oxygen demand as a consequence of the loss of RBCs, the neonatal heart will be driven to compensate by increasing output—which can in turn lead to cardiac failure and fetal demise.

Case with Error

The patient is a 24-year-old Asian American (G1P0) female who pres- ents for her fi rst OB/GYN appointment at week 8 of gestation (esti- mated by last menstrual period). The attending obstetrician notes that her past medical history is signifi cant for traumatic splenectomy at age 12 secondary to a motor vehicle collision, for which she received 2 units of RBCs. As part of her routine laboratory analysis, a blood type and screen is ordered. The patient’s blood type is group O Rh- positive. The indirect antibody screen is positive. Additional blood bank serologic evaluation reveals an alloantibody directed against the E antigen (RhCE blood group system) with a titer of 1 to 2. The

EEichbaum_87963_PTR_CH03_10-01-12_051-136.inddichbaum_87963_PTR_CH03_10-01-12_051-136.indd 5656 111/10/121/10/12 1:001:00 PMPM Identifi cation and Management of Clinically Signifi cant Alloantibodies 57

mother’s serologic RBC phenotype is performed, and she is E antigen negative. The biological father is unknown. Upon return to clinic, the physician orders repeat type and screen tests on a biweekly basis for 4 months, which remain positive, but the titer (now performed on mul- tiple testing platforms at 32 weeks gestation) is 1 to 256. The patient is transferred to the high-risk obstetrics unit (maternal-fetal medicine) for further evaluation of the developing fetus. The initial middle cere- bral artery (MCA) ultrasound of the fetus demonstrates signifi cantly increased fl ow rates.

Clinical Pitfall

Failure to recognize clinically signifi cant alloantibodies to minor RBC antigens that can cause severe HDFN. Not all antibodies (allo- or auto-antibodies) identifi ed within the blood bank are clinically signifi cant, or have been implicated in HDFN. It is important for the clinician to recognize those antibodies that are capable of causing HDFN, especially those that can result in severe HDFN. Only maternal antibodies that are IgG (IgG1, IgG3, and IgG4) are capable of crossing the placenta. This transfer is most evident within the third trimester. When evaluating patients with suspected HDFN, it is important to consider serial antibody titrations in the context of the patient’s overall clinical status. The determination of maternal antibody titers aids in identifying cases that may require intra-uterine transfusion or premature delivery. Serologic testing of the biologic father for pres- ence or absence of the suspected antigen can obviate the need for repeat testing/titrations.

Explanation and Consequences

Without the biologic father’s serologic antigen status, it is critical that this patient’s antibody titers and clinical status be closely followed. Antibodies directed against the RhCE blood group system can cause HDFN, but are far less frequent in HDFN than are other implicated RBC antibodies. Minor RBC antibodies can persist for decades, caus- ing signifi cant morbidity and/or mortality in subsequent pregnancies.

EEichbaum_87963_PTR_CH03_10-01-12_051-136.inddichbaum_87963_PTR_CH03_10-01-12_051-136.indd 5757 111/10/121/10/12 1:001:00 PMPM 58 3. Errors Involving Specifi c Clinical Scenarios

The RBC E antigen is expressed in approximately 30% to 35% of the US population. Without a rising titer, and with no other changes in symptoms from the expectant mother, additional tests are not indi- cated for this pregnancy. The critical titer cutoff point for clinical decision making regard- ing possible intrauterine transfusion, intrauterine exchange, or possible delivery is a titer ≥1 to 32. It is important that this patient be followed closely and that the same methodology(ies) be used for ongoing moni- toring of the antibody level. There have been rare case of HDFN where the type and screen is positive and the identifi ed antibody is one that is typically considered clinically insignifi cant (eg, anti-Lea ). Antibody identifi cation should always be placed in the clinical context.

STANDARDS OF CARE

Fetal transfusion is most commonly performed using blood group O Rh-negative that is compatible with the implicated maternal anti- body. These units should be leukoreduced, irradiated, and relatively fresh. Multiple transfusions can be performed to support the fetus through gestation to delivery. It is important to note that pregnant women receiving multiple transfusions secondary to HDFN often form multiple RBC antibodies.

RECOMMENDED READING

Joy SD, Rossi KQ, Krugh D, et al. Management of pregnancies complicated by anti-e alloimmunization. Obstet. Gynecol. 2005; 105:24–28. Vaughn JI, Manning M, Warwick RM, et al. Inhibition of erythroid progenitor cells by anti-kell antibodies in fetal alloimmune anemia. N. Engl. J. Med . 1998;338:798–803.

EEichbaum_87963_PTR_CH03_10-01-12_051-136.inddichbaum_87963_PTR_CH03_10-01-12_051-136.indd 5858 111/10/121/10/12 1:001:00 PMPM Liberal Versus Restrictive Transfusion Strategies 59

LIBERAL VERSUS RESTRICTIVE TRANSFUSION STRATEGIES

OVERVIEW

Postsurgical anemia increases a patient’s risk of death, particularly in the immediate postoperative setting and in the intensive care unit. The degree of asymptomatic anemia and subsequent decision to transfuse RBCs is a complex decision. Historical transfusion thresholds, coupled with non-evidence- based blood product ordering, has resulted in a renewed drive towards evidence-based decision making for RBC transfusions. The overall goal of RBC transfusion is to enhance tissue oxy- genation, but the decision to transfuse must be balanced against other factors, such as potential risks of infection, alloimuniza- tion, and general availability of the blood components.

Case with Error

An 81-year-old male with signifi cant past medical history of smoking (1 pack/day for 56 years), type II diabetes mellitus, and hypertension, is admitted to the orthopedic department after falling from a sitting position onto his left hip. The patient has radiographic evidence of a left hip femoral neck fracture and is taken to the operating room for surgical correction. Postoperatively, the patient is asymptomatic and oxygenating at 100% on room air. The patient’s pretransfusion hemo- globin level was 10.7 g/dL. The clinical team decides to transfuse 2 units of crossmatch compatible RBCs. The posttransfusion hemo- globin level is 13 g/dL.

Clinical Pitfall

Failure to follow evidence-based guidelines for managing asymptom- atic anemia.

EEichbaum_87963_PTR_CH03_10-01-12_051-136.inddichbaum_87963_PTR_CH03_10-01-12_051-136.indd 5959 111/10/121/10/12 1:001:00 PMPM 60 3. Errors Involving Specifi c Clinical Scenarios

Historic thresholds for transfusion were once summarized by the “10/30 rule” by which clinicians ordered blood products for any hemoglobin value less than 10 g/dL, or for a hematocrit of less than 30%. These guidelines are not evidence based and result in unneces- sary transfusion with associated risks.

Explanation and Consequences

Studies of adult patients in intensive care units (ICUs) recommend a hemoglobin threshold of 7 g/dL as a transfusion trigger in patients without a signifi cant history of coronary artery disease. Studies in adult orthopedic patients with a signifi cant history of cardiac disease have reinforced the validity of the notion that a restrictive threshold for RBC transfusion is not inferior to a liberal threshold.

STANDARDS OF CARE

Studies from intensive care units have suggested a “trigger” for trans- fusion at a hemoglobin value of 7 g/dL. Controversy still remains regarding the effect of the age of the transfused RBC unit(s), with older units likely being inferior in quality (the “storage lesion” effect).

RECOMMENDED READING

Carson JL, Duff A, Poses RM, et al. Effect of anaemia and cardio- on surgical mortality and morbidity. Lancet 1996; 348:1055–1060. Hérbert PC, Wells G, Blajchman MA, et al. A multicenter, randomized, controlled clinical trial of transfusion requirements in critical care. Trans- fusion Requirements in Critical Care Investigators, Canadian Critical Care Trials Group. N. Engl. J. Med . 1999;340:409–417. Hébert PC, Wells G, Tweeddale M, et al. Does transfusion practice affect mortality in critically ill patients? Transfusion Requirements in Critical Care (TRICC) Investigators and the Canadian Critical Care Trials Group. Am. J. Respir. Crit. Care Med . 1997;155:1618–1623.

EEichbaum_87963_PTR_CH03_10-01-12_051-136.inddichbaum_87963_PTR_CH03_10-01-12_051-136.indd 6060 111/10/121/10/12 1:001:00 PMPM Non-Evidence-Based Practices in Prophylactic Platelet Transfusions 61

NON-EVIDENCE-BASED PRACTICES IN PROPHYLACTIC PLATELET TRANSFUSIONS FOR MINOR PROCEDURES OVERVIEW

Lumbar puncture (LP) is a common diagnostic and thera- peutic procedure performed in both adult and pediatric patient populations. There is range of potential complications that can occur with LPs that includes headache, backache, cerebellar herniation, trauma to the conus medullaris, iatrogenic meningitis, and bleeding. Despite these complications, throm- bocytopenia is by itself not a major contraindication to perform- ing an LP on a patient. Nonetheless, the prophylactic platelet transfusion thresholds prior to performing an LP are controver- sial. Physicians performing this procedure are concerned about the risk for permanent neurologic injury.

Case with Error

A 13-year-old female presents with a new onset of severe fatigue and prolonged epistaxis. Evaluation of a peripheral blood smear demon- strates circulating blasts, subsequently confi rmed by owfl cytometry and cytogenetics as B-cell lymphoblastic leukemia. For staging pur- poses, an LP is required. On the day of the procedure, the platelet count is 64,000/μL. The clinical team requests 2 units of apheresis platelets as prophylaxis prior to performing the LP. Ultimately, 1 unit of apheresis platelets is transfused prior to the procedure, while the second is transfused during the procedure. The patient tolerates both transfusions and the LP without incident.

Clinical Pitfall

Failure to recognize evidence-based platelet transfusion guidelines for minimally invasive procedures.

Eichbaum_87963_PTR_CH03_10-01-12_051-136.indd 61 11/10/12 1:00 PM 62 3. Errors Involving Specifi c Clinical Scenarios

The patient was unnecessarily transfused with 2 units of platelets. This not only presents additional risks to the recipient but is also a waste of this blood product resource that has a relatively short shelf life of 5 days.

Explanation and Consequences

The precise threshold for prophylactic transfusion of platelets for minimally invasive procedures is an ongoing controversy. The most conservative estimates for safety have set a threshold of greater than 50,000/μL, but more recent studies have challenged this threshold. The lowest published threshold for pediatric patients with thrombocyt- openia has shown that these patients can safely undergo LP without an increased risk of bleeding complications by sustaining a platelet count of just greater than 10,000/μL.

STANDARDS OF CARE

Data to support a specifi c platelet count are limited. However, pub- lished reports on retrospective analyses and expert opinion support platelet counts greater than 10,000/μL to 50,000/μL.There are no data to support the need for a platelet count of 100,000/μL.

RECOMMENDED READING

Howard SC, Gajjar A, Ribeiro RC, et al. Safety of lumbar puncture for children with acute lymphoblastic leukemia and thrombocytopenia. JAMA 2000;284:2222–2224. Kruskall MS. The perils of platelet transfusions. N. Engl. J. Med . 1997;337:1914–1915.

EEichbaum_87963_PTR_CH03_10-01-12_051-136.inddichbaum_87963_PTR_CH03_10-01-12_051-136.indd 6262 111/10/121/10/12 1:001:00 PMPM Molecular Differences in the RhD Protein and the Need for Rh Immune Globulin 63

MOLECULAR DIFFERENCES IN THE RhD PROTEIN AND THE NEED FOR Rh IMMUNE GLOBULIN

OVERVIEW

The RhD protein is the most immunogenic of the Rh proteins. These proteins are expressed exclusively on the erythroid cells and can pose a signifi cant risk for hemolytic transfusion reactions, as well as HDFN. The RhD protein is a transmembrane protein that has numerous variants based on alterations in the DNA sequence. Alterations in the DNA sequence often translate to structural changes in the RhD protein. Various mutations (deletion, single nucleotide polymorphism, and pseu- dogene) have been shown to alter different regions within the cytoplasmic, transmembrane, and external portion of the protein, each resulting in unique phenotypic protein expression.

If the protein alteration is on the external surface of the RBC, the changes in the three-dimensional structure of the protein are exposed to the immune system and can elicit different immune surveillance patterns. As a result, when patients with mutated extracellular branches of the RhD protein are transfused with RBCs from nonmutated donors, the recipi- ent’s immune system may recognize the external protein changes as foreign and mount an immune response. This structural alteration in the exposed extracellular part of the protein constitutes the biological basis for the immune response, termed “partial D.” If the mutation in the RhD protein occurs on the intracellular portion of the protein, the resulting changes most frequently result in only a reduced level of expression of the RhD protein.

Case with Error

A 38-year-old G2P1 female with no history of HDFN, no prior trans- fusions, and no other signifi cant medical history presents for clinical

EEichbaum_87963_PTR_CH03_10-01-12_051-136.inddichbaum_87963_PTR_CH03_10-01-12_051-136.indd 6363 111/10/121/10/12 1:001:00 PMPM 64 3. Errors Involving Specifi c Clinical Scenarios

evaluation at 37 weeks gestation. The patient recently immigrated to the United States from Southeast Asia and denies having had any prior prenatal care. The patient’s gestational measurements were large for her dates, and an emergent ultrasound demonstrates severe macroso- mia (weight 5300 g), polyhydramnios, , and car- diomegaly. An MCA Doppler scan shows increased maximum systolic velocity. Maternal laboratory test values are negative for infectious dis- ease markers. Her blood type is reported as A Rh-positive. The child is immediately delivered by cesarean section, and both mother and child are transferred to a tertiary medical institution. Upon arrival at this hos- pital, the laboratory testing is repeated. Results indicate the maternal blood type to be group A Rh-negative with a positive screen for anti-D antibody at a titer of 1 to 256. This signifi cantly elevated anti-D titer necessitates daily exchange transfusions of the baby, which ultimately expires from hydrops fetalis.

Clinical Pitfall

Failure to recognize the limitations of weak D/partial D antigen test- ing and their clinical signifi cance. Inadequate prenatal care can be devastating, resulting in an in creased morbidity and mortality. Accurate RhD identifi cation is critical to establish the need for both Rh immunoprophylaxis (RhIG) and/or the need for Rh-negative blood products. Once a pregnant female is identifi ed as having a clinically signifi cant alloantibody, serial anti- body titers should be performed to monitor the maternal response to fetal antigens. Once the titer reaches or surpasses a critical titer, addi- tional testing, surveillance, and possible intrauterine or postpartum neonatal exchange therapy should be initiated.

Explanation and Consequences

The original blood type of the patient in this case was incorrectly assigned, and as a result clinical monitoring and surveillance for poten- tial HDFN from anti-D antibody was not performed. Currently, there is considerable controversy regarding the universal practice of screen- ing pregnant patients for the weak D antigen. As molecular testing

EEichbaum_87963_PTR_CH03_10-01-12_051-136.inddichbaum_87963_PTR_CH03_10-01-12_051-136.indd 6464 111/10/121/10/12 1:001:00 PMPM Recommended Reading 65

continues to reveal novel mutations within the RhD gene, the clinical implications for managing these patients may change.

STANDARDS OF CARE

The fi rst prenatal visit should include a thorough pregnancy history, as well as blood type and screen. At present, many FDA-approved reagents, used for anti-D testing, combine a monoclonal IgM anti- body, and a monoclonal or polyclonal IgG antibody, for the deter- mination of a weak D. When the RhD type of a patient is undertaken, a weak D test is not part of the standard of care. Only in situations where the infant of a mother is at risk for D alloimmunization, will a weak D test be performed.

RECOMMENDED READING

Denomme GA, Wagner FF, Fernandes BJ, et al. Partial D, weak D types, and novel RHD alleles among 33,864 multiethnic patients: implications for anti-D alloimmunization and prevention. Transfusion 2005;45:1554–1560. Flegel, WA. How I manage donors and patients with a weak D pheno- type. Curr. Opin. Hematol . 2006;13:476–483. Garratty G. Do we need to be more concerned about weak D antigens? Transfusion 2005;45:1547–1551.

EEichbaum_87963_PTR_CH03_10-01-12_051-136.inddichbaum_87963_PTR_CH03_10-01-12_051-136.indd 6565 111/10/121/10/12 1:001:00 PMPM 66 3. Errors Involving Specifi c Clinical Scenarios

RECOGNITION OF IMMUNE-MEDIATED HEMOLYSIS IN A PEDIATRIC PATIENT

OVERVIEW

Approximately 20% of children in the United States will be anemic at some stage prior to their 18th birthday. The cause for this anemia is quite variable. Although immune hemolytic more commonly affect adult populations, pediatric patients can also be affected. When present in pediatric popula- tions it frequently follows a postinfectious period. Immune- mediated anemias can be further classifi ed, according to the implicated antibody-antigen reaction, as follows: warm autoim- mune hemolytic anemia (WAIHA), cold agglutinin disease, mixed type autoimmune-mediated hemolytic anemia, drug- induced immune hemolytic anemia (DIIHA), and paroxysmal cold hemoglobinuria. Each of the above disorders implicates an autoantibody directed against an RBC antigen that reduces the lifespan of the circulating RBC. The clinical severity of these disorders is variable and depends on the strength of the antibody- antigen interaction, the ability to fi x complement on the RBC surface, and the range of temperatures at which the antibody is capable of binding to RBCs.

Case with Error

A 20-month-old, full-term, healthy Hispanic female born to a G2P2 mother who received appropriate prenatal care presents to the ED with increasing fever, having experienced a tonic–clonic seizure lasting 15 seconds. The ED physician notices that the child is pale, but he makes the clinical diagnosis of febrile seizure and discharges the child with instructions to the family about appropriate care. The fam- ily returns 14 hours later with complaints that the child now looks “very pale” and that the urine is now “dark brown.” The laboratory

EEichbaum_87963_PTR_CH03_10-01-12_051-136.inddichbaum_87963_PTR_CH03_10-01-12_051-136.indd 6666 111/10/121/10/12 1:001:00 PMPM Recognition of Immune-Mediated Hemolysis in a Pediatric Patient 67

results on re-admission are shown in the table below. The staff notes that the biological father, the primary caregiver for the patient, also has signifi cant signs and symptoms of an acute respira- tory disorder.

Patient Reference Result Range Blood Type: Group O Rh-Positive WBC count, ×103 / μL 11.69 3.2–9.2 RBC count, ×106 / μL 1.62 4.21–5.61 Hemoglobin, g/dL 6.6 12.3–16.3 Hematocrit, % 17.9 37.4–47.0 Platelet count, ×103 / μL 210 143–398 Reticulocyte count, ×10 6 / μL 0.052 0.027–0.107 Lactate dehydrogenase (LDH), U/L 1876 91–223 Total bilirubin, mg/dL 6.1 0.2–1.1 Conjugated bilirubin, mg/dL 0.5 0–0.2 Aspartate aminotransferase (AST), U/L 106 7–36 Urine Analysis Protein 3 + None Blood 3 + None International normalized ratio (INR) 1.1 0.9–1.2 Partial thromboplastin time, seconds 25 25–35

In light of these laboratory results, a DAT is performed yielding the following results: 3+ binding for complement C3d; negative for IgG; negative for saline control. An additional cold agglutinin screen is performed. Results show a panreactivity of 2+ with reagent cells at 4°C, and no reactivity at 30°C or 37°C. Based upon these results, the transfusion medicine service recommends a Donath-Landsteiner (DL) test.

EEichbaum_87963_PTR_CH03_10-01-12_051-136.inddichbaum_87963_PTR_CH03_10-01-12_051-136.indd 6767 111/10/121/10/12 1:001:00 PMPM 68 3. Errors Involving Specifi c Clinical Scenarios

Clinical Pitfall

Failure to evaluate the patient for clinically signifi cant immune-mediated hemolysis, resulting in delay of a correct diagnosis. The differential diagnosis for anemia in a child is broad. Per- forming the appropriate evidence-based tests in a systematic fashion is essential to arriving at the correct diagnosis. A positive DAT that shows binding of complement only and not IgG, together with a posi- tive cold agglutinin screen, should prompt the evaluation for a bipha- sic hemolysin (DL IgG antibody).

Explanation and Consequences

RBCs strongly coated with complement (C3d) as detected by the DAT require further investigation. Given the fi ndings of the DAT, a DL serologic evaluation was performed. Although cold agglutinin disease is in the differential diagnosis, this disease entity is usually seen in adult patients, with a peak incidence in the seventh decade of life. The patient had a positive DL test, identifying a positive biphasic hemolysin; this autoantibody had specifi city to the P RBC antigen, reacted with RBCs at reduced temperatures (extremities), and was shown to fi x complement. As blood returns to the body from the periphery, it becomes warmed, causing complement-coated cells to be cleared by the liver and spleen. Additional intravascular lysis occurs secondary to this complement deposition. Paroxysmal cold hemoglobinuria (PCH) was historically associated with syphilis, but the incidence of this disease has declined in the United States. Numerous studies have demonstrated viral (measles, mumps, cytomegalovirus, mononucleo- sis, mycoplasma pneumonia, and varicella) and bacterial infections (Klebsiella pneumoniae, Escherichia coli, and Haemophilus infl uenzae) as precursors to PCH. In addition, vaccinations have been implicated. Treatment is typically supportive observation and maintenance of a warm environment.

EEichbaum_87963_PTR_CH03_10-01-12_051-136.inddichbaum_87963_PTR_CH03_10-01-12_051-136.indd 6868 111/10/121/10/12 1:001:00 PMPM Recommended Reading 69

STANDARDS OF CARE

The DL test should be considered when a pediatric patient in the postinfectious period presents with a positive DAT with isolated complement fi xation.

RECOMMENDED READING

Sokol RJ, Hewitt S, Stamps BK. Autoimmune hemolysis: an 18-year study of 865 cases referred to a regional transfusion center. Br. Med. J. (Clin. Res. Ed) . 1981;282:2023–2027. Sokol RJ, Booker DJ, Stamps R. The pathology of autoimmune hemo- lytic anemia. J. Clin. Pathol . 1992;45:1047–1052. Wynn RF, Stevens RF, Bolton-Maggs PH, et al. Paroxysmal cold hemo- globinuria of childhood: a review of the management and unusual pre- senting features of six cases. Clin. Lab. Hematol . 1998;20:373–375.

Eichbaum_87963_PTR_CH03_10-01-12_051-136.indd 69 11/10/12 1:00 PM 70 3. Errors Involving Specifi c Clinical Scenarios

INAPPROPRIATE PLATELET TRANSFUSION FOR PATIENTS ON ASPIRIN

OVERVIEW

Platelet transfusions are critical interventions intended to aid in maintaining hemostasis. There are two licensed platelet products available in the United States: apheresis plate- lets and whole blood-derived platelet concentrates. Currently, the majority of platelet transfusions in the United States involve the use of apheresis platelet products (single donor) which, by AABB standards, should contain at least 3 × 10 11 platelets in 90% of sampled units. Transfusion “triggers” or thresholds dif- fer considerably depending on the clinical scenario and the patient’s underlying disease. For example, in neurosurgical interventions, the consensus platelet threshold for the patient preoperatively is greater than 100,000/μL.

Concomitant use of antiplatelet medications (eg, acetylsalicylic acid) can complicate the effectiveness of platelet transfusions. Aspirin irre- versibly inhibits both cyclooxygenase-1 and 2 enzymes via acetylation. This irreversible inhibition prevents arachidonic acid conversion to thromboxane A2. The net effect of inhibiting thromboxane A2 genera- tion is to reduce platelet aggregation.

Case with Error

A 71-year-old female with a signifi cant past medical history of coro- nary artery disease presents to the ED with mental status changes. Stat neuroimaging demonstrates an acute-on-chronic unilateral sub- dural hematoma. The patient is transfused with a single unit of apher- esis platelets in the ED. The patient is hemodynamically stable, and hourly neurological checks do not reveal progression of symptoms. Twenty-two hours following presentation to the ED, the patient is sent for evacuation of the hematoma. The patient’s preoperative platelet

EEichbaum_87963_PTR_CH03_10-01-12_051-136.inddichbaum_87963_PTR_CH03_10-01-12_051-136.indd 7070 111/10/121/10/12 1:001:00 PMPM Inappropriate Platelet Transfusion for Patients on Aspirin 71

count is 371,000/μL. The clinical team requests 3 additional units of apheresis platelets. There is no patient documentation of antiplatelet medication, platelet dysfunction, or underproduction of platelets in the bone marrow.

Clinical Pitfall

Failure to consider aspirin pharmacology in platelet transfusion therapy. The prescription of aspirin for underlying chronic medical con- ditions (eg, coronary artery disease) as well as for minor aches and pains, can complicate an emergent case of suspected intracranial bleeding. The half-life of aspirin is dose dependent. For a dose of 300 to 600 mg, the half-life is approximately 3 hours. In intracranial bleeding, the increasing volume of a hematoma has been shown to be a poor prognostic indicator. Given that expan- sion of a hematoma most frequently occurs within 6 hours of onset, rapid administration of platelet products is frequently performed in an effort to stabilize the bleed. Such therapeutic intervention is not always effective and can introduce infectious and noninfectious com- plications from exposure to blood products. Rapid point-of-care test- ing is underutilized to determine whether aspirin (or other antiplatelet medications) have inhibited platelet function.

Explanation and Consequences

The patient was on a standard daily dose of aspirin prior to admis- sion. Commercially available assays for platelet function can have a turn-around time of about 30 minutes. One example of such an assay is the VerifyNow test that can determine a patient’s response to aspirin. In the above case, without performing this test, the clinical decision to ensure adequate hemostasis was to transfuse additional platelets despite an adequate platelet count. Furthermore, the number of units of platelets requested (4 in total) was not based on the known antici- pated post-transfusion increment of 30,000 to 50,000 platelets/μL. Failure to achieve adequate hemostasis in both the immediate and the perioperative settings for patients with intracranial hemorrhage is

EEichbaum_87963_PTR_CH03_10-01-12_051-136.inddichbaum_87963_PTR_CH03_10-01-12_051-136.indd 7171 111/10/121/10/12 1:001:00 PMPM 72 3. Errors Involving Specifi c Clinical Scenarios

associated with worse clinical outcomes. The desire to produce hemo- stasis in a bleeding patient should be weighed against platelet inven- tory management. Rapid assessment of platelet function can help determine which patients might benefi t from platelet transfusions. The underutilization of such laboratory tests may lead to unneces- sary transfusions, which may have immediate clinical consequences (eg, transfusion reactions) or result in long-term complications (eg, transfusion-transmitted infections, alloimmunization).

STANDARDS OF CARE

For , a consensus preprocedure platelet count is 100,000/␮L, although the optimal count remains to be determined. Rapid platelet function testing can be useful for optimizing platelet transfusions.

RECOMMENDED READING

Broderick JP, Brott TG, Duldner JE, et al. Volume of intracerebral hemorrhage. A powerful and easy-to-use predictor of 30-day mortality. Stroke 1993;24:987–993. Brott T, Broderick J, Kothari R, et al. Early hemorrhage growth in patients with intracerebral hemorrhage. Stroke 1997;28:1–5. Schramm B, Leslie K, Myles PS, Hogan CJ. Coagulation studies in pre-operative neurosurgical patients. Anesth. Intensive Care 2001; 29:388–392.

EEichbaum_87963_PTR_CH03_10-01-12_051-136.inddichbaum_87963_PTR_CH03_10-01-12_051-136.indd 7272 111/10/121/10/12 1:001:00 PMPM Unexpected Posttransfusion Purpura (PTP) 73

UNEXPECTED POSTTRANSFUSION PURPURA (PTP)

OVERVIEW

The majority of platelet transfusions in the United States are administered prophylactically, secondary to thrombo- cytopenia. These transfusions are not devoid of risk and are sus- ceptible to both infectious and noninfectious complications. Although acute reactions are more readily identifi ed, delayed reactions to platelet transfusion can nonetheless cause signifi - cant morbidity and occasional mortality. Although rare, immu- nologic destruction of transfused platelets can occur. In pregnant patients, exposure to fetal platelet antigens can stimulate a mater- nal immune response leading to the severe clinical consequences of neonatal alloimmune thrombocytopenia (NAIT). Patients exposed to novel platelet antigens from platelet transfusions can also develop an immune response, causing a precipitous decrease in the platelet count that is usually only identifi ed with serial measurements of the platelet count.

Case with Error

A 65-year-old male with poorly controlled type 2 diabetes is admit- ted to hospital for a left-sided below-the-knee amputation. Five years ago, the patient had undergone a complex aorto-femoral bypass pro- cedure on the same leg, requiring a prolonged hospital stay and mul- tiple RBC transfusions. Outside medical records are not available for review. The patient’s current surgical case is completed without incident (intraoperative blood use: 2 units of RBCs and 1 unit of apheresis platelets) until postoperative day 7, when the patient has an acute decrease in his platelet count from baseline (admission platelet count: 154,000/ ␮L; postoperative day 7: 14,000 /␮L). The clinical team suspects

EEichbaum_87963_PTR_CH03_10-01-12_051-136.inddichbaum_87963_PTR_CH03_10-01-12_051-136.indd 7373 111/10/121/10/12 1:001:00 PMPM 74 3. Errors Involving Specifi c Clinical Scenarios

heparin-induced thrombocytopenia (HIT), and all heparin based medi- cations and fl ushes are discontinued. The patient is transfused with a single unit of apheresis platelets. However, his platelet count continues to decline to 4,000 /␮L. The patient’s fi brinogen, PT, PTT, D-dimer, and peripheral blood cultures are all normal. The nursing staff notes pro- found skin changes, with extensive purpura that include the oral mucosa.

Clinical Pitfall

Failure to consider posttransfusion purpura in the differential diagno- sis of thrombocytopenia. The differential diagnosis of thrombocytopenia is broad and includes surgical blood loss, occult blood loss, and immune-mediated platelet destruction. Thrombocytopenia due to posttransfusion pur- pura (PTP) is generally underdiagnosed. In a surgical case with throm- bocytopenia, heparin-induced thrombocytopenia (HIT) is more likely than posttransfusion purpura. In the above case, as is typical for PTP, the platelet count was signifi cantly lower than what is typically found in patients with HIT (usually >20,000 /␮L).

Explanation and Consequences

Although occurring more frequently in multiparous females due to the mother’s exposure to fetal platelet antigens, PTP can also affect male patients. This rare acquired form of thrombocytopenia has an onset of approximately 7 to 14 days posttransfusion, allowing the recipient’s immune system the necessary time to recognize foreign antigens and develop an alloantibody response. The alloantibody most frequently developed is against the human platelet antigen 1a (HPA- 1a). Approximately 2% of the population is HPA-1b/1b, rendering them most susceptible to PTP as they could develop antibodies to the HPA-1a antigen. These alloantibodies can lead to rapid decrease in the platelet count, often eliminating all transfused platelets and even destroying native (“bystander”) platelets. The exact mechanism for this “bystander effect” is not currently understood although several mechanisms have been proposed. The clinical course of PTP is vari- able, but in severe cases, the mortality rate can approach 10% to 20%.

EEichbaum_87963_PTR_CH03_10-01-12_051-136.inddichbaum_87963_PTR_CH03_10-01-12_051-136.indd 7474 111/10/121/10/12 1:001:00 PMPM Recommended Reading 75

STANDARDS OF CARE

PTP must be considered in the differential diagnosis of thrombocy- topenia in patients exposed to platelet transfusions and in women who have been pregnant, particularly when the platelet count is very low. In cases of life-threatening bleeding, HPA-1a/1a negative units should be used for platelet transfusions unless platelet antigen testing sug- gests otherwise.

RECOMMENDED READING

Buchta C, Felfernig M, Höcker P, et al. Stability of coagulation factors in thawed, solvent/detergent-treated plasma during storage at 4 degrees C for 6 days. Vox Sang . 2004;87:182–186. Morrison FS, Mollison PL. Post-transfusion purpura. N. Engl. J. Med . 1966;275:243–248. Shtalrid M, Shvidel L, Vorst E, et al. Post-transfusion purpura: a chal- lenging diagnosis. Isr. Med. Assoc. J . 2006;8:672–674.

EEichbaum_87963_PTR_CH03_10-01-12_051-136.inddichbaum_87963_PTR_CH03_10-01-12_051-136.indd 7575 111/10/121/10/12 1:001:00 PMPM 76 3. Errors Involving Specifi c Clinical Scenarios

PHENOTYPE MATCHING TO MITIGATE ALLOIMMUNIZATION IN SICKLE CELL DISEASE OVERVIEW

Worldwide, sickle cell disease (SCD) affects many differ- ent patient populations. In the United States, most cases of SCD affect African Americans. Despite multiple clinical trials demonstrating hydroxyurea as an effective therapy for acute life-threatening complications of SCD, RBC transfusion con- tinues to be the more regularly performed therapy. Maintaining the hemoglobin S percentage below 30% during chronic trans- fusion therapy has been shown to reduce the risk of stroke.

The potential for increased alloimmunization and consequent hemo- lytic transfusion reactions is increased by the high degree of vari- ability in antigen matching between the recipient and donor blood products. It is known that the rate of alloimmunization in patients with SCD is disproportionately higher than in other chronically trans- fused patient populations. The etiology of this discrepancy is not fully understood.

Case with Error

A 15-year-old African American male with SCD who has been pre- viously maintained on hydroxyurea and subsequently transitioned to a chronic transfusion program (4–5 unit RBC exchange every 5 weeks for the past 2 years) goes on summer vacation with his family. While on vacation the patient is unable to keep his scheduled appoint- ment for RBC exchange and develops an acute pain crisis, similar to prior episodes that he had experienced before the exchange program was initiated. The patient’s stat hemoglobin level in the ED is low at 6.2 g/dL.The outside hospital provides symptomatic management of his pain with narcotics and transfuses the patient with 2 units of crossmatch compatible RBCs while in the ED.

Eichbaum_87963_PTR_CH03_10-01-12_051-136.indd 76 11/10/12 1:00 PM Phenotype Matching to Mitigate Alloimmunization in SCD 77

Clinical Pitfall

Failure to recognize the potential risk of alloimmunization in a chroni- cally transfused patient. A type and screen that was sent to the blood bank laboratory prior to the patient being transfused showed the patient’s blood type to be A Rh-positive, with a negative antibody screen. Three weeks later, when the patient returned for his regularly scheduled RBC exchange therapy, his indirect antibody screen was positive, with two antibod- ies identifi ed: anti-Kell (Kell blood group system) and anti-E (RhCE blood group system).

Explanation and Consequences

The patient had been on a regular RBC exchange program in which his immunohematology evaluation was normal. His RBCs had been serologically phenotyped as positive for C, c, and e, but nega- tive for E and K. The RBC units used in his exchange had been purposely matched to be negative for E and K. The newly identi- fi ed alloantibodies are clinically signifi cant and capable of causing a hemolytic transfusion reaction in ensuing transfusions. Further- more, given the antigen frequency in the US blood donor population (~9% K antigen positive and ~30% E antigen positive), the total num- ber of potential crossmatch compatible units available for transfusion of this patient is small. Although serological testing of donor/recipient RBCs is essential in order to reduce the rate of alloimmunization in chronically trans- fused patients, such testing of SCD patients is quite variable in the United States. Approximately 65% of North American transfusion laboratories provide only ABO and Rh matched units for nonallo- immunized SCD patients, despite ample evidence that prospective phenotype-matching of RBCs for transfusion in this patient popula- tion considerably reduces the rate of alloimmunization and hemolytic transfusion reactions. The alloimmunization rate in SCD patients ranges from 18% to 46%, with a rate of 1.7 to 3.8 alloantibodies per 100 units transfused.

EEichbaum_87963_PTR_CH03_10-01-12_051-136.inddichbaum_87963_PTR_CH03_10-01-12_051-136.indd 7777 111/10/121/10/12 1:001:00 PMPM 78 3. Errors Involving Specifi c Clinical Scenarios

STANDARDS OF CARE

For patients requiring chronic transfusion support (eg, SCD, thalas- semia, and other hemoglobinopathies), there is currently no univer- sal standard; however, studies have demonstrated that prophylactic phenotype matching for the RhCE and K antigens results in reduc- tion in alloimmunization rates to minor RBC antigens.

RECOMMENDED READING

LaSalle-Williams M, Nuss R, Le T, et al. Extended red blood cell antigen matching for transfusions in sickle cell disease: a review of a 14-year experience from a single center. Transfusion 2011; 51:1732–1739. Osby M, Shulman IA. Phenotype matching of donor red blood cell units for nonalloimmunized sickle cell disease patients: a survey of 1182 North American laboratories. Arch. Pathol. Lab. Med . 2005; 129:190–193. Vichinsky EP, Luban, NL, Wright E, et al. Prospective RBC phenotype matching in a stroke-prevention trial in sickle cell anemia: a multi- center transfusion trial. Transfusion 2001;41:1086–1092.

EEichbaum_87963_PTR_CH03_10-01-12_051-136.inddichbaum_87963_PTR_CH03_10-01-12_051-136.indd 7878 111/10/121/10/12 1:001:00 PMPM Refusing Blood Transfusion, When Patient and Physician Beliefs Fail to Align 79

REFUSING BLOOD TRANSFUSION, WHEN PATIENT AND PHYSICIAN BELIEFS FAIL TO ALIGN

OVERVIEW

The medical and surgical management of patients who refuse allogeneic and/or autologous blood is complex. The genesis of such decisions can be religious, cultural, and/or per- sonal in nature, and is not limited to patients who identify them- selves as Jehovah’s Witness. The refusal of blood transfusion may confl ict with the medical responsibility for preserving life, yet the decision by a competent and informed adult patient to decline treatment must be respected, even if there are clear med- ical indications for such a transfusion.

Case with Error

A 30-year-old female with a history of myomectomy of the uterus presents at 26 weeks gestation in premature labor. The attending phy- sician decides that a vaginal delivery is contraindicated given the fetal position and past surgical history. In obtaining consent for a cesar- ean delivery, the patient is told there is 50% probability that she will require a transfusion. The physician’s opinion is that a failure to trans- fuse would not place the fetus at risk, but would likely result in mater- nal demise. The patient and the biologic father both agree that she will not accept a transfusion under any circumstances, and would rather die than receive blood. The hospital then petitions the local circuit court to appoint a temporary guardian who would consent to the transfusion, if the physician decides it is necessary. The court rules against the hospi- tal, stating that the patient is competent to refuse transfusion. The case is then emergently sent to the Court of Special Appeals. The mother successfully delivers the child without incident before the court’s deci- sion is rendered.

EEichbaum_87963_PTR_CH03_10-01-12_051-136.inddichbaum_87963_PTR_CH03_10-01-12_051-136.indd 7979 111/10/121/10/12 1:001:00 PMPM 80 3. Errors Involving Specifi c Clinical Scenarios

Clinical Pitfall

Failure to respect and understand the patient’s autonomy regarding their care. Recognizing the patient’s legal right to autonomy is essential in determining the appropriate medical or surgical therapy. Nonetheless, a patient’s right to refuse transfusion does not preclude the provider from understanding how such a decision was reached. Particularly with Jehovah’s Witness patients, the decision to accept or reject allogeneic and/or autologous blood, solid organ transplants, or the wide array of available blood components (that includes fractionated plasma-derived therapy such as IVIG, albumin, antivenoms, and artifi cial blood prod- ucts) can be quite idiosyncratic. In obtaining such a patient’s consent, all discussions about the risk/benefi ts of transfusion should be thoroughly documented.

Explanation and Consequences

This case highlights the legal right of a competent adult to make deci- sions about receiving a transfusion. Physicians who disagree with the patient’s decision to refuse transfusion may, in nonemergent cases, consider referring such a patient to another provider. Developing a good rapport with patients who have not consented to transfusion is essential; many hospitals have liaison groups (including risk manage- ment) that can assist with such communication. For scheduled procedures or prolonged hospitalizations, limit- ing the iatrogenic loss of blood may mitigate transfusion needs. In the setting of adequate contingency planning and advanced medi- cal directives, appropriate support can be provided to patients who decline transfusion. Alternatives to transfusion might also be consid- ered. These include preoperative recombinant erythropoietin adminis- tration, intraoperative cell savage (cell saver and acute normovolemic hemodilution), and postoperative hyperbaric therapy.

EEichbaum_87963_PTR_CH03_10-01-12_051-136.inddichbaum_87963_PTR_CH03_10-01-12_051-136.indd 8080 111/10/121/10/12 1:001:00 PMPM Recommended Reading 81

STANDARDS OF CARE

A competent adult has the legal right to make decisions about receiving a blood transfusion. In the event of a life-threatening blood loss, a court order is not required for transfusion of a minor (even if the legal guardian says no).

RECOMMENDED READING

Bamberger DH. Mercy Hospital, Inc. v. Jackson: a recurring dilemma for health care providers in the treatment of Jehovah’s Witnesses. MD Law Rev. 1987;46:514–532. Gohel MS, Bulbulia RA, Slim FJ, et al. How to approach major surgery where patients refuse blood transfusion (including Jehovah’s Witnesses). Ann. R. Coll. Surg. Engl. 2005;87:3–14. Rogers DM, Crookston KP. The approach to the patient who refuses blood transfusion. Transfusion 2006;46:1471–1477. Shah J, Fitz-Henry J. Peri-operative care series. Ann. R. Coll. Surg. Engl . 2011;93:265–267.

EEichbaum_87963_PTR_CH03_10-01-12_051-136.inddichbaum_87963_PTR_CH03_10-01-12_051-136.indd 8181 111/10/121/10/12 1:001:00 PMPM 82 3. Errors Involving Specifi c Clinical Scenarios

CRYPT ANTIGEN ACTIVATION, ADVERSE CONSEQUENCES

OVERVIEW

T activation, also known as the Thomsen-Hubener- Friedenrich phenomenon, is an enzymatic modifi cation of the RBC membrane proteins that exposes, or “activates,” the nor- mally “cryptic” (hidden) T, or Thomsen, antigen. The T antigen is normally sequestered on cellular membranes until it is exposed through the removal of N -acetyl-neuraminic acid (sialic acid) residues by neuraminidase, an enzyme produced by a variety of organisms such as Clostridia and Streptococcus pneumoniae . Adult plasma contains naturally preformed antibodies to crypt antigens. Such transfused plasma can thus propagate an ongoing immune-mediated hemolysis.

Case with Error

A 7-month-old male, previously healthy, develops an upper respira- tory infection and presents in the emergency department with fever, lethargy, and loose stools. He is minimally responsive with marked pallor and tachycardia. A chest X-ray shows complete opacifi cation of the right middle lung lobe. Antibiotics are initiated. Stool culture results are negative for the presence of pathogens (including Esche- richia coli O157:H7). Subsequent blood cultures grow S. pneumoniae . He develops anuria on the evening of admission with a blood urea nitrogen value of 70 mg/dL and a creatinine of 2.6 mg/dL (both mark- edly elevated). Hemolytic uremic syndrome (HUS) is diagnosed based on the fi ndings of microangiopathic hemolytic anemia (MAHA), thrombocytopenia, and renal failure. Peritoneal dialysis is started. The patient’s hemoglobin level is 5.7 g/dL. So the clinical team decides to transfuse RBCs. The patient’s CBC fails to show an appropriate incre- ment in hemoglobin following this transfusion.

EEichbaum_87963_PTR_CH03_10-01-12_051-136.inddichbaum_87963_PTR_CH03_10-01-12_051-136.indd 8282 111/10/121/10/12 1:001:00 PMPM Crypt Antigen Activation, Adverse Consequences 83

Clinical Pitfall

Failure to consider cryptic antigen exposure and to understand the associated transfusion risks. Multiple microorganisms contain the enzyme neuraminidase as a virulence factor. This enzyme can uncover normally silent antigens on RBCs, platelets, glomerular capillary endothelium, and renal tubular endothelium. This uncovering of cryptic antigens may be clinically signifi cant in transfused patients because almost all adult blood prod- ucts contain anti-T antibodies. Shiga-like toxin-producing Escherichia coli is the most frequent cause of HUS in children. However, HUS can also be seen as a complication of invasive pneumococcal infection. Antigen–antibody interactions cause damage to the surface of cells expressing this antigen, resulting in hemolysis of RBCs, destruction of platelets, and renal microangiopathy, leading to the clinical mani- festations of HUS.

Explanation and Consequences

There is a subset of T antigens (Th, Tk, Tx), which can be distinguished by their reactivity to various plant lectins. Although most frequently associated with necrotizing enterocolitis (NEC), a series of cases of T antigen activation in children with pneumococcus has also been described. Once the clinical decision to transfuse has been made, wash- ing of blood products should be considered in order to reduce the anti- body titer of anti-T antibodies. Failure to do so may contribute to further immune-mediated hemolysis. In pediatric populations, antibody titers to the T antigen can be detected in most infants by 6 months of age. The involvement of antibodies to the T antigen in pneumococcal HUS is controversial. HUS is an unpredictable potential consequence of Streptococcus pneumoniae infection, and documented cases have shown increased morbidity and mortality. The decision to modify the blood product prior to transfusion can be problematic. Washing blood products can lead to a signifi cant loss of cells, which may in turn increase the clinical need for additional transfusions. Washing may elicit cellular activation, and the delay entailed in transfusing modifi ed blood products may compromise care.

EEichbaum_87963_PTR_CH03_10-01-12_051-136.inddichbaum_87963_PTR_CH03_10-01-12_051-136.indd 8383 111/10/121/10/12 1:001:00 PMPM 84 3. Errors Involving Specifi c Clinical Scenarios

STANDARDS OF CARE

Washing of blood products may be necessary to reduce the risk of passive anti-T transmission. Consideration of crypt antigen exposure in patients with infections should be factored into decisions.

RECOMMENDED READING

Copelovitch L, Kaplan BS. Streptococcus pneumonia—associated hemolytic uremic syndrome: classifi cation and the emergence of sero- type 19A. Pediatrics 2010;125:e174–e182. Crookston KP, Reiner AP, Cooper LJ, et al. RBC T activation and hemolysis: implications for pediatric transfusion management. Trans- fusion 2000;40:801–812. Eder AF, Manno, CS. Does red-cell T activation matter? Br. J. Hematol . 2001;114:25–30.

Eichbaum_87963_PTR_CH03_10-01-12_051-136.indd 84 11/10/12 1:00 PM Failure to Recognize Specifi c Risk Factors 85

FAILURE TO RECOGNIZE SPECIFIC RISK FACTORS THAT ARE ASSOCIATED WITH ADVERSE REACTIONS IN BLOOD DONORS

OVERVIEW

Healthy persons can donate up to 500 mL of blood with only a transient impact on their circulatory system (the FDA limits donation to 10.5 mL/kg). Nonetheless, syncopal episodes do occur among blood donors. The incidence ranges between 2% and 5% of all donors, and is especially common in fi rst time donors mostly due to anxiety and vasovagal reactions.

Case with Error

A 16-year-old healthy female, 5 foot 4 inches in height, with a weight of 110 lbs volunteers to donate blood at her high school blood drive. She obtains signed parental permission. After donating blood, she states that she feels fi ne and declines the juice and cookies offered to her. She lingers in the postdonation area for 15 minutes. As she gets up to join her friends, she suddenly feels dizzy and collapses, hitting her head on a table that knocks her unconscious. Staff mem- bers immediately come to her assistance, and she is taken to the local emergency department for evaluation of her head injury that reveals a small hematoma.

Clinical Pitfall

Failure to recognize risk factors for syncopal reactions in blood donors. Many states allow 16- and 17-year-old teenagers to participate in blood donation. As a consequence, most high schools now host blood drives. Young age, female gender, and fi rst time donation status are independent risk factors for donation-related complications. Strate- gies have been developed to mitigate these risks.

EEichbaum_87963_PTR_CH03_10-01-12_051-136.inddichbaum_87963_PTR_CH03_10-01-12_051-136.indd 8585 111/10/121/10/12 1:001:00 PMPM 86 3. Errors Involving Specifi c Clinical Scenarios

Explanation and Consequences

Although uncomfortable for the donor, the clinical effects of dona- tion (eg, presyncope, hematoma) are usually inconsequential. However, the risks of injury are greater with younger donors. Syncope- related injury is about 3 times more likely in 16- and 17-year-old compared with 18- and 19-year-old donors (10,000/6ف) donors injuries/10,000 donations) and is 14 times more likely in this 2ف) young age group compared with donors aged 20 years, or older. Almost half of all injuries recorded by the donor collection center staff in nine American Red Cross regions occurred in 16- and 17-year-old donors. Many were severe enough to require outside medical care (eg, concussion, laceration requiring stitches, dental injuries, broken jaw). Even minor complications are known to reduce the likelihood of a donor returning for donation.

STANDARDS OF CARE

Young age, gender, fi rst time donation, and anxiety are known risk factors for postdonation syncope. Donors who meet these criteria should be carefully monitored.

RECOMMENDED READING

Eder AF, Hillyer CD, Dy BA, et al. Adverse reactions to allogeneic whole blood donation by 16- and 17-year-olds. JAMA 2008;299: 2279–2286. Wieling W, France CR, van Dijk N, et al. Physiologic strategies to prevent fainting responses during or after whole blood donation. Transfusion 2011;51:2727–2738.

EEichbaum_87963_PTR_CH03_10-01-12_051-136.inddichbaum_87963_PTR_CH03_10-01-12_051-136.indd 8686 111/10/121/10/12 1:001:00 PMPM Misinterpretation of the Direct Antiglobulin Test (DAT) 87

MISINTERPRETATION OF THE DIRECT ANTIGLOBULIN TEST (DAT)

OVERVIEW

The DAT, also known as the direct Coombs’ test, indicates whether antibody(IgG) and/or complement (usually C3d) is bound to the patient’s RBCs. A positive DAT can have various interpretations and requires appropriate additional testing. The DAT should be interpreted in the context of the patient’s medi- cal history and underlying condition. Thus the clinical signifi - cance of a positive DAT should be determined by taking account of both clinical and laboratory information.

A positive DAT can be succeeded by an elution assay in which antibody adhering to the patient’s RBCs is removed from these cells and evaluated, with the goal of determining whether alloantibodies not detected in the patient’s plasma are adhering to the RBCs. The eluted antibody can either be identical to that found in the plasma or may represent an additional underlying alloantibody not detected in the plasma. How such antibody elutions are performed also depends on whether the patient has been recently transfused. A positive DAT is often misinterpreted as being indicative of RBC hemolysis, but cau- tion should be exercised as there are other causes of a positive DAT.

Case with Error

An oncologist concerned about the worsening anemia of his patient with chronic lymphocytic leukemia (CLL) considers the possibility that the patient may have developed an autoimmune hemolytic anemia (AIHA). He is aware that about 11% of CLL patients develop an AIHA and that the signifi cance of this association remains unknown. The oncologist orders a DAT. The result is positive at a moderate strength of 2+. The antibody adhering to the RBCs is an IgG but, on elution, reacts as an autoantibody that is found to be also present in the plasma

EEichbaum_87963_PTR_CH03_10-01-12_051-136.inddichbaum_87963_PTR_CH03_10-01-12_051-136.indd 8787 111/10/121/10/12 1:001:00 PMPM 88 3. Errors Involving Specifi c Clinical Scenarios

and lacks specifi city. The oncologist concludes that his patient’s ane- mia is a result of RBC hemolysis possibly due to an underlying AIHA. To confi rm this suspicion, the oncologist orders a lactate dehydroge- nase (LDH) test, an indirect bilirubin test, and a haptoglobin level. He is surprised that the results of these tests are within normal range. He decides to examine a blood smear and sees no evidence of immune RBC hemolysis.

Clinical Pitfall

Failure to understand multiple causes of a positive DAT. A positive DAT is not necessarily indicative of hemolysis but may be due to many other factors including drug interactions and non- specifi c adherence of protein to the RBCs.

Explanation and Consequences

The reason for this patient’s positive DAT is not clear, and is likely due to nonspecifi c binding of antibody to the RBCs. It does not appear to be indicative of hemolysis or of an AIHA. If hemolysis is suspected in a patient, other tests for assessing hemolysis, such as the LDH, bili- rubin, and haptoglobin tests, should be performed before the DAT is ordered (even though these tests are also not highly specifi c). About 80% of hospitalized patients who have a positive DAT will have a nonreactive eluate because there is nonspecifi c and clinically insignifi cant adherence of protein (including antibodies) to the RBCs. The eluate of a positive DAT is considered “nonreactive” when it does not demonstrate any specifi c binding to a panel of test RBCs expressing clinically signifi cant antigens.

STANDARDS OF CARE

A positive DAT should be interpreted in the context of the underly- ing disease and results of the associated laboratory tests. It may be clinically insignifi cant or may be associated with an underlying disease such as AIHA, drug-induced anemia, or HDFN.

EEichbaum_87963_PTR_CH03_10-01-12_051-136.inddichbaum_87963_PTR_CH03_10-01-12_051-136.indd 8888 111/10/121/10/12 1:001:00 PMPM Recommended Reading 89

In the setting of antibody identifi cation, performing an eluate in which antibodies are removed from the patient’s RBCs for further testing, may sometimes be useful as it very occasionaly reveals the presence of an alloantibody not present in the plasma. In the setting of a transfusion reaction, a DAT should be performed on posttransfusion as well as on the pretransfusion samples of the blood, in order to determine whether the strength of the reaction has increased. Especially if the DAT has increased in strength, an eluate should be performed to determine the specifi city of the anti- body coating the RBCs.

RECOMMENDED READING

Das SS, Chaudhary R, Khetan D. A comparison of conventional tube test and gel technique in evaluation of direct antiglobulin test. Hema- tology 2007;12:175–178.

EEichbaum_87963_PTR_CH03_10-01-12_051-136.inddichbaum_87963_PTR_CH03_10-01-12_051-136.indd 8989 111/10/121/10/12 1:001:00 PMPM 90 3. Errors Involving Specifi c Clinical Scenarios

WARFARIN REVERSAL—INAPPROPRIATE USE OF FRESH FROZEN PLASMA

OVERVIEW

Warfarin (Coumadin) exerts its anticoagulant effect by inhibiting the hepatic synthesis of the clotting Factors II, VII, IX, and X (as well as protein C and protein S). All these factors depend on the active form of vitamin K for synthesis. Vitamin K administration usually corrects the anticoagulant effect of warfarin, but the time required for correction depends on the dose of vitamin K and on the route of vitamin K adminis- tration (IV is fastest; oral is slowest). In nonemergent situations when the bleeding is not signifi cant, withholding the warfarin and administering vitamin K are suffi cient. With signifi cant bleeding or if emergent surgery is required, plasma or prothro mbin complex concentrates (PCCs) are also usually administered. Four-factor PCC preparations that include factor VII can more immediately reverse the warfarin effect, but these PCCs are as yet not available in the United States, despite being well tolerated in patients. PCCs can also be administered more quickly and in smaller volumes and on that basis are advantageous for emergent reversal of warfarin.

In anticoagulant-associated intracerebral hemorrhage (AAICH) that occurs with signifi cant bleeding, vitamin K should be immedi- ately administered to the patient. It is usually given together with fresh frozen plasma (FFP) or PCCs because vitamin K depends on hepatic synthesis of new coagulation factors and takes hours to induce coagu- lation factor synthesis. The intravenous route for vitamin K may be preferred as this works faster than oral or subcutaneous routes. Although FFP may be readily available and contains all the coagulation factors, administration is relatively slow and replacement of factors may still be insuffi cient. Administering FFP alone is not

EEichbaum_87963_PTR_CH03_10-01-12_051-136.inddichbaum_87963_PTR_CH03_10-01-12_051-136.indd 9090 111/10/121/10/12 1:001:00 PMPM Warfarin Reversal—Inappropriate Use of Fresh Frozen Plasma 91

appropriate in AAICH with concomitant bleeding, not only because administration is relatively slow but also because the larger volumes required to achieve the desired effect may not be tolerated by the patient. Recombinant factor VIIa (rFVIIa) promptly reverses a prolonged INR in warfarin users, but it does not substitute for all four of the decreased clotting factors; it is generally not recommended for war- farin reversal.

Case with Error

A 65-year-female on warfarin with a diagnosis of atrial fi brillation and primary hypertension has a sudden onset of slurred speech and confusion. She is brought by ambulance to the closest hospital for assessment of a possible hemorrhagic stroke. Her INR is shown to be 6 (high), and her blood pressure is 148/88. She wears a medic-alert bracelet warning that she is on warfarin therapy. To reverse the anticoagulant effect of the warfarin, the resident- on-call recommends immediately transfusing 2 units of FFP at a fast rate in order to rapidly reverse the anticoagulant effect of her war- farin. The FFP is transfused, and she is transferred to a tertiary care hospital for radiologic assessment and further evaluation. On arrival 1 hour later, her condition has signifi cantly worsened. Her INR remains unchanged from its original value, and her blood pressure is 160/90.

Clinical Pitfall

Failure to recognize that warfarin reversal for severe bleeding requires treatment with vitamin K as well as with fresh frozen plasma. When signifi cant bleeding occurs in AAICH, vitamin K should be administered immediately by slow intravenous infusion, with a pos- sible repeat dose at 12 hours. Vitamin K should be supplemented with FFP (initially emergency release AB plasma), “three-factor” PCCs (there are small amounts of factor VII in these), or four-factor PCCs, if available. Vitamin K depends on hepatic synthesis of new coagula- tion factors and takes several hours to take effect, whereas FFP and

EEichbaum_87963_PTR_CH03_10-01-12_051-136.inddichbaum_87963_PTR_CH03_10-01-12_051-136.indd 9191 111/10/121/10/12 1:001:00 PMPM 92 3. Errors Involving Specifi c Clinical Scenarios

PCCs act more quickly to reverse the warfarin effect. In hypertension, administering large volumes of FFP to a stroke patient at an accel- erated rate can lead to volume overload and aggravate the patient’s condition.

Explanation and Consequences

Patients with atrial fi brillation are commonly treated with warfa- rin anticoagulant therapy. This increases the risk for AAICH. Such patients on anticoagulant therapy should have their INR regularly monitored to keep it in the therapeutic range. The patient in this case was not given vitamin K, but only FFP. This may also have aggravated her hypertension, leading to worse outcomes.

STANDARDS OF CARE

Several differing international guidelines are available for warfarin reversal in the context of intracerebral hemorrhage, most of which recommend intravenous vitamin K supplemented with either FFP or PCCs, as vitamin K alone is not considered adequate. In settings that do not involve signifi cant bleeding, the 2012 CHEST guidelines recommend achieving anticoagulation broadly as follows: i. Patients taking vitamin K antagonists and with INRs between 4.5 and 10, but with no evidence of bleeding, do not need to routinely take vitamin K. ii. Patients with INRs greater than 10, with no evidence of bleed- ing, should take oral vitamin K. In cases of serious bleeding in patients with warfarin-associated ICH, recent guidelines recommend a vitamin K dose of 10 mg given intravenously at a slow infusion rate (over 30 minutes) with consideration of a repeat dose at 12 hours. Supplementation with either FFP, PCCs, or Factor VIIa is generally considered necessary for rapid correction of the INR, but evidence-based guidance and correlated outcomes for each of these therapies is lacking. The use of plasma in the setting of warfarin-induced ICH was recently given only a “weak” recommendation by the AABB

EEichbaum_87963_PTR_CH03_10-01-12_051-136.inddichbaum_87963_PTR_CH03_10-01-12_051-136.indd 9292 111/10/121/10/12 1:001:00 PMPM Recommended Reading 93

(American Association of Blood Banks). This low grading may be due to delayed administration and inadequate dosing of the product. A minimum of 15 mL/kg and ≤30 mL/kg has been recommended, unless the patient cannot tolerate the volume. Four-factor PCCs are effective for patients with AAICH, whereas the benefi t of three-factor PCC therapy has not been fully estab- lished and is therefore as yet not approved for AAICH. Recent randomized clinical trials showed that four-factor PCCs had a simi- lar hemostatic effi cacy to plasma at 24 hours in patients requiring warfarin reversal and were actually superior to plasma in achiev- ing target international normalized ratio (INR) correction within 30 minutes after infusion. The American Heart Association and American Stroke Associa- tion do not recommend the routine use of Factor VIIa for warfarin reversal for AAICH.

RECOMMENDED READING

Chowdary P, Saayman AG, Paulus U, et al. Effi cacy of standard dosing and 30 mL/kg fresh frozen plasma in correcting laboratory parameters of haemostasis in critically ill patients. Br. J. Haematol . 2004;125:69–73. Flaherty ML. Anticoagulant-associated intracerebral hemorrhage. Semin. Neurol . 2010;30:565–572. Whitlock RP, Sun JC, Fremes SE, et al. Antithrombotic and throm- bolytic therapy for valvular disease: antithrombotic therapy and prevention of thrombosis, 9th ed: American College of chest physi- cians evidence-based clinical practice guidelines. Chest 2012;141 (2 suppl):e576S–e600S.

Eichbaum_87963_PTR_CH03_10-01-12_051-136.indd 93 11/10/12 1:00 PM 94 3. Errors Involving Specifi c Clinical Scenarios

ANTI-KELL ALLOANTIBODIES—MISSED DIAGNOSIS OF HEMOLYTIC DISEASE OF THE FETUS AND NEWBORN (HDFN)

OVERVIEW

HDFN occurs mostly through maternal alloimmunization against an antigen present on fetal RBCs (more rarely, it can also occur through prior maternal transfusion). The maternal antibody crosses the placenta and coats the fetal RBCs, which are then removed from circulation by splenic macrophages, resulting in fetal anemia. The titer of the maternal alloantibody can be helpful in monitoring the potential for HDFN. Amniotic fl uid bilirubin levels can also serve as an indicator of active hemolysis. Both of these results should, however, be interpreted with caution when anti-Kell alloantibodies are implicated in the disorder. Newer approaches to diagnosing HDFN use Doppler measurements of peak systolic blood fl ow velocity in the fetal middle cerebral artery (MCA).

Case with Error

A 32-year-old pregnant female is discovered, during a pregnancy screen at 18 weeks, to have developed an anti-Kell alloantibody. Although the titer of the antibody is only 1 to 4, percutaneous umbili- cal cord sampling (PUBS) reveals that the fetal hemoglobin has recently decreased from 12 to 9.8 g/dL. Her doctor is concerned about her potential for developing HDFN and decides to request from the laboratory serial titers of the anti-Kell antibody, as well as amniotic fl uid bilirubin levels. Over the ensuing 4 weeks, the doctor feels reas- sured because her antibody titer does not increase, and her amniotic fl uid bilirubin levels have also not appreciably risen. The doctor is further persuaded against a diagnosis of HDFN by a panel of hemo- lysis test results and a blood smear that show no evidence of RBC hemolysis. Nonetheless, the doctor remains perplexed that the fetal hemoglobin has now further decreased to 8.8 g/dL.

EEichbaum_87963_PTR_CH03_10-01-12_051-136.inddichbaum_87963_PTR_CH03_10-01-12_051-136.indd 9494 111/10/121/10/12 1:001:00 PMPM Standards of Care 95

Clinical Pitfall

Failure to understand the role in HDFN of maternal antibodies that suppress erythropoiesis and cause anemia without causing hemolysis. HDFN is frequently assessed clinically by following the maternal alloantibody titer and amniotic fl uid bilirubin levels. Although com- monly utilized, these indices are not accurate predictors of the sever- ity of the HDFN. This applies in particular to HDFN caused by the anti-Kell blood group antibodies. As antigens of the Kell blood group system are expressed very early during erythropoiesis, antibodies to these antigens can destroy RBC precursor cells, and thereby clinically suppress erythropoiesis. This may result in a clinical picture of anemia without observable hemolysis or elevated bilirubin. The doctor may erroneously rule against a diagnosis of HDFN and seek alternate rea- sons for the patient’s anemia.

Explanation and Consequences

The increasing anemia of the fetus may be due to the presence of the maternal anti-Kell antibody binding the Kell antigens and causing suppression of erythropoiesis without evident hemolysis and without an elevated bilirubin. Anti-Kell titers as low as 1 to 4 have been associ- ated with HDFN. The low anti-Kell antibody titer, low-bilirubin lev- els, and ostensible lack of RBC hemolysis led the doctor erroneously to rule out the possibility of HDFN. For anti-D-antibodies, consider titers of 1 to 32 as critical for the development of HDFN, but interpret antibody titers more cautiously for other antibody specifi cities associ- ated with HDFN.

STANDARDS OF CARE

After the risk of HDFN has been established through prenatal and neonatal testing for maternal alloimmunization as well as phenotyping of the biological father’s RBCs (and fetal phenotyping or genotyping to determine the antigen status of the fetus), mater- nal antibody titers to the implicated antigen should be measured every 2 to 4 weeks, beginning at 18 weeks gestational age.

EEichbaum_87963_PTR_CH03_10-01-12_051-136.inddichbaum_87963_PTR_CH03_10-01-12_051-136.indd 9595 111/10/121/10/12 1:001:00 PMPM 96 3. Errors Involving Specifi c Clinical Scenarios

Amniotic fl uid bilirubin levels can be helpful for monitoring hemo- lysis but should be interpreted with caution when anti-Kell group antibodies are implicated, as fetal anemia may be driven by sup- pression of erythropoiesis rather than by active RBC hemolysis.

RECOMMENDED READING

Eder AF. Update on HDFN: new information on long-standing contro- versies. Immunohematology 2006;22:188–195. Koelewijn JM, Vrijkotte TG, van der Schoot CE, et al. Effect of screening for red cell antibodies, other than anti-D, to detect the molytic disease of the fetus and newborn: a population study in the Netherlands. Transfusion 2008;48:941–952.

EEichbaum_87963_PTR_CH03_10-01-12_051-136.inddichbaum_87963_PTR_CH03_10-01-12_051-136.indd 9696 111/10/121/10/12 1:001:00 PMPM IgA Defi ciency—Misinterpretations and Assumptions 97

IgA DEFICIENCY—MISINTERPRETATIONS AND ASSUMPTIONS

OVERVIEW

IgA defi ciency is relatively common: about 1 in 900 in donors of European descent in the United States and United Kingdom, but about 1:500 in Finland; and only about 1:18,500 in Japan. However only 20% to 30% of IgA defi cient individuals will form an anti-IgA antibody (80% in individuals with autoimmune disease). If IgA defi ciency is suspected, an IgA level should be ordered. If IgA is severely defi cient (<0.05 mg/ dL), the presence of an anti-IgA antibody should be determined. A search for IgA defi cient donors should be initiated, as fi nding such donors can take time. Some tests for IgA involve nephelom- etry, which is usually not suffi ciently sensitive to identify the level of severity of defi ciency associated with the production of an anti-IgA antibody.

Anti-IgA may be present as a natural autoantibody detected in normal human sera, and/or as an alloantibody. It can be stimulated by expo- sure to IgA, but is also formed regardless of previous transfusion or pregnancy. Anti-IgA is usually of the IgG class, but it can also be IgE or IgM and can be of variable specifi city. Not all individuals with IgA defi ciency and anti-IgA antibody will have anaphylactic reactions. Conversely, it is also important to note that anaphylactic reactions have been noted in IgA defi cient patients who have no detectable anti- IgA antibody. Severe allergic/anaphylactic reactions require immediate dis- continuation of the transfusion, with maintenance of open intrave- nous line access, the availability of oxygen (sometimes requiring intubation), and usually treatment with epinephrine (as well as pos- sibly vasopressors, steroids, and/or H1 and H2 receptor antagonists). Common symptoms seen in anaphylaxis include dyspnea, laryngeal edema, circulatory collapse, and hypotension. Fever is usually absent.

EEichbaum_87963_PTR_CH03_10-01-12_051-136.inddichbaum_87963_PTR_CH03_10-01-12_051-136.indd 9797 111/10/121/10/12 1:001:00 PMPM 98 3. Errors Involving Specifi c Clinical Scenarios

Case with Error

A 54-year-old man with systemic lupus erythematosus and hemolytic anemia is being transfused with RBCs. Within 5 minutes after ini- tiation of transfusion, his blood pressure falls from 120/80 to 90/63. He becomes severely dyspneic and is given oxygen and epinephrine before he stabilizes. The physician suspects IgA-associated anaphy- laxis and orders an IgA level. The preliminary result shows a value less than 20 mg/dL. To determine the actual level of the defi ciency, a sample is sent to an outside laboratory for testing. The test indicates a severe defi ciency of less than 0.05 mg/dL. The physician is aware that at this level of defi ciency, the patient could likely have developed an anti-IgA antibody that may account for his anaphylaxis. He orders a test for the presence of such an anti-IgA antibody and initiates a search for IgA defi cient donors. The test result is, however, negative for the presence of an anti-IgA antibody. The physician assumes that the patient’s anaphylactic reaction cannot have been due to IgA defi ciency, but rather to some other aller- gen in the RBC unit previously transfused. He terminates the search for IgA defi cient donors. He proceeds to transfuse the patient with another unit of RBCs. Eight minutes into the transfusion, the patient again suffers a precipitous decrease in blood pressures, becomes severely dyspneic and has to be emergently intubated.

Clinical Pitfall

Failure to appropriately follow up a presumed diagnosis of IgA defi ciency. While severe IgA defi ciency (<0.05 mg/dL) frequently results in the development of anti-IgA antibodies, this is not always the case. Anaphylactic reactions may be seen in IgA defi cient patients without detectable anti-IgA antibody. The physician has failed to understand that severely IgA defi cient individuals who lack an anti-IgA antibody may nonetheless still experience an anaphylactic reaction due to their IgA defi ciency.

EEichbaum_87963_PTR_CH03_10-01-12_051-136.inddichbaum_87963_PTR_CH03_10-01-12_051-136.indd 9898 111/10/121/10/12 1:001:00 PMPM Recommended Reading 99

Explanation and Consequences

The assumption was made that the patient’s anaphylaxis was not due to IgA defi ciency but likely to some other allergen in the RBC unit previously transfused. The physician, therefore, assumed it safe to proceed with transfusion of another unit of RBCs. While the trig- ger for anaphylaxis in this setting is not clear, a trial of washed prod- ucts or IgA-defi cient products would have been a safer intermediary step. Also, in this case, premedication with steroids or H1 receptor antagonists may have mitigated the severity of the second anaphylactic reaction.

STANDARDS OF CARE

When an anaphylactic transfusion reaction occurs and IgA defi ciency is suspected: If IgA defi ciency is less than 0.05 mg/dL, check for the presence IgA antibody. Initiate search for blood products from IgA defi cient patients. If IgA defi cient products are not available, give washed RBCs and washed platelet products. Plasma and cryoprecipitate for IgA defi cient patients must be col- lected from IgA defi cient donors. Premedication with steroids and antihistamines may be benefi cial. Do not assume that the absence of an anti-IgA antibody precludes anaphylaxis due to IgA defi ciency, or conversely that individuals with IgA defi ciency and an anti-IgA antibody will necessarily have anaphylactic reactions.

RECOMMENDED READING

Sandler SG, Zantek ND. Review: IgA anaphylactic transfusion reac- tions. Part II. Clinical diagnosis and bedside management. Immunohe- matology 2004;20:234–238.

EEichbaum_87963_PTR_CH03_10-01-12_051-136.inddichbaum_87963_PTR_CH03_10-01-12_051-136.indd 9999 111/10/121/10/12 1:001:00 PMPM 100 3. Errors Involving Specifi c Clinical Scenarios

MISINTERPRETING THE CAUSE OF HYPOTENSION DURING TRANSFUSION

OVERVIEW

The differential diagnosis for hypotension associated with transfusion includes acute hemolytic reactions, anaphy- laxis, and bacterial contamination with sepsis. Hypotension asso- ciated with sepsis is usually accompanied with a fever greater than 38.5°C, rigors, and vomiting during transfusion often with the added complications of shock, oliguria, and disseminated intravascular coagulation. As with all suspected transfusion reac- tions, the fi rst step is to stop the transfusion while maintaining an open intravenous access line. Severe allergic/anaphylactic reac- tions require immediate discontinuation of the transfusion, with appropriate supportive treatment to maintain oxygenation and blood pressure. However, hypotension may also be unrelated to the transfusion and instead be due to the patient’s underlying condition.

By defi nition, a primary hypotensive transfusion reaction occurs as an isolated decrease in systolic and/or diastolic blood pressure of 30 mm Hg or more, within minutes of starting transfusion and resolves with supportive treatment soon after the transfusion is stopped. Such hypo- tensive reactions may be due to blood passing through charged fi lters (usually negatively charged) in patients who are taking angiotensin-con- verting enzyme inhibitors (ACE-i). In such patients on ACE-i receiving blood through negatively charged fi lters, the generation of bradykinin and/or its metabolite des-Arg -bradykinin appear to be implicated.

Case with Error

A 53-year-old man with Goodpasture’s syndrome and hypertensive kid- ney disease is undergoing therapeutic plasma exchange (TPE) to decrease his elevated titer of antiglomerular basement membrane antibody. He

EEichbaum_87963_PTR_CH03_10-01-12_051-136.inddichbaum_87963_PTR_CH03_10-01-12_051-136.indd 100100 111/10/121/10/12 1:001:00 PMPM Misinterpreting the Cause of Hypotension during Transfusion 101

has been admitted to the hospital in an unstable condition with hemopty- sis and blood pressure fl uctuations between 98/68 and 148/90. Because his hematocrit is low, he is transfused with RBCs immediately prior to starting TPE. Within 6 minutes of starting TPE, he experiences fl ushing and his blood pressure falls from 116/73 to 80/41 mmHg. TPE is immediately discontinued. The patient is infused with bolus of normal saline, and his blood pressure quickly returns to nor- mal. A transfusion reaction report is fi led. The physician believes that the patient’s decrease in blood pressure that occurred during TPE may have been associated with his underlying medical condition, rather than being related to the transfusion, as his blood pressure had already been fl uctuating prior to transfusion. Nonetheless, to be safe, the physician also considers the possibility of sepsis due to a bacterially contaminated unit of RBCs transfused immediately prior to TPE, and therefore places the patient on broad-spectrum antibiotics. By the next day, the patient appears stable and has not had any fevers, so the phy- sician decides to attempt TPE again. Within a few minutes of starting TPE, the patient’s blood pressure again falls precipitously.

Clinical Pitfall

Failure to fully explore the potential causes for hypotension in a patient being transfused. Changes in vital signs and symptoms occurring during a blood transfusion may be a result of the transfusion, or may be causally unre- lated and coincidental to the transfusion. In this case, the physician also considered bacterial contamination of the RBC unit as a possibil- ity, but decided to proceed with TPE before receiving back the culture results. Other possible causes for the hypotension were not adequately explored. Knowing a patient’s current medications is important in analyzing a transfusion reaction. This patient with hypertensive renal disease was found to be on captopril, an ACE-inhibitor.

Explanation and Consequences

Patients on ACE-i are at risk of experiencing hypotensive transfu- sion reactions when receiving blood products that are exposed to

Eichbaum_87963_PTR_CH03_10-01-12_051-136.indd 101 11/10/12 1:00 PM 102 3. Errors Involving Specifi c Clinical Scenarios

charged surfaces, as occurs during therapeutic apheresis procedures. These hypotensive reactions are thought to be mediated by bradykinin and des-Arg -bradykinin, two vasoactive kinins generated when plasma comes into contact with a negatively charged surface. Patients receiving ACE-i should have these medications discontinued at least 24 hours or more prior to starting apheresis procedures. The physi- cian in this case was unaware that the patient was taking an ACE-i and therefore did not discontinue the medication prior to starting TPE.

STANDARDS OF CARE

Hypotension in the setting of a transfusion should be carefully evaluated and not assumed to be due to the patient’s underlying condition. Each option in the differential diagnosis should be con- sidered in the context of the patient’s past medical history, clinical scenario, and the laboratory results. Knowing a patient’s medications is important in analyzing a sus- pected transfusion reaction. In patients undergoing TPE, ACE-i can interact with charged membrane surfaces in the apheresis instru- ment. These drugs should therefore be discontinued prior to start- ing TPE (at least 24 hours before starting apheresis in patients on short-acting ACE-i such as captopril and quinapril; or several days before starting apheresis in the case of longer-acting ACE inhibi- tors such as enalapril and lisinopril). If the hypotension is accompanied by signifi cant fever, bacterial contamination should be ruled out by culturing the blood product.

RECOMMENDED READING

Sandler SG, Zantek ND. Review: IgA anaphylactic transfusion reactions. Part II. Clinical diagnosis and bedside management. Immunohematology 2004;20:234–238. Squires JE. Risks of transfusion. South. Med. J. 2011;104(11):762–769.

EEichbaum_87963_PTR_CH03_10-01-12_051-136.inddichbaum_87963_PTR_CH03_10-01-12_051-136.indd 102102 111/10/121/10/12 1:001:00 PMPM Inappropriate Application of Premedication For Transfusion 103

INAPPROPRIATE APPLICATION OF PREMEDICATION FOR TRANSFUSION

OVERVIEW

Allergic and febrile nonhemolytic transfusion reactions (FNHTR) are the most commonly reported transfusion reactions. They are generally mild and do not cause signifi cant morbidity. FNHTRs are isolated febrile reactions that occur during transfusions that are not associated with any hemolysis. Mild allergic reactions can be treated by temporarily stopping the transfusion and administering diphenhydramine, and resum- ing transfusion if the patient is stable. Such isolated reactions generally do not necessitate routine premedication for subse- quent transfusions.

Acetaminophen and diphenhydramine are generally considered effective therapies for fever and , respectively. In the setting of transfusion, however, their routine usage is controversial since their effi cacy has not been proven, and because fever and allergic reactions from transfusion are often temporally limited and self-resolving. Nonetheless, it is esti- mated that physicians routinely prescribe these medications in up to 80% of blood product transfusions in order to avert the possibility of reac- tions. In some institutions all transfusions receive premedication. The tendency to premedicate is usually greater if the patient has previously suffered an allergic or febrile reaction while undergoing transfusion.The medical record is not always reviewed to determine if there was ever an indication for such medication. Patients who have experienced an isolated allergic or febrile reaction associated with transfusion are most often no more likely to have a repeat reaction than those who have never experienced a reaction. Acetaminophen and diphenhydramine are both associated with potential toxicities and should not be routinely, and without good rea- son, administered to patients prior to transfusion. For severe trans- fusion reactions, more effective alternatives to these medications are also available.

EEichbaum_87963_PTR_CH03_10-01-12_051-136.inddichbaum_87963_PTR_CH03_10-01-12_051-136.indd 103103 111/10/121/10/12 1:001:00 PMPM 104 3. Errors Involving Specifi c Clinical Scenarios

Case with Error

A 69-year-old female patient with a platelet count of 10,000/μL sta- tus post peripheral blood stem cell transplant is being transfused with platelets when she develops, after receiving 80 mL of the product, a reaction of hives on her arms and parts of her abdomen. The nurse stops the transfusion and gives the patient 50 mg of diphenhydramine and 200 mg of acetaminophen. The nurse issues a report of a sus- pected transfusion reaction. The hives resolve after 30 minutes but rather than continuing with the transfusion, the nurse waits for the results of the transfusion reaction report from the transfusion service. The report appears in the medical record the next day. A decision is made to premedicate the patient with diphenhydramine and acetaminophen 30 minutes prior to platelet transfusion in order to preempt a similar reaction. Once again the patient develops hives. The nurse-practitioner decides the medication doses were insuffi cient and gains the physi- cian’s permission to administer 50 mg of intravenous diphenhydr- amine and another 325 mg of acetaminophen.The patient becomes restless and agitated, and the nurse calls the attending physician to consider giving the patient lorazepam to calm her.

Clinical Pitfall

Failure to recognize that universal routine premedication with diphen- hydramine and acetaminophen does not prevent or reduce the rate of allergic and/or FNHTR transfusion reactions. Premedicating patients with diphenhydramine and/or acetamino- phen before transfusion in order to avoid urticarial and/or transfusion- related fever reactions has been shown in several randomized controlled trials to be generally ineffective. Such reactions are usu- ally due to factors in the specifi c product transfused, or to an unde- termined interaction between product and host factors. Premedication is often performed routinely and repeatedly without consideration of the original rationale. It can lead to harmful , unnecessary delays in transfusion and to the needless withholding of essential blood products.

EEichbaum_87963_PTR_CH03_10-01-12_051-136.inddichbaum_87963_PTR_CH03_10-01-12_051-136.indd 104104 111/10/121/10/12 1:001:00 PMPM Recommended Reading 105

Explanation and Consequences

The patient’s reaction of hives to the transfusion of platelets prompted the nurse to delay this necessary transfusion. The nurse might instead have waited for the reaction to subside and resumed the transfusion at a slower rate. Premedicating the patient was ineffective in averting another reaction. Failure to recognize the limited effi cacy of premedi- cation prompted the nurse to increase the dose of medication when the patient experienced another reaction of hives. Diphenhydramine usually has anticholinergic effects, such as drowsiness, dry mouth and urinary retention but it can also cause restlessness and nervousness and has been associated with cardiotoxicity and arrhythmias.

STANDARDS OF CARE

Mild allergic reactions, such as hives, associated with transfusion can be treated by temporarily stopping the transfusion, maintaining open intravenous access and waiting for the symptoms to subside before resuming transfusion at a slower rate under close observa- tion and monitoring. Diphenhydramine (or other antihistamines) can be judiciously used to treat an urticarial reaction but should not be routinely given as a prophylactic premedication. For recurring and severe reactions, alternatives to premedication such as washed or plasma-reduced products may be helpful. For severe allergic reactions, alternative medications such as antihis- tamines other than diphenhydramine, H1 receptor antagonists and corticosteroids may be helpful, but their effi cacy has not been unequivocally established.

RECOMMENDED READING

Geiger TL, Howard SC. Acetaminophen and diphenhydramine pre- medication for allergic and febrile non-hemolytic transfusion reactions: good prophylaxis or bad practice? Transfus. Med. Rev . 2007;21:1–12.

EEichbaum_87963_PTR_CH03_10-01-12_051-136.inddichbaum_87963_PTR_CH03_10-01-12_051-136.indd 105105 111/10/121/10/12 1:001:00 PMPM 106 3. Errors Involving Specifi c Clinical Scenarios

INCOMPLETE EVALUATION OF PLATELET REFRACTORINESS

OVERVIEW

Thrombocytopenia is associated with a broad range of med- ical conditions. The causes of refractory thrombocytopenia (when the count does not increase despite transfusion of platelets) are both immune and nonimmune, and often multifactorial. Treat- ment of the underlying illness will frequently alleviate nonim- mune causes of refractoriness. Immune-mediated causes may require HLA-matched or crossmatched platelets.

To distinguish immune from nonimmune cases of refractory throm- bocytopenia, it is essential that the peripheral blood platelet count be performed accurately. For an increment in the platelet count to be accurate, it should be measured between 15 and 60 minutes immedi- ately following completion of the platelet transfusion. The increment in the count should be determined using the corrected count increment (CCI—see ‘Standards of Care’) that takes into account the patient’s size via a body surface area measurement. Obese patients with large blood volumes and patients with splenomegaly who have sequestra- tion of platelets from circulation may show peripheral blood platelet counts that are corrected by the calculation of the CCI. The CCI should be calculated following two consecutive platelet transfusions before making a determination of “platelet refractoriness.” Since the introduction and widespread implementation of uniform prestorage leukoreduction, the immune causes of platelet refractori- ness via alloimmunization to HLA Class 1 antigens have decreased. Platelets express HLA Class IA and IB antigens. Matched platelets can be selected either by HLA antigen based selection, or by cross- matching which does not require knowledge of HLA type. An unreliable platelet count increment following transfusion may lead to either the unnecessary transfusion of additional platelets to increase the count or to a premature determination of “refractoriness.”

EEichbaum_87963_PTR_CH03_10-01-12_051-136.inddichbaum_87963_PTR_CH03_10-01-12_051-136.indd 106106 111/10/121/10/12 1:001:00 PMPM Incomplete Evaluation of Platelet Refractoriness 107

Case with Error

A 66-year morbidly obese woman who has undergone stem cell trans- plant for acute myeloid leukemia is on broad spectrum antibiotics and has been spiking fevers for the past 3 days. Her platelet count is 10,500/ μL. The doctor orders a pool of fi ve random donor platelets in the morn- ing and asks the nurse to transfuse these platelets. That evening, per pro- tocol, a CBC is ordered. The doctor expects the platelet count to have increased up to 40,000/μL but fi nds it at only 25,000/μL. He considers the result a laboratory error and requests a repeat count on a fresh sam- ple. The count remains the same, so he decides to transfuse additional platelets. He checks the count in the evening, but again it has failed to increase. Frustrated, he orders a pool of random donor platelets. How- ever, this transfusion has to be stopped after only one half of the product has been transfused because the patient develops a hypertensive reaction. The doctor assumes that the patient has developed platelet refractoriness and decides to order HLA-matched platelets.

Clinical Pitfall

Failure to take a platelet count between 15 and 60 minutes following transfusion and to calculate the CCI before making a determination of platelet refractoriness. Taking a platelet count later than 60 minutes posttransfusion and/or reporting the count without calculating the CCI to account for body size can lead either to unnecessary additional transfusions or to a false determination of “platelet refractoriness.”

Explanation and Consequences

The combination of not measuring the platelet count within the appro- priate time interval, and failing to take into account the patient’s large body size by calculating the CCI, yielded a falsely low plate- let count. This led to additional unnecessary platelet transfusions that precipitated a transfusion reaction, without a defi nitive diagnosis of refractoriness being made.

EEichbaum_87963_PTR_CH03_10-01-12_051-136.inddichbaum_87963_PTR_CH03_10-01-12_051-136.indd 107107 111/10/121/10/12 1:001:00 PMPM 108 3. Errors Involving Specifi c Clinical Scenarios

STANDARDS OF CARE

Determine the platelet count within 15 to 60 minutes following transfusion of platelets. From the platelet count, calculate the CCI, taking into account the patient’s specifi c body surface area, using the formula: Body surface area (m2 ) × Platelet count increment (platelets/␮L) × 1011 CCI = Number of platelets transfused Perform two consecutive platelet counts and CCI calculations before making a determination of “platelet refractoriness.” Rule out nonimmune causes of refractoriness before ordering HLA- matched platelets to manage alloimmunization to HLA class I antigens.

RECOMMENDED READING

Dzik, S. How I do it: platelet support for refractory patients. Transfu- sion 2007;47:374–378. Slichter SJ, Davis K, Enright H, et al. Factors affecting post-transfusion platelet increments, platelet refractoriness, and platelet transfusion intervals in thrombocytopenic patients. Blood 2005;105:4106–4114.

Eichbaum_87963_PTR_CH03_10-01-12_051-136.indd 108 11/10/12 1:00 PM Thrombotic Thrombocytopenic Purpura (TTP)—Missed Diagnosis 109

THROMBOTIC THROMBOCYTOPENIC PURPURA (TTP)—MISSED DIAGNOSIS

OVERVIEW

Thrombotic thrombocytopenic purpura (TTP) is classi- cally diagnosed by the “pentad” of anemia, thrombocyto- penia, neurologic signs, fever, and renal failure. While the ADAMTS13 assay can yield a more rapid and potentially accu- rate measurement of the disease, its utility in acute management of TTP is disputable. The full pentad of symptoms is not always observed in patients presenting with TTP. While fever is present in 24% of patients, neurologic signs in 63% and renal abnor- malities in 60%, each of the conditions can also manifest in a variety of ways; and nonspecifi c symptoms such as nausea, weakness, and abdominal pain may further confuse the diagnosis. More importantly, the absence of fever, neurologic signs, or renal dysfunction does not preclude a diagnosis of TTP.

The diagnosis of TTP should be seriously considered in patients with thrombocytopenia and MAHA presenting in the absence of an underly- ing disease. MAHA is diagnosed in the setting of a hemolytic anemia by the simultaneous presence, on a peripheral blood smear, of schistocytes. The diagnosis of hemolytic anemia is further suggested by elevated val- ues for serum LDH, indirect bilirubin, and reticulocyte count. As MAHA and thrombocytopenia are not specifi c for TTP, other causes for these conditions should also be evaluated through addi- tional laboratory tests, such as the PT, PTT, a direct antiglobulin test (DAT), and liver function tests. In short, the diagnosis of TTP should not be too strictly based on the recognition of the classic pentad.

Case with Error

A 36-year-old woman in good health is brought to a hospital after experiencing an isolated seizure at home. No other symptoms are

EEichbaum_87963_PTR_CH03_10-01-12_051-136.inddichbaum_87963_PTR_CH03_10-01-12_051-136.indd 109109 111/10/121/10/12 1:001:00 PMPM 110 3. Errors Involving Specifi c Clinical Scenarios

evident and the resident orders a panel of laboratory tests, including a CBC. The patient’s PT and PTT are normal, but her platelets are low at 45,000/␮L, and her hematocrit is low at 26%. She has never been ane- mic. In anticipation of transfusing her, the doctor orders a blood type and screen, and DAT. Her antibody screen and DAT are both nega- tive. The doctor transfuses her and orders platelets. The next day he is puzzled to see that neither her hematocrit nor her platelet count have increased to expected levels but have, to the contrary, both decreased. She now also complains of a severe headache. The doctor examines a blood smear for evidence of hemolysis as a cause of her anemia and observes a signifi cant number of schis- high powered fi eld). He considers the possibility of TTP/8ف) tocytes and decides to check her renal function by ordering creatinine and bilirubin tests. Both are normal. She has no fever. The doctor is per- plexed that she does not exhibit the pentad of criteria for the diagnosis of TTP and decides to order the ADAMTS13 metalloproteinase test as a specifi c test for TTP. The ADAMTS13 test result is negative, and the doctor concludes that she does not have TTP. As her hematocrit has now decreased to 22% and her platelets have decreased to 10,000/μL, the doctor decides to transfuse more RBCs and platelets. Several hours after the platelet transfusion, the patient suffers another seizure.

Clinical Pitfall

Failure to correctly diagnose TTP. The diagnosis of TTP often requires only the dyad of unexplained thrombocytopenia and microangiopathic anemia rather than the full clas- sic pentad. Given the mortality rate of greater than 90% for untreated TTP, treatment using therapeutic plasma exchange (TPE) with FFP should be instituted emergently. TPE is the primary therapy for TTP, but therapy can also be emergently initiated by transfusing plasma products (FFP or cryoprecipitate reduced plasma). The value of the ADAMTS13 assay in the diagnosis of TTP is somewhat controversial. Prospective cohort studies have shown that fewer than half of patients with idiopathic TTP had severe ADAMTS13 defi ciency. In a patient with a high degree of suspicion for TTP, a nor- mal ADAMTS13 should not delay initiation of plasma exchange.

EEichbaum_87963_PTR_CH03_10-01-12_051-136.inddichbaum_87963_PTR_CH03_10-01-12_051-136.indd 110110 111/10/121/10/12 1:001:00 PMPM Recommended Reading 111

Explanation and Consequences

The ADAMTS13 metalloproteinase test should be used with caution, as a defi ciency of this enzyme may not always be present to fi rmly establish a diagnosis of TTP. The ADAMTS13 assay is neither neces- sary nor suffi cient for a diagnosis, and should not delay initiation of plasma exchange therapy in a patient with a high of index of suspicion for TTP.

STANDARDS OF CARE

The diagnosis of TTP can be made based on unexplained MAHA and thrombocytopenia and does not require the classic “pentad” of symptoms that includes fever, neurologic symptoms, and renal dysfunction. TPE is typically performed for one total plasma exchange daily until the platelet count is above 150,000/μL, and is then often con- tinued for two more days after this platelet count is achieved. Blood samples for laboratory tests such as ADAMTS13 should be collected before commencing plasma exchanges or transfusions, and caution should be used in the interpretation of ADAMTS13 values.

RECOMMENDED READING

Coppo P, Wolf M, Veyradier A, et al. Prognostic value of inhibitory anti-ADAMTS13 antibodies in adult-acquired thrombotic thrombocy- topenic purpura. Br. J. Haematol . 2006;132:66–74. George JN. Clinical practice. Thrombotic thrombocytopenic purpura. N. Engl. J. Med . 2006;354:1927–1935.

EEichbaum_87963_PTR_CH03_10-01-12_051-136.inddichbaum_87963_PTR_CH03_10-01-12_051-136.indd 111111 111/10/121/10/12 1:001:00 PMPM 112 3. Errors Involving Specifi c Clinical Scenarios

TRANSFUSION-RELATED ACUTE LUNG INJURY (TRALI)—FAILURE TO DIAGNOSE

OVERVIEW

The diagnosis of TRALI is based on a set of consensus criteria that involves both clinical and radiologic fi ndings. These criteria serve as a guide only, and a high level of suspi- cion should be maintained for diagnosing this disease given its relatively high (transfusion-related) mortality. Respiratory distress in the absence of left atrial hypertension (circulatory overload) occurring within 6 hours of transfusion should alert the physician to the possibility of TRALI. Patients with TRALI also do not, by the consensus criteria, have pretransfusion lung injury. However, mild and atypical cases presenting with symp- toms such as only mild shortness of breath should not preclude consideration of a possible diagnosis of TRALI, even though the symptoms may not completely fi t the consensus criteria.

The exact pathogenesis of TRALI is unknown but usually involves anti-HLA antibodies (and/or antineutrophil antibodies) that trigger a cascade of events resulting in bilateral lung infi ltrates involving both interstitial and alveolar spaces. In some cases, the presentation of TRALI following RBC transfusions is thought to be mediated by RBC membrane components, specifi cally lysophosphatidylcholine. The presence of the pathogenic antibodies can be determined with tests such as the panel reactive antibody (PRA) assay although this assay will not detect antineutrophil antibodies or biologically acti- vated lipids that can also cause TRALI.

Case with Error

A 35-year-old male with an established diagnosis of myasthenia gra- vis is undergoing therapeutic plasma exchange (TPE) to decrease his

EEichbaum_87963_PTR_CH03_10-01-12_051-136.inddichbaum_87963_PTR_CH03_10-01-12_051-136.indd 112112 111/10/121/10/12 1:001:00 PMPM Transfusion-Related Acute Lung Injury (TRALI)—Failure to Diagnose 113

elevated titer of acetylcholine receptor antibodies. He has undergone several rounds of TPE over the past 3 months without complication. About an hour after starting this round of TPE, he develops a fever with rigors and experiences mild shortness of breath. The procedure is discontinued. His vital signs are pulse, 88; blood pressure, 140/65; tem- perature, 39.2°C; and oxygen saturation of 89% on room air. The nurse practitioner gives the patient acetaminophen and diphenhydramine. Oxygen is given via nasal cannula, and his saturation improves. To rule out the possibility of contaminated blood products, the plasma units are sent for culture. The patient is returned to the ward for observation. Two hours later the attending physician receives a call that the patient is severely dyspneic. A chest X-ray is ordered and shows bilat- eral infi ltrates involving alveolar and interstitial spaces.

Clinical Pitfall

Failure to consider a diagnosis of TRALI when the presenting symp- toms are mild or atypical. The source of the error in this case is not maintaining a suffi ciently high index of suspicion for TRALI when the presenting symptoms are mild or atypical. An error can also occur or when an underlying disease may offer alternative explanations for the symptoms. TRALI remains the leading cause of transfusion-related mortality reported to the Food and Drug Administration (FDA), and suspicious symptoms should therefore be appropriately pursued with further evaluation.

Explanation and Consequences

The bilateral infi ltrates on chest X-ray suggested the possibility of a TRALI reaction to the unit(s) of the plasma exchanged during the TPE. Testing of the plasma units for anti-HLA antibodies revealed that one of the donors had an HLA Class I PRA of 90% and an HLA Class II PRA of 82% as determined by an assay involving Luminex beads (One Lambda, Inc.). Antigen specifi city testing of the patient confi rmed the possibility of an antibody/antigen reaction as a potential trigger for the TRALI reaction. The patient’s atypical presenting symptoms of chills, rigors, and mild shortness of breath diverted suspicion from TRALI

EEichbaum_87963_PTR_CH03_10-01-12_051-136.inddichbaum_87963_PTR_CH03_10-01-12_051-136.indd 113113 111/10/121/10/12 1:001:00 PMPM 114 3. Errors Involving Specifi c Clinical Scenarios

toward more common explanations for the patient’s symptoms. The use of male-only plasma for TPE in the United Kingdom and some centers in the United States has served to mitigate the incidence of TRALI where such plasma is used.

STANDARDS OF CARE

Given the relative mortality risk of TRALI, a high index of suspicion should be maintained even when symptoms are mild, especially in the setting of other possible disease etiologies. The Canadian Con- sensus Panel and the NIH criteria for TRALI should generally be applied in diagnosing the condition, but a high level of suspicion should nonetheless be maintained for mild and atypical cases. Plasma units from donors (especially female; multiparous) should be analyzed for anti-HLA Class I or Class II antibodies (or antineu- trophil antibodies). When such antibodies are discovered, the patient is typically tested for the cognate antigen to understand whether an antibody/antigen interaction may have triggered the transfusion reaction. Donors of positive units should be excluded from further donation of plasma, but they may be allowed to donate platelets.

RECOMMENDED READING

Kleinman S, Caulfi eld T, Chan P, et al. Toward an understanding of transfusion-related acute lung injury: statement on the consensus panel. Transfusion 2004;44:1774–1789. Toy P, Gajic O, Bacchetti P, et al. Transfusion-related acute lung injury: incidence and risk factors. Blood 2012;119(7):1757–1767. Davis A, Mandal R, Johnson M, et al. A touch of TRALI. Transfusion 2008;48:541–545.

EEichbaum_87963_PTR_CH03_10-01-12_051-136.inddichbaum_87963_PTR_CH03_10-01-12_051-136.indd 114114 111/10/121/10/12 1:001:00 PMPM Failure to Recognize that Serious, Potentially Fatal Hemolytic Transfusion Reactions 115

FAILURE TO RECOGNIZE THAT SERIOUS, POTENTIALLY FATAL HEMOLYTIC TRANSFUSION REACTIONS CAN OCCUR WITH BLOOD PRODUCTS OVERVIEW

Severe allergic reactions can occur in response to plasma proteins or other agents contained in blood products. Type I hypersensitivity responses occur very rapidly following contact with the relevant antigens and recur on subsequent occasions via an IgE-mediated degranulation of mast cells and basophils. The organ systems affected include skin, and the mucosa of the gas- trointestinal and respiratory tracts where vascular leakage and tissue edema may occur. Arterial dilatation may cause headache and hypotension, whereas bronchoconstriction can cause respi- ratory distress. The mediators of these responses include hista- mine, serotonin and bradykinin, lymphokines, leukotrienes, and the anaphylatoxins C3a and C5a.

Case with Error

A 30-year-old pregnant female with a history of thrombocytopenia (platelet count of 55,000/μL) is about to undergo a cesarean (C-) sec- tion. With the initial incision, the obstetrician perceives more bleeding than expected and requests platelets for transfusion. The entire bag of 340 mL of platelets is transfused within 7 minutes. As the transfusion is completed, the blood pressure drops precipitously from 140/60 to 65/38 mm Hg. The patient becomes tachypneic, requiring intubation and pressor support. An emergency C-section is performed, and the patient is transferred to the ICU.

Clinical Pitfall

Failure to be prepared for anaphylactic transfusion reactions, particu- larly following rapid platelet transfusions.

Eichbaum_87963_PTR_CH03_10-01-12_051-136.indd 115 11/10/12 1:00 PM 116 3. Errors Involving Specifi c Clinical Scenarios

Most allergic reactions are mild and often limited to cutaneous mani- festations (hives, redness). It is important to be aware that a spectrum of more severe reactions may occur. To cope with such reactions, an intensive care plan should be devised, and support systems (eg, Advanced Cardiac Life Support) should be available in close proximity to the patient being transfused. Knowing the patient’s transfusion history, especially if the patient has had previous allergic episodes, is also important.

Explanation and Consequences

This patient experienced an anaphylactic transfusion reaction follow- ing platelet transfusion. A cause that should always be considered is severe IgA defi ciency that can lead to the development of an anti-IgA antibody. As most blood products contain IgA, immune complexes can be formed between this IgA in the product and the recipient’s anti- IgA antibody. These immune complexes can activate complement, resulting in the release of the anaphylatoxins C3a and C5b. A diagnosis of anaphylaxis due to IgA defi ciency is made by demonstrating severe IgA defi ciency and the presence of an anti-IgA antibody in the recipient. Up to 59% of normal individuals have anti- IgA in their serum, but not all individuals with anti-IgA, or with IgA defi ciency, develop anaphylactic reactions. Furthermore, anaphylac- tic reactions have been observed in individuals with IgA defi ciency but without detectable anti-IgA. IgA defi cient individuals should therefore not be routinely restricted to washed or IgA-defi cient blood products without at least a trial of unmodifi ed blood products. How- ever, patients who have documented anaphylactic reactions due to a diagnosed IgA defi ciency should possibly carry this information on a wristband to alert physicians. Even though RBCs and platelets both contain a signifi cant amount of plasma, platelets are more likely to cause severe allergic-type trans- fusion reactions. The reasons for this difference are not known. Slow- ing down the infusion of platelets can help mitigate the severity of the reaction. AABB guidelines recommend platelet transfusion rates for adults of 200 to 300 mL/h, and of 60 to 120 mL/h for pediatric patients. This patient received approximately 340 mL in 7 minutes, which likely contributed to the severity of the transfusion reaction.

EEichbaum_87963_PTR_CH03_10-01-12_051-136.inddichbaum_87963_PTR_CH03_10-01-12_051-136.indd 116116 111/10/121/10/12 1:001:00 PMPM Recommended Reading 117

STANDARDS OF CARE

Supportive care (Advanced Cardiac Life Support and ICU support) should be readily available. Platelets should be transfused slowly and by gravity. Although plate- lets can be given via a pump, they should never be given by rapid infusion. IgA defi cient patients, especially those with documented anaphy- lactic reactions, should be transfused with products derived from other IgA-defi cient blood donors or with washed cellular blood products.

RECOMMENDED READING

Sandler SG. How I manage patients suspected of having had an IgA anaphylactic transfusion reaction. Transfusion 2006;46:10–13. Sandler SG, Mallory D, Malamut D, Eckrich R. IgA anaphylactic transfusion reactions. Transfus. Med. Rev .1995;9:1–8.

EEichbaum_87963_PTR_CH03_10-01-12_051-136.inddichbaum_87963_PTR_CH03_10-01-12_051-136.indd 117117 111/10/121/10/12 1:001:00 PMPM 118 3. Errors Involving Specifi c Clinical Scenarios

FAILURE TO RECOGNIZE DRUG-INDUCED HEMOLYTIC ANEMIA (DIHA)

OVERVIEW

DIHA is a rare complication occurring in less than one in a million individuals. Drugs may induce the generation of an antibody against RBCs that recognize the RBC only in the presence of the drug (classic form). However, they may also generate true autoantibodies to RBCs that do not require the presence of the drug for recognition. The number of drugs now known to be associated with DIHA is about 130. The most fre- quently implicated drugs are piperacillin and the cephalospo- rins. At least four explanatory mechanisms of DIHA have been described. One such mechanism entails binding of the drug to the RBC membrane that then elicits an antibody directed largely to the drug itself and that can lead to extravascular hemolysis.

Case with Error

A 23-year-old woman with cystic fi brosis presents with acute respira- tory failure. She is admitted to the medical intensive care unit and started on intravenous piperacillin-tazobactam for treatment of pre- sumed Pseudomonas aeruginosa pneumonia. At admission, the patient’s hemoglobin is 13.1 g/dL. During the fi rst 11 days of her hospitalization, her hemoglobin ranges from 8.4 to 9.8 g/dL. On hospital day 12, her hemoglobin begins to trend down, reaching a nadir of 4.7 g/dL on hospital day 15, despite the transfusion of 2 units of RBCs. Initially, the etiology of this anemia is unknown. Sources of occult bleeding are sought; iron defi ciency is considered but fi nally the patient assumes a “DNR” status. Examination of the peripheral blood smear demonstrates marked anemia, reticulocytosis, polychroma- of total RBCs), which %65ف) sia, and profound microspherocytosis prompts further evaluation from the laboratory physicians. Her LDH

EEichbaum_87963_PTR_CH03_10-01-12_051-136.inddichbaum_87963_PTR_CH03_10-01-12_051-136.indd 118118 111/10/121/10/12 1:001:00 PMPM Failure to Recognize Drug-Induced Hemolytic Anemia (DIHA) 119

is elevated at 531 U/L, and total bilirubin is increased at 2.9 mg/dL. Prior to the transfusion of RBCs, her serum haptoglobin was 122 mg/ dL but decreased to undetectable levels after transfusion. The DAT is 3+ positive for IgG and negative for complement. The eluate is negative. DIHA is suspected and the drug piperacillin-tazobactam is with- drawn. The patient subsequently receives two additional units of RBCs, after which her hemoglobin stabilizes and then increases over the next several days without further transfusion. She is discharged on hospital day 22 with a hemoglobin of 10.4 g/dL. Subsequent testing identifi es hemolytic antipiperacillin antibod- ies in the patient’s serum as well as anti-C3 on the patient’s RBCs. The patient is informed of her adverse reaction to piperacillin and her elec- tronic medical record is updated to avoid future exposure to the drug.

Clinical Pitfall

Failure to diagnose DIHA in a critically ill patient receiving medica- tions known to be implicated in the disease. Because DIHA is rare, it is often not considered in patients who present with unexplained anemia. Such patients may be brought to attention by the transfusion service if the laboratory detects a positive DAT with a negative eluate in a signifi cantly anemic patient. The elu- ate test is often routinely performed on patients who have a positive screen for an RBC antibody in their plasma. However, if the patient’s antibody screen is negative, no DAT is usually performed. One should have a high index of suspicion in patients receiving a new drug regi- men who present with relatively acute onset of anemia. This applies in particular to drugs known to be implicated in DIHA (eg, penicillin, piperacillin, the cephalosporins, and methyldopa).

Explanation and Consequences

Laboratory confi rmation of DIHA is often not readily available; so if there is a high index of suspicion for this disorder, the drug should be discontinued. Typically, one should see correction of laboratory val- ues and improvement in the hematocrit within 3 days. In this patient, the abundant microspherocytes in her peripheral smear strongly

EEichbaum_87963_PTR_CH03_10-01-12_051-136.inddichbaum_87963_PTR_CH03_10-01-12_051-136.indd 119119 111/10/121/10/12 1:001:00 PMPM 120 3. Errors Involving Specifi c Clinical Scenarios

suggested immune hemolysis. The DAT was strongly positive, but the eluate was negative—a classic fi nding in DIHA. Piperacillin is one of the most common causes of DIHA. In contrast to penicillin, the in vivo destruction of RBCs by piperacillin may be complement medi- ated. Owing to the number of medications the patient is taking, or the lack of available serological testing, it may be diffi cult to identify the causative medication.

STANDARDS OF CARE

DIHA should be considered in patients who present with unex- plained anemia, particularly if they are receiving drugs associated with DIHA.

RECOMMENDED READING

Garratty G. Drug-induced immune hemolytic anemia. Hematol. Am. Soc. Hematol. Educ. Program 2009;1:73–79. Garbe E, Andersohn F, Bronder E, et al. Drug induced immune hae- molytic anaemia in the Berlin case-control surveillance study. Br. J. Haematol . 2011;154:644–653.

EEichbaum_87963_PTR_CH03_10-01-12_051-136.inddichbaum_87963_PTR_CH03_10-01-12_051-136.indd 120120 111/10/121/10/12 1:001:00 PMPM Failure to Recognize that Donor HIV Exposure 121

FAILURE TO RECOGNIZE THAT DONOR HIV EXPOSURE IS ASSOCIATED WITH A SPECIFIC FEDERALLY MANDATED PROCESS FOR RECIPIENT NOTIFICATION

OVERVIEW

A look-back investigation on the transfusion service func- tions to identify potential transfusion-infected recipients and the implicated donors. For some disease entities, such as HIV, federal regulations guide the investigations. Transmission of human immunodefi ciency virus (HIV) through transfusion of contaminated blood components was documented in the United States in 1982. Virtually all of these transfusion-associated cases occurred before 1985, when HIV antibody testing was not available. Since then, the risk for transfusion-transmitted HIV infection has been almost eliminated through the use of ques- tionnaires to exclude the high risk donor, and through the appli- cation of sensitive laboratory screening tests to identify infected blood donors.

The risk for acquiring HIV infection through blood transfusion today is estimated conservatively to be about one in two million, based on 2007 to 2008 data. When a blood collection center identifi es a donor whose blood may be infected with hepatitis C virus (HCV), HIV-1 and/or HIV-2, written notifi cation of the infection is sent to all medi- cal directors of blood banks that might have received previous blood products from that infected donor. Federal regulations also contain stipulations regarding such notifi cations about infected donors.

Case with Error

A blood center in the Midwest United States discovers that blood components from a 45-year-old male donor with a long history of suc- cessful donations now tests positive for HIV by nucleic acid (PCR)

EEichbaum_87963_PTR_CH03_10-01-12_051-136.inddichbaum_87963_PTR_CH03_10-01-12_051-136.indd 121121 111/10/121/10/12 1:001:00 PMPM 122 3. Errors Involving Specifi c Clinical Scenarios

testing, as well as by serologic testing for HIV antibody and an HIV- specifi c protein. These tests are repeated and confi rmed. The donor is asked to return to the donation center and receives counseling. The blood center contacts the hospital to which they had sent the com- ponents, providing them with a notifi cation of the type of product shipped, its ABO/Rh type, the date of shipment, and product expira- tion date. The RBC product from this donation was transfused to a 60-year-old female with breast cancer. The FFP and platelets gener- ated from this donation were not transfused. The FFP was quarantined and destroyed. The transfusion service staff attempted to notify the , but that person had left the practice and no longer cared for this patient. The transfusion service does not follow- up further.

Clinical Pitfall

Failure to be informed of the FDA-mandated regulations governing donor and recipient notifi cations about HIV and/or HCV infected blood products. Although the incidence of transfusion-transmitted HIV and HCV cases is low, transfusion service staff, physicians, and patient care providers must be aware of their respective responsibilities dur- ing the look-back process. Notifi cation must be completed within a specifi c time frame. Transfusion services must have documenta- tion that the recipient was notifi ed prior to fi nalizing the look-back investigation.

Explanation and Consequences

All donor and transfusion services are required to follow specifi c look-back guidelines for tests repeatedly reactive to HIV antibody for all products collected within 5 years from the originally positive test. Typically the transfusion service notifi es the attending physician to inform the recipient of the need for HIV testing and counseling. If the physician is unavailable or declines to notify the recipient, the transfu- sion service must notify the recipient and inform the recipient of the need for testing and counseling.

Eichbaum_87963_PTR_CH03_10-01-12_051-136.indd 122 11/10/12 1:00 PM Recommended Reading 123

STANDARDS OF CARE

Consent must be obtained from all patients for blood product admin- istration. As part of that consent process, the risk of infectious dis- eases, including HIV and HCV, must be addressed. The physician and blood bank should keep a record of these notifi cations and test results. When supplemental testing for HIV or HCV is positive, the transfu- sion service medical director is responsible for notifying the donor as well as all patients who may be affected. Reasonable attempts must be made to notify the recipient within 8 weeks after receiving the donor testing results. A legal representative or relative of the recipient should be notifi ed if the recipient is a minor, deceased, or adjudged incompetent. At least three attempts to notify the affected parties must be made.

RECOMMENDED READING

Centers for Disease Control and Prevention (CDC). HIV transmission through transfusion—Missouri and Colorado, 2008. MMWR Morb Mortal Wkly Rep. 2010;59(41):1335–1339.

EEichbaum_87963_PTR_CH03_10-01-12_051-136.inddichbaum_87963_PTR_CH03_10-01-12_051-136.indd 123123 111/10/121/10/12 1:001:00 PMPM 124 3. Errors Involving Specifi c Clinical Scenarios

RISK OF HYPERKALEMIA FROM RBC TRANSFUSION

OVERVIEW

Damage to the red cell membrane or lysis of the RBC results in release of intracellular potassium. Complications of hyperkalemia can arise in patients after receiving multiple RBC transfusions. A major determinant of the risk of hyperka- lemia following transfusion is the potassium concentration of the unit. However, other factors are also involved, including the age of the patient, presence of tissue hypoperfusion, and the effects on the patient of the rate of transfusion. Pediatric patients with cardiac disease are particularly sensitive to hyperkalemia. Many institutions wash RBCs to reduce total potassium load prior to issuing units to pediatric patients. How- ever, adults with poor cardiac output are also at risk.

Case with Error

A 55-year-old male with long-standing type 2 diabetes is undergoing triple bypass surgery. The surgeons order 4 units of RBCs. Not wishing to waste blood, the blood bank issues 4 units of RBCs that had been irradiated for another patient but returned to the inventory within the regulated time frame and at the appropriate temperature. During the surgery the patient receives the 4 units of RBCs over 2 hours. Shortly after receiving the units, the patient’s electrocardiogram (ECG) reveals ventricular fi brillation indicative of cardiac arrest. The patient is resuscitated but continues to experience hypo- tension and hypoxia. His ECG and chest X-ray are now similar to those recorded on admission. Laboratory chemistry results show the patient’s serum potassium to be 7.2 mEq/L. The physician considers the patient to be hyperglycemic and orders a blood glucose measure- ment, but the blood glucose result is within the normal range.

EEichbaum_87963_PTR_CH03_10-01-12_051-136.inddichbaum_87963_PTR_CH03_10-01-12_051-136.indd 124124 111/10/121/10/12 1:001:00 PMPM Risk of Hyperkalemia from RBC Transfusion 125

Clinical Pitfall

Failure to recognize that hyperkalemia may be associated with RBC transfusions. The mean potassium concentration in an RBC unit is propor- tional to the age of the unit. RBC units stored for less than 14 days are believed to contain less potassium than ones stored for longer time. Fresh blood units less than 5 days old typically have mean potassium concentrations of less than 10 mEq/L. After a unit has been irradiated, potassium concentration of the unit increases markedly over the next 4 hours. Therefore, any banked unit that has been previously irradiated will have a very high potassium level (>50 mEq/L) and should not be issued to vulnerable patients.

Explanation and Consequences

This adult patient experienced cardiac arrest due to hyperkalemia fol- lowing the infusion of only 4 units of RBCs over 2 hours, a rate that would not be considered exceptional (as in a “massive transfusion”). At the time, the patient was hyperglycemic (glucose of 288 mg/dL), acidotic (7.2 pH), and hypocalcemic (ionized calcium of 3.3 mg/dL). The potassium levels of the implicated units were measured using the residual sections of unit tubing retained by the blood bank. The age of all the units was less than 14 days. The mean potassium levels, however, of each of the units exceeded 60 mEq/L. All of the units had been previ- ously irradiated in anticipation of transfusion to a different patient but, when not used, were returned to general inventory. The units transfused in this patient had exceedingly high levels of potassium, likely due to having been previously irradiated. Factors that may contribute to hyperkalemia due to alterations in potassium redistribution in the patient include hyperglycemia, hypo- calcemia, hypothermia, and acidosis. Surgical stress and shock cause potassium to exit the cells. Hypocalcemia (which often accompanies massive transfusion) causes cardiac membrane instability at lower potassium levels. Acidosis contributes to an extracellular potassium shift, which can compound the effect of hyperkalemia. Although not experienced by this patient, hypothermia can also exacerbate the prob- lem by sensitizing the myocardium to the toxic effects of potassium.

EEichbaum_87963_PTR_CH03_10-01-12_051-136.inddichbaum_87963_PTR_CH03_10-01-12_051-136.indd 125125 111/10/121/10/12 1:001:00 PMPM 126 3. Errors Involving Specifi c Clinical Scenarios

STANDARDS OF CARE

Irradiated RBC units should ideally be used within 6 to 8 hours of irradiation, although AABB standards allow longer out-dates. If issue of irradiated products within 6 to 8 hours is not possible, efforts should nonetheless be made not to issue these units to the surgical or ICU patient populations, as they are at higher risk for the metabolic derangements associated with hyperkalemic cardiac complications.

RECOMMENDED READING

O’Leary MF, Szklarski P, Klein TM, Young PP. Hemolysis of red blood cells after cell washing with different automated technologies: clinical implications in a neonatal cardiac surgery population. Trans- fusion 2011;51:955–960. Smith HM, Farrow SJ, Ackerman JD, et al. Cardiac arrests associated with hyperkalemia during red blood cell transfusion: a case series. Anesth. Analg . 2008;106:1062–1069.

EEichbaum_87963_PTR_CH03_10-01-12_051-136.inddichbaum_87963_PTR_CH03_10-01-12_051-136.indd 126126 111/10/121/10/12 1:001:00 PMPM Failure to Recognize that Intensive Plasma Exchange 127

FAILURE TO RECOGNIZE THAT INTENSIVE PLASMA EXCHANGE CAN CAUSE METABOLIC ALKALOSIS

OVERVIEW

Thrombotic thrombocytopenic purpura (TTP) is a rare blood condition that causes microthrombi to form in small blood vessels throughout the body. Classically, the following fi ve features are indicative of TTP: neurologic symptoms, kid- ney failure, fever, thrombocytopenia, and MAHA. For unclear reasons, the kidney microvasculature is particularly sensitive to the formation of microthrombi, and some level of kidney dys- function is, therefore, present in many TTP patients. TPE with donor plasma as fl uid replacement is the fi rst-line therapy for TTP, a life-threatening disease with considerable mortality (despite the effi cacy of TPE). TPE is typically performed daily until the platelet count and LDH are back in the normal range.

Case with Error

A 52-year-old African American female has a history of chronic relaps- ing TTP. Previous acute episodes of TTP had resulted in partial loss of sight in the right eye and chronic renal insuffi ciency. She returns to the hospital as a result of an acute relapse. She begins to expe- rience headaches and feels weak the night of admission. On admis- sion, her platelet count is 25,000/μL; LDH, 1,200 U/L; Hgb, 7 g/dL; and creatinine, 4.5 mg/dL, which is increased from her baseline of 1.2 mg/dL. TPE processing of 2.5 blood volumes begins immediately and is performed daily. Each day, she receives approximately 3.7 L of exchanged FFP as part of her TPE. On the 10th hospital day, her bicarbonate concentration is 38 mEq/L and arterial pH is alkalotic at 7.54. The service is consulted, and their assessment is that the patient is becoming alkalotic due to surplus citrate from plasma

EEichbaum_87963_PTR_CH03_10-01-12_051-136.inddichbaum_87963_PTR_CH03_10-01-12_051-136.indd 127127 111/10/121/10/12 1:001:00 PMPM 128 3. Errors Involving Specifi c Clinical Scenarios

exchange in the setting of renal insuffi ciency. Their recommendation is to perform TPE using heparin instead of citrate as the anticoagulant.

Clinical Pitfall

Failure to correctly assess the source of metabolic alkalosis resulting from blood products during intensive TPE. Plasma exchange is usually performed using citrate to prevent clotting in the extracorporeal circuit. The citrate returned to the patient’s body during the procedure is metabolized to bicarbonate in the liver. It is often assumed that the surplus bicarbonate is derived from the citrate used for the apheresis procedure. In reality, the pro- cedure returns a very small amount of citrate. The major source of the surplus citrate is the therapeutic administration of large amounts of donor plasma. In patients with signifi cant renal disease, the kidney fails to effi ciently excrete the bicarbonate surplus and this can lead to metabolic alkalosis.

Explanation and Consequences

This patient was receiving a small amount of citrate from the proce- dure itself, approximately 25 mg over the entire procedure. However, citrate is the major anticoagulant used for whole blood collection with approximately 70 mg of citrate contained in each unit of whole blood. A unit of whole blood is processed to generate packed RBCs, platelets, and plasma. The majority of the citrated plasma ends up in the platelet and plasma products. As a result of receiving almost 4 L of FFP daily, this patient is receiving approximately 280 mg of citrate from donor plasma. Patients undergoing intensive plasma exchange for TTP develop increasing plasma bicarbonate concentrations while concomitantly developing lower plasma potassium levels. Metabolic alkalosis is not observed in patients undergoing long-term plasma exchange who receive albumin as replacement fl uid. As the source of the alkalosis is the plasma, there is no need to modify the treatment methodology (ie, perform plasma exchange using heparin) in order to avoid it.

Eichbaum_87963_PTR_CH03_10-01-12_051-136.indd 128 11/10/12 1:00 PM Recommended Reading 129

STANDARDS OF CARE

Patients receiving large amounts of plasma, particularly those with renal insuffi ciency, should be assessed with venous pH and bicar- bonate measurements because of the risk of metabolic alkalosis.

RECOMMENDED READING

Marques MB, Huang ST. Patients with thrombotic thrombocytope- nic purpura commonly develop metabolic alkalosis during therapeutic plasma exchange. J. Clin. Apher . 2001;16:120–124.

EEichbaum_87963_PTR_CH03_10-01-12_051-136.inddichbaum_87963_PTR_CH03_10-01-12_051-136.indd 129129 111/10/121/10/12 1:001:00 PMPM 130 3. Errors Involving Specifi c Clinical Scenarios

FAILURE TO RECOGNIZE THE RISK OF ALLOIMMUNE THROMBOCYTOPENIA IN A PRIMIGRAVIDA

OVERVIEW

NAIT occurs as a result of maternal alloimmunization to fetal platelet antigens that lead to the destruction of fetal platelets. Unlike HDFN, in which fetal anemia develops due to maternal sensitization of fetal red cell antigens and hemolysis arises in second or later pregnancies, one-third to one-half of infants with NAIT are affected during the fi rst pregnancy. The mechanism of cellular destruction is analogous to that of red cell incompatibility and destruction that occurs in HDFN.

Transplacental passage of fetal platelets in early second trimester sensitizes the mother, who produces IgG antibodies directed against platelet specifi c (or more rarely HLA) antigens of paternal origin on the fetal platelets. The neonate may present with a bleeding diathesis of variable severity after birth, or more rarely in utero. NAIT complicates 1.5 per 1000 to 1 per 5000 live birth Caucasian births. Although NAIT accounts for only 3% of all general fetal and neonatal thrombocytope- nias (<150,000/␮L ), it accounts for about 30% of all severe thrombo- cytopenias with platelet counts of less than 50,000 /␮ L.

Case with Error

A 20-year-old female G1P0 gives birth at term by spontaneous vaginal delivery to a baby boy with good APGAR scores and a normal new- born exam. On day 2 after birth, the nurse notes some bruises on the baby’s back and legs. A CBC is obtained, and the baby’s platelet count is noted to be 45,000/␮L. The team begins an evaluation for thrombocytopenia, which does not include NAIT in the differential diagnosis due to the fact that this is a fi rst pregnancy. The evalua- tion for infection and DIC are negative. The presumptive diagnosis is

EEichbaum_87963_PTR_CH03_10-01-12_051-136.inddichbaum_87963_PTR_CH03_10-01-12_051-136.indd 130130 111/10/121/10/12 1:001:00 PMPM Failure to Recognize the Risk of Alloimmune Thrombocytopenia 131

ITP. The team considers both intravenous gamma globulin (IVIG) and steroids as treatments but notices that the platelet count seems to be improving on its own in 1 week.

Clinical Pitfall

Failure to understand passive alloimmune destruction of platelets in neonates can occur in fi rst pregnancies. Platelets express specifi c antigens, such as the human platelet antigens, HPA1a, and HPA2a, which are polymorphisms of platelet glycoproteins. The antigens most often implicated in NAIT are highly prevalent in the population. In Caucasians, HPA1a (formerly PLA1) is the major cause of NAIT, followed by HPA5b (formerly Br-a) and PLA3a (formerly Baka). Most clinical neonatologists assume that all suspected cases of NAIT are due to HPA1a and often request HPA1a- negative platelets for transfusion. It should not, however, be assumed that all cases of NAIT in the United States are secondary to anti- HPA1a. Efforts to obtain special platelets should not be undertaken without confi rmation of the platelet antigen specifi city. The treatment for NAIT fortunately is very similar to ITP (IVIG with or without ste- roids). While platelet transfusion should generally be avoided in ITP, they are not contraindicated in NAIT.

Explanation and Consequences

Although 2% of all Caucasians are HPA1a negative, NAIT develops in only approximately 10% of HPA1 negative mothers, suggesting that the mother’s immunogenetic background plays a major role in reducing the frequency of this disorder. The comprehensive testing strategy for NAIT involves examining maternal plasma for antiplate- let antibodies. If antiplatelet antibodies are identifi ed, a test is per- formed to confi rm that the identifi ed antibodies recognize and react to the father’s and the baby’s platelets. Some centers will also genotype the mother and father for major platelet antigen polymorphisms, pro- viding an additional level of confi rmation. Without knowledge of the implicated antigen, it is best to transfuse the baby with washed and irradiated maternal platelets, as these should be compatible with the infant. Efforts should be made to eliminate or reduce maternal plasma

EEichbaum_87963_PTR_CH03_10-01-12_051-136.inddichbaum_87963_PTR_CH03_10-01-12_051-136.indd 131131 111/10/121/10/12 1:001:00 PMPM 132 3. Errors Involving Specifi c Clinical Scenarios

in the transfused products as it contains the offending antibody that recognizes and mediates immune clearance of the baby’s platelets. Some centers prefer washing maternal platelets as this more com- pletely reduces maternal antibodies. Washed platelets suffer some loss of function through degranulation, so some centers prefer to vol- ume reduce the platelet unit. If the implicated antigen is known to be HPA1a, recruiting an HPA1a-negative platelet donor to provide trans- fusion support for the patient is an option. If NAIT is confi rmed, sub- sequent pregnancies should be more carefully monitored. IVIG can be used prenatally as treatment until birth even without confi rming fetal thrombocytopenia. IVIG is thought to act by one or more of the fol- lowing mechanisms: suppressing platelet antibody synthesis, block- ing antibody transfer across placenta, competitively inhibiting platelet binding to platelet antibodies, and interfering with phagocyte- mediated immune clearance of platelets by the reticuloendothelial system.

STANDARDS OF CARE

NAIT should be considered in the differential diagnosis of newborns with thrombocytopenia, even for fi rst pregnancies. Laboratory test- ing should be used to determine the specifi city of the implicated antigen/antibody. Prophylactic transfusions are often recommended for low-neonatal platelet counts ranging between 30,000 and 50,000 / ␮L . After a fi rst case of NAIT, subsequent pregnancies should be care- fully monitored for the development of NAIT. The mother can also receive prenatal treatment with IVIG.

RECOMMENDED READING

Bassler D, Greinacher A, Okascharoen C, et al. A systematic review and survey of the management of unexpected neonatal alloimmune thrombocytopenia. Transfusion 2008;48:92–98. Bessos H, Seghatchian J. What’s happening? The expanding role of apheresis platelet support in neonatal alloimmune thrombocytopenia: current status and future trends. Transfus. Apher. Sci. 2005;33:191–197.

EEichbaum_87963_PTR_CH03_10-01-12_051-136.inddichbaum_87963_PTR_CH03_10-01-12_051-136.indd 132132 111/10/121/10/12 1:001:00 PMPM Unnecessary Platelet Transfusions 133

UNNECESSARY PLATELET TRANSFUSIONS

OVERVIEW

The causal relationship between thrombocytopenia and an increased risk of bleeding is well established. Recommen- dations for a threshold number of platelets to prompt transfusion for in-patients, without any clinical bleeding, range between 5,000–10,000/ ␮L. In the treatment of leukemias, only limited guidance is available on prophylactic platelet transfusion trig- gers for minor procedures such as central venous line placement.

Case with Error

A 53-year-old male with acute lymphocytic leukemia (ALL) requires the placement of a central venous four-lumen catheter for transfu- sion support and administration of chemotherapy. The patient has undergone induction chemotherapy, and his hematocrit is 29% with a platelet count of 28,000/ ␮L. Other than some mild mucositis, the patient does not have any complications from therapy. The primary clinical team contacts interventional for catheter placement. The radiologists request the transfusion of 2 units of apheresis plate- lets prophylactically. The platelets are appropriately transfused before starting the procedure and the catheter is successfully placed. The post-procedure platelet count is 90,000 / ␮L .

Clinical Pitfall

Failure to recognize the appropriate prophylactic trigger for transfu- sion of platelets. The benefi ts of platelet transfusions must be balanced against the potential risks of transfusion. Platelet transfusions are associated with a number of side effects, including febrile or allergic transfusion reactions, transmission of bacterial and viral infections, circulatory overload, alloimmunization leading to refractoriness to future platelet

EEichbaum_87963_PTR_CH03_10-01-12_051-136.inddichbaum_87963_PTR_CH03_10-01-12_051-136.indd 133133 111/10/121/10/12 1:001:00 PMPM 134 3. Errors Involving Specifi c Clinical Scenarios

transfusions, and TRALI. Moreover, platelets have a short shelf life of only 5 days, making them a relatively scarce and expensive commodity.

Explanation and Consequences

A commonly confronted uncertainty is the threshold at which a platelet transfusion should be initiated prior to a minimally invasive procedure. Several randomized prospective trials have shown that in patients with hematologic malignancies, a lower prophylactic platelet transfusion threshold of 10,000/μL results in an approximately 30% cost saving and does not present a higher risk of bleeding when com- pared with the higher threshold of 20,000/μL. To control or prevent bleeding in trauma and surgery, higher platelet transfusion thresholds are currently recommended: 100,000/μL for neurosurgical proce- dures and approximately 50,000/μL for trauma and other surgical procedures. A recent retrospective study of 604 catheter placements suggests that a platelet count of 20,000/μL is a safe level for central venous catheter placement. A second important question is how many platelets to trans- fuse. An average-sized adult should receive an increase of about 30,000 to 50,000/μL platelets with a standard transfusion of apheresis or random donor platelets. There are many things that may affect this increase, including body surface area of the patient, the number of platelets in the bag (typically at least 3 × 10 11 ), splenomegaly, infec- tion, and the recipient’s medications. In addition, a patient may fail to have an optimal response due to immune destruction of platelets. It is important to obtain a platelet count within 15 to 60 minutes after each transfusion to effectively guide therapy.

STANDARDS OF CARE

A generally accepted platelet transfusion threshold for a “nonbleed- ing” in-hospital patient, without any coagulopathy, is 10,000/μL. Current recommendations for platelet transfusion thresholds for is 50,000/μL. The response to a platelet transfusion is best assessed by obtaining a posttransfusion platelet count. This should be obtained between 15 and 60 minutes after completion

Eichbaum_87963_PTR_CH03_10-01-12_051-136.indd 134 11/10/12 1:00 PM Recommended Reading 135

of the transfusion. The effectiveness of the response is assessed by calculating a CCI (see case “Incomplete Evaluation of Platelet Refractoriness”).

RECOMMENDED READING

Oh H, Loberiza FR Jr, Zhang MJ, et al. Comparison of graft-versus- host-disease and survival after HLA-identical sibling bone marrow transplantation in ethnic populations. Blood 2005; 105:1408–1416. Zeidler K, Arn K, Senn O, et al. Optimal preprocedural platelet trans- fusion threshold for central venous catheter insertions in patients with thrombocytopenia. Transfusion 2011;51:2269–2276.

EEichbaum_87963_PTR_CH03_10-01-12_051-136.inddichbaum_87963_PTR_CH03_10-01-12_051-136.indd 135135 111/10/121/10/12 1:001:00 PMPM EEichbaum_87963_PTR_CH03_10-01-12_051-136.inddichbaum_87963_PTR_CH03_10-01-12_051-136.indd 136136 111/10/121/10/12 1:001:00 PMPM Index

ABO typing discrepancy, due to less Asymptomatic anemia common ABO subgroups, failure to follow evidence-based 28–29 guidelines for management Acetylsalicylic acid, 70 of, 59–60 ADAMTS13 test, 109–111 Autologous RBC transfusions Albumin, 38, 39, 44 inappropriate use of, 31–33 Allergic and febrile nonhemolytic transfusion reaction Bilirubin, 26 (FNHTR), 103–105 Blood donors, syncopal reactions in Alloantibodies, 26. See also failure to recognize risk factors Anti-Kell alloantibodies for, 85–86 in pregnancy, identifi cation and Blood sample collection, error in management of, 56–58 resulting in inaccurate type and American Association of Blood screen, 23–24 Banks (AABB), 8, 33 Blood transfusion, refusal of, 79–80 American College of Obstetrics and Gynecology (ACOG), 8 Captopril, 101 Anaphylactic transfusion reaction CCI, calculation of, 106–108 following platelet transfusion, Cephalosporins, 118 failure to prepare for, Chronic anemia 115–117 rapid transfusion in, resulting in Anaphylaxis, diagnosis of, 116 volume overload, 34–35 Angiotensin-converting enzyme Citrate, 47, 128 inhibitors (ACE-i), 100–102 Clostridia, 82 Anti-A IgM class antibodies, 42 Coagulation factors, dilution Anti-A1 lectin, 28–29 of, 44–45 Anti-HLA antibodies, 112, 113 Coagulation screening tests, 3 Anti-IgA antibody, 97, 116 Cold agglutinin disease (CAD), 66 Anti-Kell alloantibodies insignifi cance of low antibody missed diagnosis of HDFN, 94–96 titers, 36–37 Anti-M alloantibody, 20, 21 Corifact, 13 Anticoagulant-associated Coumadin, 90 intracerebral hemorrhage Cryoprecipitate (AAICH), 90 failure to calculate correct Apheresis platelets, 70 dosage, 12–13 Aspirin inappropriate use of, 12–14 inappropriate platelet transfusion Cryptic antigen exposure, failure to for patients on, 70–72 consider, 82–83

EEichbaum_87963_PTR_IND_10-01-12_137-144.inddichbaum_87963_PTR_IND_10-01-12_137-144.indd 137137 111/10/121/10/12 12:2112:21 PMPM 138 Index

Delayed hemolytic transfusion failure to recognize the reactions (DHTRs), 25, 26 importance of correct blood occult anemia, searching for, sampling technique for 53–55 blood typing, 23–24 Direct antiglobulin test (DAT), 54 heparin-fl ushed lines, inadvertent misinterpretation of, 87–89 exposure to blood Direct Coombs’ test. See Direct circulation, 47–48 antiglobulin test (DAT) positive type and screen resulting Dolichos bifl orus, 28, 29 in potential delay in the Donath-Landsteiner (DL) test, 67–69 issue of blood, 20–22 Drug-induced hemolytic anemia Escherichia coli, 68, 83 (DIHA), failure to recognize, Extracorporeal photopheresis 118–120 (ECP), 47 Drug-induced immune hemolytic anemia (DIIHA), 66 Factor VIII, 13 Dysfi brinogenemia, 13 Factor XIII, 13 Fatal hemolytic transfusion Elution, 9, 87 reactions Errors in procedures occurring with blood products, ABO typing discrepancy, 28–29 failure to recognize, autologous blood, inappropriate 115–117 use of, 31–33 Fetal red blood cells, 8 cold agglutinin disease (CAD), Fetomaternal hemorrhage (FMH), insignifi cance of low 9–11 antibody titers, 36–37 Fibrinogen, 13–14 failure to compensate for dilution Fibronectin, 13 of coagulation factors from Forward type, 23, 28 plasmapheresis, 44–45 Four-factor prothrombin complex failure to diagnose hemolytic concentrates (PCCs), 90–91 transfusion reaction due Fresh frozen plasma (FFP) to minor incompatibility inappropriate use of, 90–93 between donor plasma to correct mildly elevated recipient RBCs, 41–43 prothrombin time (PT), 2–3 failure to monitor hypocalcemia usage of plasma products, during therapeutic failure to understand, 5–6 apheresis, 38–39 for volume expansion, 5–6 failure to recognize increased risk of TACO in patients Haemophilus infl uenzae, 68 with chronic anemia, 34–35 Hemolysis

EEichbaum_87963_PTR_IND_10-01-12_137-144.inddichbaum_87963_PTR_IND_10-01-12_137-144.indd 138138 111/10/121/10/12 12:2112:21 PMPM Index 139

following platelet transfusion, Intensive plasma exchange 41–43 causing metabolic alkalosis, immune-mediated hemolysis failure to recognize, 127–129 in pediatric patient, Intravenous gamma globulin recognition of, 66–69 (IVIG), 132 misinterpretation of laboratory Isohemagglutinins, 53 tests for, 25–27 Hemolytic disease of the fetus and Kidd blood group, 54–55 newborn (HDFN), 7, 56 Klebsiella pneumoniae, 68 missed diagnosis of, 94–96 Kleihauer-Betke test, 9–11 Hemolytic uremic syndrome (HUS), 82, 83 Lactate dehydrogenase (LDH) Hemophilia A, 13 level, 26 Heparin-fl ushed lines, inadvertent Lumbar puncture (LP), 61 exposure to blood circulation, 47–48 Maternal anti-Kell antibodies role HPA1a, 74, 131 in HDFN, failure to understand, HPA2a, 131 94–96 Human immunodefi ciency virus Metabolic alkalosis, failure (HIV) transmission, through to assess blood transfusion, 121–123 resulting from blood products during Hyperkalemia risk, from RBC intensive TPE, 127–129 transfusion, 124–126 Microangiopathic hemolytic anemia Hypocalcemia, 125 (MAHA), 82, 109 Hypocalcemic toxicity Minor blood group antigen symptoms of, 39 mismatches, 42 from therapeutic plasma Mixed type autoimmune-mediated exchange (TPE), 38–39 hemolytic anemia, 66 Hypofi brinogenemia, 13 Myasthenia gravis (MG), 38, 44 Hypotension misinterpretation, during transfusion, Necrotizing enterocolitis (NEC), 83 100–102 Neonatal alloimmune thrombocytopenia (NAIT) IgA defi ciency, 116 risk in primigravida, failure to misinterpretations and recognize, 130–132 assumptions, 97–99 Neuraminidase enzyme, 83 Immune-mediated hemolysis in pediatric patient, Occult anemia, searching for DHTR, recognition of, 66–69 53–55

Eichbaum_87963_PTR_IND_10-01-12_137-144.indd 139 11/10/12 12:21 PM 140 Index

Panel reactive antibody (PRA) for minimally invasive procedures, assay, 112 failure to recognize evidence- Paroxysmal cold hemoglobinuria based, 61–62 (PCH), 66, 68 unnecessary, 133–135 Partial D/weak D antigen testing, Posttransfusion purpura (PTP) failure to recognize in differential diagnosis of limitations of, 63–65 thrombocytopenia, failure Partial thromboplastin time to consider, 73–75 (PTT), 2, 23 Pregnancy, identifi cation Patient’s autonomy, failure to respect and management of and understand, 79–80 alloantibodies in, 56–58 Pediatric patient, immune-mediated Primigravida hemolysis in, 66–69 NAIT risk in, failure to Piperacillin, 118–120 recognize, 130–132 Plasma frozen within 24 hours of Product-related errors phlebotomy (FP24), 5 failure to keep platelet products Plasma products, failure to at appropriate temperature, understand appropriate use 15–17 of, 5–6 inappropriate use of Plasmapheresis, 44. See also cryoprecipitate, 12–14 Therapeutic plasma fresh frozen plasma exchange (TPE) (FFP), 2–6 failure to compensate for dilution Rh immune globulin of coagulation factors, 44–45 (RhIG), 7–11 Plasma/prothrombin complex Prophylactic platelet transfusions concentrates (PCCs), 90 for minor procedures, Platelet count determination, within non-evidence-based 15 to 60 minutes following practices in, 61–62 transfusion, 106–108 Prothrombin complex concentrates Platelet inactivation, as result of cold (PCCs), 90–93 exposure, 15–17 Platelet refractoriness Recombinant erythropoietin, incomplete evaluation of, 106–108 use of, 32 Platelet transfusion rates Recombinant factor VIIa for adults, 116 (rFVIIa), 91 for pediatric patients, 116 Red blood cells (RBCs) Platelet transfusions delayed hemolytic transfusion hemolysis following, 41–43 reactions (DHTRs) and, inappropriate, for patients on 25, 26 aspirin, 70–72 phenotype match in SCD, 76–78

Eichbaum_87963_PTR_IND_10-01-12_137-144.indd 140 11/10/12 12:21 PM Index 141

timely issue and delivery Thomsen-Hubener-Friedenrich of, 20–22 phenomenon. See T activation transfusion Thrombocytopenia, 106 risk of hyperkalemia from, differential diagnosis of, 73–75 124–126 Thrombotic thrombocytopenic Refractory thrombocytopenia, purpura (TTP), 127 causes of, 106 failure to diagnose, 109–111 Reverse type, 23, 28 Transfusion-associated circulatory Rh immune globulin (RhIG) overload (TACO), in patients calculation for, 11 with chronic anemia inadequate dosing, 9–11 failure to recognize the increased inappropriate use of, in risk of, 34–35 pregnancy, 7–8 Transfusion need for, 63–65 inappropriate application of RhD protein, molecular premedication for, 103–105 differences in, 63–65 liberal versus restrictive Rosette test, for FMH, 9 strategies, 59–60 Transfusion-related acute lung Sickle cell disease (SCD) injury (TRALI), failure to failure to recognize diagnose, 112–114 alloimmunization risk in chronically transfused Unexpected posttransfusion purpura patient, 76–78 (PTP), 73–75 Sodium citrate, 38 Streptococcus pneumoniae, 82, 83 Verify Now test, 71 Syncopal reactions in blood donors Vitamin K, 90–92 failure to recognize risk factors von Willebrand factor (vWF), 13 for, 85–86 von Willebrand factor receptor (GPIb), 16 T activation, 82 “10/30 rule,” 60 Warfarin (Coumadin) reversal Tensilon test, 44 for severe bleeding, failure to Therapeutic plasma exchange (TPE) recognize, 90–93 development of coagulopathy Warm autoimmune hemolytic secondary to, 44–45 anemia (WAIHA), 66 failure to assess metabolic Weak D/partial D antigen testing, alkalosis resulting from failure to recognize blood products during, limitations of, 63–65 127–129 Whole blood-derived platelet hypocalcemic toxicity from, 38–39 concentrates, 70

Eichbaum_87963_PTR_IND_10-01-12_137-144.indd 141 11/10/12 12:21 PM EEichbaum_87963_PTR_IND_10-01-12_137-144.inddichbaum_87963_PTR_IND_10-01-12_137-144.indd 142142 111/10/121/10/12 12:2112:21 PMPM EEichbaum_87963_PTR_IND_10-01-12_137-144.inddichbaum_87963_PTR_IND_10-01-12_137-144.indd 143143 111/10/121/10/12 12:2112:21 PMPM Eichbaum_87963_PTR_IND_10-01-12_137-144.indd 144 11/10/12 12:21 PM